PDA

نسخه کامل مشاهده نسخه کامل : اتاق ریاضیات(طرح سؤالات)



صفحه ها : [1] 2 3 4 5 6 7 8 9 10 11 12 13 14 15 16 17 18 19 20

mofidy1
19-04-2006, 00:05
بسم الله الرحمن الرحیم

با سلام خدمت دوستان خوب p30world

احساس قوی بنده - به عنوان یک معلم کوچک ریاضی- این است که در این سایت جای یک اتاق بحث و گفتگو پیرامون ریاضیات خالی است. فکر می کنم سرفصل مطالب مهم باید اینها باشد:

1. طرح و حل مسائل مختلف ریاضی در هر سطحی که باشد - دبیرستانی و دانشگاهی.

2. آموزش دروسی که معمولا دانش آموزان و دانشجویان به آن نیاز دارند.

3. ارائه نمونه سوالات امتحانی برای استفاده دانش آموزان و دانشجویان. (هر چند که تایپیکی در این زمینه در p30world
وجود دارد).

4. معرفی سایتهای خوب ریاضی- فارسی و غیر فارسی.

5. رفع اشکال دانش آموزان و دانشجویان و جواب به سوالات آنها.

6. ارائه تستهای کنکور کارشناسی و کارشناسی ارشد ریاضی برای کسانی که به این تستها دسترسی لازم ندارند.

7. معرفی و آموزش نرم افزارهای ریاضی مانند Maple- Mathematica- Matlab -...

8. مقالات خوب ریاضی در سطوح مختلف.

9. بحث پیرامون مسائل آموزشی کشور و جهان و بررسی نقاط ضعف قوت آن.

10. ...

================================================== =======

برای شروع، حل این مساله را به بحث می گذاریم:

با استفاده از اطلاعات معمولی ریاضی - در حد درس حسابان سوم ریاضی - ثابت کنید که


[ برای مشاهده لینک ، لطفا با نام کاربری خود وارد شوید یا ثبت نام کنید ]

(هدف ارائه راه حل ساده تری غیر از روش ارائه شده در حسابان است. در ضمن نمی خواهیم از قاعده هوپیتال استفاده کنیم.)

================================================== =======

منتظر مطالب دوستان هستیم

ارسال متن: 30 فروردین 1385

uhanna.marine
19-04-2006, 13:09
salam dooste aziz khob age az hopital ham estefadeh nakonim mitoonim az hamarzi estefade konim . midoonim ke lim sinx =0 vaghti xbe samte 0 meyl mikone .

uhanna.marine
19-04-2006, 13:13
ba lim x mosavie .

drweblog
19-04-2006, 13:24
با عرض سلام و خسته نباشيد
اولا دوست من بهتر بود شما قوانين مربوط به سايت رو مي خوندي تا متوجه بشي که در اين سايت حتما بايد فارسي تايپ کني
ثانيا ورود شما رو به پي سي ورلد تبريک مي گوييم
ثالثا برادر بزرگوار جناب آقاي مفيدي ( شايد هم خانم ) اگر در زمينه رياضيات احتياج به کمک داشتيد من در خدمتم
مرسسسسسسسسسيييييييي

Farid007
19-04-2006, 14:11
سلام آقاي مفيدي . خسته نباشيد . كار بسيار خوبي هست
من هم باهاتون موافقم

mofidy1
19-04-2006, 20:06
باسلام

از اینکه می بینم دوستان دعوت بنده رو این قدر سریع پذیرفتن بسیار خوشحالم. به جواب بعضی از دوستان می پردازیم:

جواب پست 2 و 3 : ببینید دوست عزیز این قضیه ای که اثباتش رو به بحث گذاشتیم، اسمش همون قانون هم ارزیه. ما که نمی
تونیم از خود قضیه برای اثبات خودش استفاده کنیم. در ضمن لطف کنید و فارسی تایپ کنید.

جواب پست 4: دوست من فکر می کنم بهترین کمک، پذیرفتن یکی از کارهای 9 گانه ایست که در بالا خدمتتان عرض کردم.
انشاءالله اگر این اتاق پا گرفت و پر طرفدار شد، یکی از بهترین کارها تقسیم مسو لیتهاست.

فکر می کنم لازم است در آینده روش مناسب تایپ ریاضی در صفحات وب را به بحث بگذاریم. برای مثال به دو صفحه زیر در همین رابطه مراجعه فرمایید:

[ برای مشاهده لینک ، لطفا با نام کاربری خود وارد شوید یا ثبت نام کنید ]

[ برای مشاهده لینک ، لطفا با نام کاربری خود وارد شوید یا ثبت نام کنید ]

mofidy1
19-04-2006, 20:12
معرفی سایتهای ریاضی

در این قسمت، سایتهای ریاضی را در دو قسمت فارسی و انگلیسی به بازدید کنندگان عزیز معرفی خواهیم کرد. با این توضیح مهم که معرفی آنها در اینجا لزوماً به معنی تایید مطالب مطرح شده در آنها نیست. با عرض معذرت از اینکه فعلاً نظم خاصی در این لیست مشاهده نمی کنید. در حال تنظیم لیست موضوعی بزرگی هستم که شامل حداقل هزار سایت ریاضی است. انشاءالله در صورت تکمیل خدمتتان تقدیم خواهد شد. از خوانندگان محترم خواهشمندم که در تکمیل این لیست به این حقیر یاری رسانند. متشکرم.

================================================== ============

سایتهای فارسی
==========

1. انجمن ریاضی ایران ([ برای مشاهده لینک ، لطفا با نام کاربری خود وارد شوید یا ثبت نام کنید ])


2. دروس ریاضی مقطع دبیرستان و پیش دانشگاهی و تستهای آنها ([ برای مشاهده لینک ، لطفا با نام کاربری خود وارد شوید یا ثبت نام کنید ])


3. آموزش حسابان سوم ریاضی ([ برای مشاهده لینک ، لطفا با نام کاربری خود وارد شوید یا ثبت نام کنید ])


4. زندگینامه دانشمندان ریاضی ([ برای مشاهده لینک ، لطفا با نام کاربری خود وارد شوید یا ثبت نام کنید ])


5. لبخند رياضي- وبلاگي در زمينه رياضيات و كاربردهاي آن ([ برای مشاهده لینک ، لطفا با نام کاربری خود وارد شوید یا ثبت نام کنید ])


6. نشریه ریاضیات مخصوص دانش آموزان دبیرستان و پیش دانشگاهی ([ برای مشاهده لینک ، لطفا با نام کاربری خود وارد شوید یا ثبت نام کنید ])


7. کتابخانه ریاضی و کامپیوتر، مخصوص دانشجویان ([ برای مشاهده لینک ، لطفا با نام کاربری خود وارد شوید یا ثبت نام کنید ])


8. انجمن ریاضی بسیج دانشجویی دانشگاه یاسوج ([ برای مشاهده لینک ، لطفا با نام کاربری خود وارد شوید یا ثبت نام کنید ])


9. سایت آموزش ریاضی- نمونه سوالات در سطوح مختلف ([ برای مشاهده لینک ، لطفا با نام کاربری خود وارد شوید یا ثبت نام کنید ])


10. سایت المپیاد ریاضی رشد ([ برای مشاهده لینک ، لطفا با نام کاربری خود وارد شوید یا ثبت نام کنید ])


11. سوالات ریاضی خود را در سطح دبیرستان و پیش دانشگاهی بپرسید و جواب آن را از کارشناسان تبیان دریافت کنید ([ برای مشاهده لینک ، لطفا با نام کاربری خود وارد شوید یا ثبت نام کنید ])


12. مجله ریاضی- مخصوص دبیرستانیها ([ برای مشاهده لینک ، لطفا با نام کاربری خود وارد شوید یا ثبت نام کنید ])


13. بانک سوالات المپیاد های ریاضی دانش آموزان ایران از سال ۱۳۶۲ تاکنون ([ برای مشاهده لینک ، لطفا با نام کاربری خود وارد شوید یا ثبت نام کنید ])


14.خانه ریاضیات (شبکه رشد، شبکه ملی مدارس ایران) ([ برای مشاهده لینک ، لطفا با نام کاربری خود وارد شوید یا ثبت نام کنید ] 8C%D8%A7%D8%B6%DB%8C%D8%A7%D8%AA)

================================================== ============

سایتهای انگلیسی
==========

1. پژوهشگاه دانشهای بنیادی ایران-بخش ریاضی ([ برای مشاهده لینک ، لطفا با نام کاربری خود وارد شوید یا ثبت نام کنید ])


2. جهان ریاضیات- سایتی وسیع و معتبر که استفاده از آن را به همه دانشجویان توصییه می کنیم. ([ برای مشاهده لینک ، لطفا با نام کاربری خود وارد شوید یا ثبت نام کنید ])


3. مسائل المپیادهای ریاضی آمریکا ([ برای مشاهده لینک ، لطفا با نام کاربری خود وارد شوید یا ثبت نام کنید ])


4. مسائل مسابقات ریاضی بین المللی ([ برای مشاهده لینک ، لطفا با نام کاربری خود وارد شوید یا ثبت نام کنید ])


5. سایتی شامل جوکهای ریاضی ([ برای مشاهده لینک ، لطفا با نام کاربری خود وارد شوید یا ثبت نام کنید ])


6. سایت یکی از اساتید ریاضی دانشگاه فیلادلفیای آمریکا- دروس مختلف ریاضی به صورت pdf ([ برای مشاهده لینک ، لطفا با نام کاربری خود وارد شوید یا ثبت نام کنید ])


7. نکات ریاضی - شامل مقالات کوتاه و بلند در شاخه های مختلف ریاضی ([ برای مشاهده لینک ، لطفا با نام کاربری خود وارد شوید یا ثبت نام کنید ])


8. سایت ریاضی دانشگاه MIT -معتبرترین دانشگاه جهان-شامل متن بعضی از دروس ارائه شده، به صورت pdf (راهنمایی: ابتدا روی یکی از دروس جدول Course Title کلیک کرده، سپس در صفحه جدید در قسمت Highlights of this course، یکی از کلمات مشخص شده را انتخاب کنید.) ([ برای مشاهده لینک ، لطفا با نام کاربری خود وارد شوید یا ثبت نام کنید ])


9.اینجا را کلیک کنید و ببینید کدام ریاضیدان امروز به دنیا آمده و کدام ریاضیدان امروز از دنیا رفته است.
([ برای مشاهده لینک ، لطفا با نام کاربری خود وارد شوید یا ثبت نام کنید ])


10. سایت بزرگ تاریخ ریاضیات - شامل زندگینامه ریاضیدانان مشهور تاریخ ، فهرست موضوعی ریاضیات و نیز فهرست بزرگی از منحنی های معروف ریاضی و ...
([ برای مشاهده لینک ، لطفا با نام کاربری خود وارد شوید یا ثبت نام کنید ])


11. مجموعه سوالات امتحانی دانشگاه مشهور و معتبر مک گیل McGill -
([ برای مشاهده لینک ، لطفا با نام کاربری خود وارد شوید یا ثبت نام کنید ])


12. سایت سوالات مسابقه مشهور پاتنام - putnam ([ برای مشاهده لینک ، لطفا با نام کاربری خود وارد شوید یا ثبت نام کنید ])


13. مجله ریاضی Notices of the AMS -یکی از معتبرترین مجلات ریاضی جهان
([ برای مشاهده لینک ، لطفا با نام کاربری خود وارد شوید یا ثبت نام کنید ])


14. یک مجله الکترونیکی ریاضی در سطح دبیرستانهای کانادا - Pi in the Sky Magazine ([ برای مشاهده لینک ، لطفا با نام کاربری خود وارد شوید یا ثبت نام کنید ])

P-Moradifar
20-04-2006, 13:39
با سلام. بسیار خوشحال شدم از اینکه یک اتاق ریاضی در انجمن بوجود امده امیدوارم در ان مطالب مفیدی گفته شود و از ان سود ببریم. در مورد سوال گفته شده باید بگویم بهتر بود برای شروع سوالی جالب تر طرح می کردید ولی خوب باز هم از شما تشکر می کنم.

P-Moradifar
20-04-2006, 13:44
:biggrin: برای تنوع این سوال هم بد نیست:
n نفر در یک اتاق قرار دارند. به چند حالت این n نفر می توانند با هم دوست شوند.
( :) از دوستان خواهش میکنم اگر کسی نمونه این سوال را قبلا مشاهده کرده به من اطلاع دهد)

P-Moradifar
20-04-2006, 13:52
:biggrin: این سوال هم برای دوست داران تابع و حساب:
تمامی اعداد حقیقی چون a را بیابید که به ازای آن تابع غیر ثابت f از مجموعه اعداد حقیقی به خودش وجود داشته باشد که: (f(a(x+y ))=f(x)+f(y

Farid007
21-04-2006, 16:33
فکر می کنم لازم است در آینده روش مناسب تایپ ریاضی در صفحات وب را به بحث بگذاریم. برای مثال به دو صفحه زیر در همین رابطه مراجعه فرمایید:

[ برای مشاهده لینک ، لطفا با نام کاربری خود وارد شوید یا ثبت نام کنید ]
سلام آقاي مفيدي . خسته نباشيد مساله هاي وبلاگ شما در فرمت GIF هستند و نوشته نيستند و در كل نميتوان در اينجا به اين صورت تايپ كرد و تا حدودي ميشود ...
مگر در فرمت عكس بسازيم ...
ممنون و موفق باشيد

mofidy1
21-04-2006, 18:08
جواب پست11:

آقا فرید سلام

منظور بنده ، نوشتن فرمولهای ریاضی به زبان معروف Tex، تبدیل آنها به فرمولهای معمولی و انتشار آن با فرمت GIF در صفحات وب است ، فرمتی که در اکثر سایتهای ریاضی از آن استفاده می شود- مگر اینکه از دستورات XHTML یا MATHML استفاده کنیم که اینجا جای آن نیست. اگر زحمتی نیست به پست 1 بنده برای فرمول sin(x)/x مراجعه کنید.

از بذل توجهتان ممنونم.

mofidy1
21-04-2006, 21:28
دوستان خوبم، به حل مساله ای که در پست 1 آورده ام ، می پردازیم. همانطور که در آنجا آورده ام سعی می کنیم روش حل، فقط در سطح حسابان سال سوم ریاضی و در حد امکان ساده تر باشد. این حقیر، روش ذکر شده در کتاب را به دلایلی مناسب دانش آموزان نمی دانم و به تجربه، حل زیر را برای تدریس مناسب تر می دانم:

تعریف کنید [ برای مشاهده لینک ، لطفا با نام کاربری خود وارد شوید یا ثبت نام کنید ]

لذا [ برای مشاهده لینک ، لطفا با نام کاربری خود وارد شوید یا ثبت نام کنید ] و در نتیجه تابع f صعودی است.
فرض کنید x>0. پس [ برای مشاهده لینک ، لطفا با نام کاربری خود وارد شوید یا ثبت نام کنید ] و لذا [ برای مشاهده لینک ، لطفا با نام کاربری خود وارد شوید یا ثبت نام کنید ]

به همین ترتیب می توان ثابت کرد که [ برای مشاهده لینک ، لطفا با نام کاربری خود وارد شوید یا ثبت نام کنید ]

حال تعریف کنید [ برای مشاهده لینک ، لطفا با نام کاربری خود وارد شوید یا ثبت نام کنید ]

با نکته ای که قبل از تعریف g گفته شد و به روش بالا ثابت کنید g صعودی است و لذا برای x های نامنفی می توان نوشت [ برای مشاهده لینک ، لطفا با نام کاربری خود وارد شوید یا ثبت نام کنید ] و در نتیجه [ برای مشاهده لینک ، لطفا با نام کاربری خود وارد شوید یا ثبت نام کنید ] و با فرض x>0 به دست می آوریم[ برای مشاهده لینک ، لطفا با نام کاربری خود وارد شوید یا ثبت نام کنید ] پس [ برای مشاهده لینک ، لطفا با نام کاربری خود وارد شوید یا ثبت نام کنید ]
حال با تغییر متغیر x=-u که x<0 و با استفاده از نتیجه بالا می توان گفت: [ برای مشاهده لینک ، لطفا با نام کاربری خود وارد شوید یا ثبت نام کنید ] که اثبات را کامل می کند.
---------------------------------------------------
منتظر نظرات و روشهای دیگر شما هستم.

Man Hunter
21-04-2006, 22:29
من هم ميتونم كمك كنم و آموزش محاسبات ذهني رو به شما رفيقام ياد بدم..انو بگم كه اين روش خودم هست و با آقاي بيات كه ميومد توي تلويزيون فرق ميكنه.درضمن..رابطه هاي قشنگي هم كه بين اعداد هست رو بهتون ميگم.
آما....آما ميريم سراغ عدد جادويي 9 .اگه تمام تشكيلات اينو ياد بگيرين...روي تمام اعداد مسلط ميشيد.
خوبي 9 اين هست كه توي هر عددي كه ضرب بشه...جمعش ميشه 9..باور ندارين؟امتحان كنين:
9×972=8748 --------8+7+4+8=15+4+8=1+5+4+8=6+4+8=10+8=1+0+8=9.
و....... :ohno: :ohno: :ohno: :ohno: :ohno: :ohno:
جالب,نيست؟براي شروع سعي كنيد از 9×1 .....تا 9×999 رو ياد بگيريد...حفظ نكنين.خودتون به يه قانون ميرسيد.

يا حق. ;)

Man Hunter
21-04-2006, 22:37
سلام...اين بار چندتا عدد تو هم ضرب كنيم ببينيم خدا چي ميخواد :biggrin:

243*16=324*12--->3888

6048*16=8064*12--->96768
بازم هست....فكر كنيد رابطه رو پيدا كنيد. :tongue: ;)
يا حق ;)

shabnam335
23-04-2006, 01:15
سوال واسه بچه هاي تجربي بذارين من هستم
ولي خدايي حسابان بلد نيستم

P-Moradifar
23-04-2006, 12:16
با سلام:
خوب! خانم شبنم و دوستان ديگر سوالي كه اقاي مفيدي احمدي و من نوشته ام هم در سطح بچه هاي تجربي و رياضي است. بسم الله... سوال را حل كنيد.

mofidy1
24-04-2006, 19:39
با سلام

سطح A

در مثلث ABC رابطه ی زیر برقرار است. سه زاویه A، B و C چند درجه هستند؟


[ برای مشاهده لینک ، لطفا با نام کاربری خود وارد شوید یا ثبت نام کنید ]


=================================

سطح B

فرض کنید M مرکز یک دایره و A و B دو نقطه روی دایره باشند که دو سر قطری از آن نیستند. مماسهایی که در A و B بر دایره رسم می شوند یکدیگر را در C قطع می کنند. فرض کنید CM دایره را در D قطع کند و مماس بر دایره در نقطه ی AC ،D و BC را به ترتیب در E وF قطع کند. ثابت کنید مساحت چهارضلعی ADBM میانگین هندسی مساحت مثلث ABM و مساحت چهارضلعی ACBM است.


=================================

سطح C

تعداد سه تایی های مرتب (x,y,z) را که x، y و z اعداد طبیعی هستند و در رابطه ی x+y+z=17 صدق می کنند، به دست آورید. اگر این سه عدد دو به دو متمایز باشند، تعداد این سه تایی های مرتب چقدر است؟



=================================

سطح ِD

فرض کنید


[ برای مشاهده لینک ، لطفا با نام کاربری خود وارد شوید یا ثبت نام کنید ]

حد زیر را محاسبه کنید:


[ برای مشاهده لینک ، لطفا با نام کاربری خود وارد شوید یا ثبت نام کنید ]

موفق باشید.

16 تیر 1386

-----حل مسائل هفته قبل----- ([ برای مشاهده لینک ، لطفا با نام کاربری خود وارد شوید یا ثبت نام کنید ])

leila*
25-04-2006, 12:56
يك نفر از دوستان به من بگه كه جواب انتگرال dx بهLnxچي ميشه؟

mofidy1
25-04-2006, 13:34
با سلام

سطح A

حاصل عبارت زیر را بدون استفاده از ماشین حساب، به طور دقیق به دست آورید:


[ برای مشاهده لینک ، لطفا با نام کاربری خود وارد شوید یا ثبت نام کنید ]


=================================

سطح B

فرض کنید x عددی ثابت و بزرگتر از 1 باشد. حد زیر را محاسبه کنید (منظور از [x] جزء صحیح x است):


[ برای مشاهده لینک ، لطفا با نام کاربری خود وارد شوید یا ثبت نام کنید ]


=================================

سطح C

فرض کنید n عددی طبیعی باشد. انتگرال معین زیر را محاسبه کنید (منظور از [x] جزء صحیح x است):


[ برای مشاهده لینک ، لطفا با نام کاربری خود وارد شوید یا ثبت نام کنید ]


=================================

سطح ِD

ثابت کنید برای هر عدد طبیعی n داریم:


[ برای مشاهده لینک ، لطفا با نام کاربری خود وارد شوید یا ثبت نام کنید ]

موفق باشید.

9 تیر 1386

با سلام

سطح A

روش امیر آقا که در پست 19 ([ برای مشاهده لینک ، لطفا با نام کاربری خود وارد شوید یا ثبت نام کنید ]) ارائه کردند، کاملاً درست است؛ از دوستان دیگه که در حل این مساله همکاری کرند تشکر می کنم. نام این دوستان در لیست سبز (!!) این تاپیک ثبت خواهد شد.

سطح B

روش امیر آقا صحیح است. به پست 24 ([ برای مشاهده لینک ، لطفا با نام کاربری خود وارد شوید یا ثبت نام کنید ]) مراجعه فرمایید. ایشان مساله را در حالت کلیتر حل کردند. از ایشان تشکر می کنم.

سطح C

روش امیر آقا درست است. به پست 20 ([ برای مشاهده لینک ، لطفا با نام کاربری خود وارد شوید یا ثبت نام کنید ]) مراجعه فرمایید.

سطح D

راه حل امیر آقا صحیح است. برای دیدن راه حل ایشان به پست 22 ([ برای مشاهده لینک ، لطفا با نام کاربری خود وارد شوید یا ثبت نام کنید ]) مراجعه فرمایید. باز هم از ایشان تشکر می کنم.

بنده مساله را به روش دیگری حل می کنم.

با توجه به شکلهای دوتابع (Ln(x و (Ln(x-1 و افراز بندی بازه ی [3,2n+1] به زیر بازه های مساوی با طول 2، داریم:


[ برای مشاهده لینک ، لطفا با نام کاربری خود وارد شوید یا ثبت نام کنید ]

پس از انتگرال گیری و ساده سازی و با استفاده از خاصیت صعودی اکید بودن تابع (exp(x به نامساویهای مطلوب مساله خواهیم رسید. (این مساله در مسابقه ی هفته ی یکی از دانشگاههای معتبر آمریکا مطرح شده بود.)

موفق باشید.

16 تیر 1386

mmsarani
25-04-2006, 13:43
خيلي ممنون كه همچين بخشي رو زديد و به رياضي اهميت داديد

Ehsan_1368
25-04-2006, 20:12
با سلام خدمت آقاي مفيدي و عرض خسته نباشيد خدمت شما معلم زحمت كش
وقتي كه فهميدم همچين تالاري باز شده خيلي خوشحال شدم شما كاري بسيار بزرگ كرده ايد
بريم سراغ جواب سوال
فكر مي كنم جواب سوال شما را مي توان از راه قضيه ي فشردگي
( ساندويچ ) حل كرد همان طور كه مي دانيد sinx را بين 1 و -1 قرار دهيد سپس 1 و -1 را بر x تقسيم كنيد آنگاه حد اطراف sinx را بدست مي آوريم و چون حد دو طرف sinx برابر با عدد 1 است بنابر قضيه فشردگي (ساندويچ) حد مساله شما هم برابر 1 مي باشد.
با تشكر از شما

mofidy1
25-04-2006, 21:35
جواب پست 22:

از اظهار لطفتان بسیار ممنونم.
دوست گرامی توجه کنید که استدلالتان درست نیست. وقتی طرفین را به x تقسیم می کنیم، عبارتی که به دست می آید معکوس x است که حد آن وقتی x به صفر میل می کند بی نهایت است نه یک. بنده این مساله را در پست 13 حل کرده ام و منتظر راه حل های دیگر دوستان هستم .

موفق باشید.

ali waezi
02-05-2006, 09:24
سلام من عضو جديدم.

eh_mn
03-05-2006, 01:17
لطفا نظرتان را راجع به این سوال بنویسید :
جدولی را در نظر بگیرید که سطرهای آن به ترتیب زیر ساخته می شوند:
سطر اول 10 عدد طبیعی دلخواه را می نویسیم. در سطر دوم زیر عدد اول تعداد تکرارهای آن در سطر اول را می نویسیم. به همین ترتیب سطر دوم را پر می کنیم. سطر های بعدی نیز بروش مشابه ساخته می شوند. نشان دهید سطرهای 4 و 5 حتما یکی هستند.
در حالت کلی اگر تعداد اعدادی که در سطر اول می نویسیم n تا باشد ، کدام سطرها مطمئنا مشابهند ؟

eh_mn
03-05-2006, 01:39
با عرض سلام و خسته نباشید.

در خط سوم اثبات بنظر می رسد که باید فرض کنیم x>0 چون در انتهای همین خط sin را بر x تقسیم کرده اید واین تنها در صورتی امکان پذیر است که x صفر نباشد.

در مجموع (بنظر این جانب) اثبات بسیار زیباست ...

موفق باشید....

eh_mn
03-05-2006, 01:47
با سلام و خسته نباشید خدمت آقای مفیدی.

لطفا در مورد روش تبدیل متن های نوشته شده با TeX به غالب GIF توضیح دهید.

با تشکر.

mofidy1
03-05-2006, 14:27
با سلام و خسته نباشید خدمت آقای مفیدی.

لطفا در مورد روش تبدیل متن های نوشته شده با TeX به غالب GIF توضیح دهید.

با تشکر.

با سلام

اگر با زبان TEX آشنایی دارید، کافیست به آدرس ذیل بروید و یک متن کوتاه را در پنجره سفید رنگ این صفحه، به زبان TEX بنویسید و دگمه Submit را فشار دهید و نتیجه را به قالب GIF در پایین این دگمه ببینید . روی این عکس کلیک راست کنید و آدرس عکس را با Copy Image Location در مرورگر Firefox و در مرورگر IE با رفتن به قسمت Properties کپی و آدرس را در مکان مناسب Paste کنید.
آدرس صفحه: [ برای مشاهده لینک ، لطفا با نام کاربری خود وارد شوید یا ثبت نام کنید ]

موفق باشید.

ارسال متن: 13 اردیبهشت 1385

eh_mn
03-05-2006, 22:14
ضمن تشکر از جناب آقای مفیدی بخاطر راهنماییشان.
لطفا نظر خود را راجع به راه حل اینجانب بگویید.
در حالتی که f ناپیوسته باشد بنده به نتیجه روشنی نرسیدم. ( فقط ضابطه f بر اعداد گویا بدست می آید).
فرض کنیم a عددی حقیقی باشد که تابع f با شرط مورد نظر برای آن موجود باشد. در اینصورت برای هر x و y حقیقی داریم


[ برای مشاهده لینک ، لطفا با نام کاربری خود وارد شوید یا ثبت نام کنید ]
یعنی


[ برای مشاهده لینک ، لطفا با نام کاربری خود وارد شوید یا ثبت نام کنید ]
و جواب این معادله تابع


[ برای مشاهده لینک ، لطفا با نام کاربری خود وارد شوید یا ثبت نام کنید ]
است که در آن


[ برای مشاهده لینک ، لطفا با نام کاربری خود وارد شوید یا ثبت نام کنید ]
از طرفی


[ برای مشاهده لینک ، لطفا با نام کاربری خود وارد شوید یا ثبت نام کنید ]
پس


[ برای مشاهده لینک ، لطفا با نام کاربری خود وارد شوید یا ثبت نام کنید ]

در نتیجه [ برای مشاهده لینک ، لطفا با نام کاربری خود وارد شوید یا ثبت نام کنید ]

توضیح : در اینجا برخی از مواردی که استفاده شده ولی مستقیما ذکر نشده است بیان می کنیم
با فرض [ برای مشاهده لینک ، لطفا با نام کاربری خود وارد شوید یا ثبت نام کنید ] داریم

[ برای مشاهده لینک ، لطفا با نام کاربری خود وارد شوید یا ثبت نام کنید ]
و در نتیجه [ برای مشاهده لینک ، لطفا با نام کاربری خود وارد شوید یا ثبت نام کنید ]
همچنین

[ برای مشاهده لینک ، لطفا با نام کاربری خود وارد شوید یا ثبت نام کنید ]

eh_mn
10-05-2006, 07:27
با سلام خدمت همه دوستان.
لطفا نظر خود را راجع به به این سوال بنویسید.
فرض کنیم n یک عدد طبیعی ( و از اینجا به بعد ثابت ) باشد . تابع پیوسته [ برای مشاهده لینک ، لطفا با نام کاربری خود وارد شوید یا ثبت نام کنید ] را طوری بیابید که برای هر x و y حقیقی داشته باشیم

[ برای مشاهده لینک ، لطفا با نام کاربری خود وارد شوید یا ثبت نام کنید ]
که در آن

[ برای مشاهده لینک ، لطفا با نام کاربری خود وارد شوید یا ثبت نام کنید ]

با آرزوی موفقیت برای شما.

Ali
13-05-2006, 01:26
دوستان تاپیک به انجمن موضوعات علمی منتقل شد :) .
موفق باشید .

Renjer Babi
14-05-2006, 06:40
بابا ایول کارت خیلی درسته ادامه بده

eh_mn
14-05-2006, 07:36
می خواهیم با استقرای قوی نشان دهیم که برای هر a و هر عدد طبیعی مانند n

[ برای مشاهده لینک ، لطفا با نام کاربری خود وارد شوید یا ثبت نام کنید ]
حکم برای n=1 برقرار است زیرا

[ برای مشاهده لینک ، لطفا با نام کاربری خود وارد شوید یا ثبت نام کنید ]
فرض کنیم حکم برای 1 ، 2 ، ... و k برقرار باشد . برای n=k+1 داریم

[ برای مشاهده لینک ، لطفا با نام کاربری خود وارد شوید یا ثبت نام کنید ]
در نتیجه حکم برای هر عدد طبیعی برقرار است !!!!!!

با آرزوی موفقیت برای همه دوستان

eh_mn
14-05-2006, 07:51
به آنچه که تناقض آمیز ، باورنکردنی و مخالف شهود باشد پارادوکس می گویند. برخی پارادوکس را جمله ای می دانند که درستی آن نادرستیش و نادرستیش درستیش را نتیجه دهد. یعنی اگر فرض کنیم جمله راست است به این نتیجه می رسیم که دروغ است و اگر دروغ باشد به این نتیجه می رسیم که راست است !!!!!
یکی از این پارادوکس ها جمله زیر است :

" هر چیزی ممکن است "
اگر جمله فوق درست باشد و هر چیزی ممکن باشد آنگاه غیر ممکن نیز ممکن خواهد شد پس چیزی هست که غیرممکن است و این تناقض است. بنابر این ، این جمله غلط است !!! و اگر جمله فوق نادرست باشد آنگاه چیزی هست که غیرممکن است و این بدان معناست که غیرممکن ، ممکن شده است و این تناقض درستی جمله فوق را نتیجه می دهد!!!

نظر شما چیست ؟

با آرزوی موفقیت برای همه دوستان

eh_mn
14-05-2006, 16:16
فرض کنید n نفر در یک مهمانی حضور دارند. ثابت کنید تعداد آشنایانِ حداقل دو نفر از این ها با هم برابر است !!! ( در تعداد آشنایان خود شخص را در نظر نگیرید )

eh_mn
15-05-2006, 18:45
برای حل این مسئله از برهان خلف استفاده کنید. فرض کنید تعداد آشنایان این افراد با هم متفاوت باشد و ...

موفق باشید

eh_mn
16-05-2006, 23:38
برای هر کدام از حاضرین در مهمانی یک عدد در نظر بگیرید که نشان دهنده تعداد آشنایان او باشد. با توجه به اینکه n نفر در مهمانی حضور دارند ( و هیچ کس آشنای خودش نیست) این عدد باید بین 0 تا با n-1 باشد. حال به برهان خلف فرض کنیم هیچ دونفری عدد یکسان ندارند. بنابراین باید n عدد متمایز از 0 تا با n-1 انتخاب کنیم. ( چون هر کدام از مهمانان باید یک عدد داشته باشند) پس در میان حاضرین یک نفر هست که عدد 0 و شخص دیگری هست که عدد n-1 را به خود اختصاص می دهد. اینکه یک نفر عدد n-1 را به خود اختصاص داده به چه معناست ؟ به این معناست که او با همه حاضرین آشناست. و این تناقض است زیرا ما نشان دادیم که شخصی وجود دارد که عدد او 0 است یعنی با هیچکس آشنا نیست.

موفق و پیروز باشید

eh_mn
16-05-2006, 23:45
پارادوکسی که برای شما بیان خواهم کرد به پارادوکس ریش تراش معروف است.

فرض کنید شهری آرایشگری دارد که فقط ریش کسانی را می تراشد که نمی توانند ریش خود را بتراشند. سوال اینست که ریش خود آرایشگر را چه کسی می تراشد ؟

خود آرایشگر نمی تواند ریش خود را بتراشد زیرا اگر خودش بتواند ریش خود را بتراشد آنگاه جزء کسانی نیست که می تواند ریش آنها را بتراشد. و اگر خود نتواند ریش خود را بتراشد آنگاه جزء کسانی است که باید ریش آنها توسط خودش تراشیده شود!!!

بنظر شما این مطلب عجیب نیست ؟؟؟

eh_mn
18-05-2006, 23:40
می خواهیم ریشه های معادله
[ برای مشاهده لینک ، لطفا با نام کاربری خود وارد شوید یا ثبت نام کنید ]
را بیابیم. چون عدد صفر در معادله صدق نمی کند پس x صفر نیست . لذا می توان طرفین را بر x تقسیم کرد . بنابراین داریم
[ برای مشاهده لینک ، لطفا با نام کاربری خود وارد شوید یا ثبت نام کنید ]
لذا
[ برای مشاهده لینک ، لطفا با نام کاربری خود وارد شوید یا ثبت نام کنید ]
یا جایگذاری در معادله اصلی داریم
[ برای مشاهده لینک ، لطفا با نام کاربری خود وارد شوید یا ثبت نام کنید ]
در نتیجه
[ برای مشاهده لینک ، لطفا با نام کاربری خود وارد شوید یا ثبت نام کنید ]
ودوباره به این دلیل که x صفر نیست می توان طرفین را در x ضرب کرد. در نتیجه x=1.!!!!!!
ولی همانطور که می بینید عدد 1 در معادله صدق نمی کند!!!!

موفق و پیروز باشید

eh_mn
19-05-2006, 23:03
فرض کنيد A ماتريس n*n باشد که مجموع درايه هاي هر سطر و هر ستون آن برابر يک است. دستگاه معادلات

[ برای مشاهده لینک ، لطفا با نام کاربری خود وارد شوید یا ثبت نام کنید ]
را در نظر بگيريد که در آن

[ برای مشاهده لینک ، لطفا با نام کاربری خود وارد شوید یا ثبت نام کنید ]
(یعنی X برداری ستونی با n سطر است)
نشان دهيد جواب اين دستگاه بصورت

[ برای مشاهده لینک ، لطفا با نام کاربری خود وارد شوید یا ثبت نام کنید ]
است.

eh_mn
19-05-2006, 23:11
فرض کنيد A ماتريس n*n باشد که مجموع درايه هاي هر سطر و هر ستون آن برابر يک است. دستگاه معادلات

[ برای مشاهده لینک ، لطفا با نام کاربری خود وارد شوید یا ثبت نام کنید ]
را در نظر بگيريد که در آن

[ برای مشاهده لینک ، لطفا با نام کاربری خود وارد شوید یا ثبت نام کنید ]
(یعنی X برداری ستونی با n سطر است)
نشان دهيد جواب اين دستگاه بصورت

[ برای مشاهده لینک ، لطفا با نام کاربری خود وارد شوید یا ثبت نام کنید ]
است.

Ehsan_1368
20-05-2006, 07:27
سلام آقاي مفيدي با عرض خسته نباشيد.
امروز به يه مشكلي برخورد كردم و از شما خواهش مي كنم مشكلم را حل كنيد:
سئوال: اگر a و b ريشه هاي معادله ي x^2 -4x+3=0 مطلوب است محاسبه ( a*SQR(b) + b*SQR(a
كه من نمي توانستم به زبان text تايپ كنم در اينجا معني علايم را مي نويسم
* : ضربدر
(SQR(b : راديكال b
( SQR(a : راديكال a
هر چه زودتر جواب بدين ممنون مي شوم /
با تشكر

mofidy1
20-05-2006, 10:10
با سلام

دوست عزیز توجه کنید که می توان نوشت:

[ برای مشاهده لینک ، لطفا با نام کاربری خود وارد شوید یا ثبت نام کنید ]

و در نتیجه

[ برای مشاهده لینک ، لطفا با نام کاربری خود وارد شوید یا ثبت نام کنید ]

که S مجموع و P حاصلضرب ریشه هاست. البته توجه کنید که اگر جواب حقیقی می خواهید، باید زیر رادیکالها همگی مثبت باشند. حال چون S=4 و P=3 پس هر دو ریشه مثبتند و جواب مساله عبارتست از:

[ برای مشاهده لینک ، لطفا با نام کاربری خود وارد شوید یا ثبت نام کنید ]

موفق باشید.

ارسال متن: 30 اردیبهشت 1385

eh_mn
21-05-2006, 02:00
فرض کنيد A ماتريس n*n باشد که مجموع درايه هاي هر سطر و هر ستون آن برابر يک است. دستگاه معادلات

[ برای مشاهده لینک ، لطفا با نام کاربری خود وارد شوید یا ثبت نام کنید ]
را در نظر بگيريد که در آن

[ برای مشاهده لینک ، لطفا با نام کاربری خود وارد شوید یا ثبت نام کنید ]
(یعنی X برداری ستونی با n سطر است)
نشان دهيد جواب اين دستگاه بصورت

[ برای مشاهده لینک ، لطفا با نام کاربری خود وارد شوید یا ثبت نام کنید ]
است.
متاسفانه اين راه حل يك اشكال بسيار بزرگ داشت.
نتيجه اخلاقي : به ازاي هر شخص ، آن شخص اين پست را نخوانده است!!!

ali_hp
22-05-2006, 13:55
سلام اقای مفیدی اثبات شما درست نیست زیرا درروند اثبات ازاین استفاده کرده اید که مشتق سینوس کسینوس می شود ودر بدست اوردن مشتق سینوس نیز ازحکمی که میخواستید اثبات کنید استفاده میشود!

ali_hp
22-05-2006, 14:06
در نتیجه گرفتن حکم استقرا از فرض استقرا از این استفاده کرده اید که aبه توان k-2 يك است در صورتي كه اگر k=1باشد k-2 بين 0,k نيست پس برايk=1 نه فرض استقرا ميگويد كه aبه توان k-2 برابر يك است نه پايه استقرا.

ali_hp
22-05-2006, 15:32
شما ثابت کرده اید که اگرعددي درمعادله ي مفروض صدق کند ان عدد در x^2=1/x نيز صدق مي كنداما ثابت نكرده ايدكه اگر عددي در اين برابري صدق كند درمعادله مفروض نيز صدق مي كند.
در حقيقت براي پيدا كردن مجموعه جوابهاي يك معادله بايد مجموعه اي معرفي كنيدكه:
1-هر عددي كه در معادله مفروض صدق مي كند در ان مجموعه باشد.
2-هر عضو ان مجموعه در معادله مفروض صدق كند.
(به بيان ديگر اگر از درستي گزارهpدرستي گزارهqرانتيجه گرفتيم لزومي ندارد كه برعكس ان نيز درست باشد)
و شما فقط گام اول را انجام داده ايد.

mofidy1
24-05-2006, 17:43
با سلام

علی آقا، از دقت نظرتان بسیار ممنونم. اشکال شما وقتی وارد است که تنها راه اثبات این مطلب که مشتق سینوس، کسینوس است، قضیه هم ارزی باشد. اما راههای مستقل و ساده تری وجود دارد که در جا ثابت می کند که هم مشتق سینوس، کسینوس است و هم مشتق کسینوس، منهای سینوس. این راه را به طور خلاصه توضیح می دهم.

متحرکی را در نظر بگیرید که با سرعت ثابت 1 روی دایره واحد با مرکز مبدا مختصات در جهت مثلثاتی حرکت کند. می دانیم که فرمول حرکت این متحرک برابر است با:

[ برای مشاهده لینک ، لطفا با نام کاربری خود وارد شوید یا ثبت نام کنید ]

که t متغیر زمان است. از طرف دیگر می دانیم که مشتق S در لحظه t ، برداری واحد و مماس بر مسیر حرکت، یعنی همان دایره است. با کمی دقت-در واقع با تشکیل دو مثلث قائم الزاویه همنهشت - می توان دید که

[ برای مشاهده لینک ، لطفا با نام کاربری خود وارد شوید یا ثبت نام کنید ]

لطفا اگر راه ساده تری سراغ دارید، مطرح فرمایید.

موفق باشید.

ارسال متن: 3 خرداد 1385

Ehsan_1368
25-05-2006, 14:16
با سلا م خدمت آقاي مفيدي
ضمن خسته نباشيد خدمت شما مي خواستم از شما خواهش كنم كه به سوال من پاسخ دهيد
سوال : اگر fو g معكوس پذير باشند آنگاه معكوس f را پيدا كنيد (f^-1)
f(2x) =5-g(1-x)/5 كه در اينجا / : كسر يا تقسيم است
/: كسر يا تقسيم
با تشكر

ali_hp
26-05-2006, 03:18
سلام اقاي مفيدي من نفهميدم چرا طول بردار سرعت واحد است.
البته با توجه به رابطه v=rwکه دران wسرعت زاویه ای(که در اینجا یک است )وrشعاع دایره (که ان هم در اینجا یک است )مقدار vهم که سرعت متحرک است یک بدست میاید.
رابطه بالامعادل این است که وقتی کهdtبه صفرمیل میکند:dp/dt=r*dh/dt
که در انd یعنی دلتا و dpجابجایی متحرک وhزاویه ای است که بردا ر مکا ن متحرک با محور طولها میسازد و*یعنی ضرب.از طرفی چون dp=2rsin(dh/2)l خاهیم داشت:
r*dh/dt=2r*sin(dh/2)/dt به سادگی میتوان دید که این هم معادل این است که وقتی dhبه صفر میل می کند(چون dtبه صفر میل میکند) داشته باشیم :

dh/2=sin(dh/2)l
لطفا توضيح دهيد كه شما چگونه بردار سرعت را واحد بدست اورده ايد.

اگر هم كمي درباره چگونگي اثبات رابطه هاي محيط ومساحت دايره توضيح دهيد خيلي ممنون ميشم.
(مخصوصامحيط چون نميدونم منظور از طول منحني چيست.)

mofidy1
26-05-2006, 11:58
با سلام

علی آقا طول بردار سرعت واحد است زیرا بنابر فرض متحرک ما با سرعت ثابت 1 در مسیر دایره ای مذکور در حرکت است. توجه کنید که اندازه بردار مماس بر منحنی نشان دهنده اندازه سرعت در آن نقطه است.

علی آقا دقت نظر شما در مطالعه مسائل این اتاق، بنده را بسیار خوشحال و امیدوار کرده است. اگر موافق باشید رسماً مسئولیت یکی از کارهایی را که در اولین پست این اتاق توضیح داده شده است، به عهده بگیرید تا انشاءالله این اتاق به یک اتاق خوب و پراستفاده برای بازدید کنندگان تبدیل شود. آیا موافق هستید؟

راستی، میزان تحصیلات شما چقدر و مکان تحصیل شما کجاست؟
منتظر پاسخ شما هستم.

موفق باشید.

ali_hp
26-05-2006, 12:35
اقا ي مفيدي شما در فرض شعاع را واحد و سرعت را واحد در نظر گرفته ايد در حقيقت سوال من اين است كه چرا با اين فرض ها معادله مكان اين چنين ميشود ( يعني چرا سرعت زاويه اي كه ضريب T است 1 ميشود ) .
من پیش دانشگاهی در رشته ریاضی و در مرکز پیش دانشگاهی صنیعی فر منطقه 15 تهران تحصیل میکنم .
با توجه به این که من دانشجو نیستم فکر میکنم بتوانم درطرح و حل بعضی از مسائل ریاضی و رفع اشكال در مباحثي كه نياز به اطلاعات زيادي ندارد در خدمت عزيزان باشم .

با تشكر و ارزوي موفقيت
علي حسين پوران

ali_hp
26-05-2006, 17:21
اقا یا خانمeh_mn شما در پست 29 دو حکم زیر را بدون استفاده از فرض پیوستگی ثابت کرده اید:
f(x+y)=f(x)+f(y)l,f(ax)=f(x)l
اگر در رابطه دوم قرار دهیم x=at-t,y=t(که t عدد دلخواهی است)داریم:
f(at-t+t)=f(at-t)+f(t)l,
وچونf(at-t+t)=f(at)=f(t)l پس برای هر tداریم:f((a-1)t)=0
اگرaیک نباشد برای هر kدلخواه اگر k/a-1را به جای tدر رابطه بدست امده قرار دهیم بدست می اید f(k)=0
واگر هم a=1 تابعf(x)=cx برای هر cمخالف صفر شرا یط مسا له را دارد .

SANR
27-05-2006, 22:51
:sad: سلام اقا من الان سه سالهكه پشت سر هم در درس رياضي نمره ي تك ميارم.
:whistle: امروز هم امتحان رياضي كشوري داشتم خيلي واسه ي من وديگر هم كلاسي ها سخت بود كه به سوالات پاسخ بدم

ممنون از تايپيكتون كه با حداقل هزينه به كمك هم وطنان با استعداد مي شتابيد.

eh_mn
28-05-2006, 22:08
اقا یا خانمeh_mn شما در پست 29 دو حکم زیر را بدون استفاده از فرض پیوستگی ثابت کرده اید:
f(x+y)=f(x)+f(y)l,f(ax)=f(x)l
اگر در رابطه دوم قرار دهیم x=at-t,y=t(که t عدد دلخواهی است)داریم:
f(at-t+t)=f(at-t)+f(t)l,
وچونf(at-t+t)=f(at)=f(t)l پس برای هر tداریم:f((a-1)t)=0
اگرaیک نباشد برای هر kدلخواه اگر k/a-1را به جای tدر رابطه بدست امده قرار دهیم بدست می اید f(k)=0
واگر هم a=1 تابعf(x)=cx برای هر cمخالف صفر شرا یط مسا له را دارد .

با سلام
احسنت . خیلی زیبا و جالب بود.
در واقع شما ثابت کردید که اگر a یک نباشد و قرار باشد f با خاصیت مذکور موجود باشد ، آنگاه f متحد با صفر است.

با تشکر

ali_hp
28-05-2006, 22:14
قابلی نداشت . من منظور سوال پست 30رو نفهمیدم لطفا بیشتر توضیح بدین

mofidy1
28-05-2006, 23:28
با سلام خدمت دوستان اتاق ریاضیات

خدا را شکر. احساس می کنم که این اتاق کم کم در حال تبدیل شدن به محلی با نشاط برای استفاده قشر وسیعی از دانش آموزان و دانشجویان است. در حدود 40 روزی که از افتتاح این اتاق می گذرد، به طور متوسط روزانه 35 بازدید از آن صورت گرفته است، با اینکه هنوز مطالب ارائه شده در آن خیلی زیاد نیست.

از امروز با اجازه شما بخش «مساله هفته » را آغاز می کنیم. شنبه ها مساله را طرح و جمعه ها حل کامل آن را ارائه خواهیم کرد. در طول هفته منتظر حل مساله از طرف بازدید کنندگان محترم خواهیم بود و روز جمعه، راه حلهای درست را مشخص می کنیم. توجه فرمایید که هدف، ارائه راه حلهای مختلف برای یک مساله است و فقط حل مساله هدف ما نیست.
از دوستان انتظار شرکت فعال در حل این مسائل داریم حتی اگر روشها متفاوت و طولانی باشد.

ذکر این نکته لازم است که «مساله خوب» مساله ای لزوما مشکل نیست. «مساله خوب» مساله ای است که حل آن به ایده های «زیبا» و «کلی» نیازمند باشد. بنابر این مساله های هفته، نه مشکل و نه آسانند. از طرف دیگر مسائل این بخش لزوما دبیرستانی نیستند، هر چند سعی خواهیم کرد که مسائل عمومی تر باشند.

مساله هفته اول:

فرض کنید:

[ برای مشاهده لینک ، لطفا با نام کاربری خود وارد شوید یا ثبت نام کنید ]

ثابت کنید:

[ برای مشاهده لینک ، لطفا با نام کاربری خود وارد شوید یا ثبت نام کنید ]

ارسال متن: 8 خرداد 1385

ali_hp
29-05-2006, 21:26
[ برای مشاهده لینک ، لطفا با نام کاربری خود وارد شوید یا ثبت نام کنید ]

mofidy1
29-05-2006, 21:32
سوالات سی‌امین دوره مسابقات ریاضی دانشجویی کشور، ۱۹ تا ۲۲ اردیبهشت ۱۳۸۵، دانشگاه تفرش (به همراه پاسخنامه) به همراه سوالات سالهای قبل: [ برای مشاهده لینک ، لطفا با نام کاربری خود وارد شوید یا ثبت نام کنید ]

ارسال متن: 8 خرداد 1385

ali_hp
29-05-2006, 21:33
با سلام اقای مفیدی مشکل من در باره پست 45 حل نشده است و هنوز نفهمیدم چرا معادله حرکت چنین میشود

mofidy1
29-05-2006, 23:41
با سلام

ببینید دوست عزیز، فرض کنید که متحرکی روی دایره واحد با سرعت واحد حرکت کند. بنابر این پس از t ثانیه، این متحرک t متر را می پیماید. بنابراین پس از 2pi ثانیه متحرک به نقطه آغاز می رسد و پس از pi/2 به نقطه (0و1) می رسد و ...
بنابر این تناظری یک به یک بین نقاط دایره و نقاط بین 0 و 2pi موجود است. از طرف دیگر فرمول S نشان دهنده جای متحرک روی منحنی است پس از t ثانیه. اما می دانید که هر نقطه روی دایره واحد را با زوج مرتبی نشان می دهند که طول آن کسینوس و عرض آن سینوس زاویه ای است که راس این زاویه در همان نقطه و ابتدای آن مبدا مختصات است.

ضمنا علی آقا مطالعه پست 58 شما مشکل است چون مطالب تقریبا در هم و بر هم است. بی زحمت به پست 28 مراجعه کنید و نمادها را آنگونه که در آنجا توضیح داده شده آماده کنید - همانند پستهای اینجانب و پستهای دوستمان eh_mn در تایپ مسائل. ممنونم.

موفق باشید.

ali_hp
30-05-2006, 21:22
باسلام سرعت خطي متحرك واحد است اما چرا متحرك پس t ثانيه tمتر از كمان دايره را مي پيمايد.
من با زبان texاشنايي ندارم.

mofidy1
30-05-2006, 22:51
با سلام

علی آقا در اینجا عبارت "متحرکی روی دایره با سرعت واحدحرکت می کند " به طور دقیقتر یعنی "سرعت زاویه ای آن واحد است" و این هم یعنی "بعد از t ثانیه ، متحرک t رادیان روی دایره حرکت کرده است" و این به طور شهودی همان است که در پست 61 توضیح دادم.

آقای حسين پوران، زبان tex - حداقل در مراحل ابتدایی آن - زبان پیشرفته و پیچیده ای نیست. به طور مثال برایتان دستورات تایپ دو فرمول اول و دوم پست 57 را با تصویر زیر توضیح می دهم. با دقت و تقلید از آنها متوانید فرمولهای بسیاری را تولید کنید. حداقل یکبار این کار را انجام دهید. (بی زحمت به پست 28 نیز مراجعه کنید.)

موفق باشید.

[ برای مشاهده لینک ، لطفا با نام کاربری خود وارد شوید یا ثبت نام کنید ]

ali_hp
31-05-2006, 12:15
خيلي ممنونم اقاي مفيدي من پست 58 ويرايش كردم.
لطفا يك بار ديگر به پستهاي 48و51و63 مراجعه كنيد . متحركي رادر نظر مي گيريم که روی دایره واحد در حرکت است .اگر wرا یک در نظر بگیریم چر ا طول بردار سرعت واحد می شود واگر vرا یک در نظر می گیریم چرا معادله حرکت به صورت زیر است.
s(t)=(sint,cost)l (که در حقیقت معادل این است که w یک باشد) اگر هم بخواهیم هر دو را یک در نظر بگیریم چراچنین متحرکی وجود دارد .
پست50درباره معادل بودن رابطه v=rwو قضیه هم ارزی است .
اینکه مشتق sinxدر نقطه صفر می شود cos0=1فقط جزیی از این است که مشتق sinxمی شود cosx
با این حال اگر تعریف مشتق را برای تابع sinxدر نقطه صفر بنویسیم می بینیم که این(منظور یک شدن مشتق sinxدر صفر است) معادل قضیه هم ارزی است.

mofidy1
02-06-2006, 15:33
با سلام خدمت دوستان اتاق ریاضیات

خدا را شکر. احساس می کنم که این اتاق کم کم در حال تبدیل شدن به محلی با نشاط برای استفاده قشر وسیعی از دانش آموزان و دانشجویان است. در حدود 40 روزی که از افتتاح این اتاق می گذرد، به طور متوسط روزانه 35 بازدید از آن صورت گرفته است، با اینکه هنوز مطالب ارائه شده در آن خیلی زیاد نیست.

از امروز با اجازه شما بخش «مساله هفته » را آغاز می کنیم. شنبه ها مساله را طرح و جمعه ها حل کامل آن را ارائه خواهیم کرد. در طول هفته منتظر حل مساله از طرف بازدید کنندگان محترم خواهیم بود و روز جمعه، راه حلهای درست را مشخص می کنیم. توجه فرمایید که هدف، ارائه راه حلهای مختلف برای یک مساله است و فقط حل مساله هدف ما نیست.
از دوستان - به ویژه eh_mn و ali_hp - انتظار شرکت فعال در حل این مسائل داریم حتی اگر روشها متفاوت و طولانی باشد.

ذکر این نکته لازم است که «مساله خوب» مساله ای لزوما مشکل نیست. «مساله خوب» مساله ای است که حل آن به ایده های «زیبا» و «کلی» نیازمند باشد. بنابر این مساله های هفته، نه مشکل و نه آسانند. از طرف دیگر مسائل این بخش لزوما دبیرستانی نیستند، هر چند سعی خواهیم کرد که مسائل عمومی تر باشند.

مساله هفته اول:

فرض کنید:

[ برای مشاهده لینک ، لطفا با نام کاربری خود وارد شوید یا ثبت نام کنید ]

ثابت کنید:

[ برای مشاهده لینک ، لطفا با نام کاربری خود وارد شوید یا ثبت نام کنید ]

ارسال متن: 8 خرداد 1385

با سلام

طبق قراری که گذاشتیم به حل مساله هفته اول می پردازیم و فردا مساله هفته دوم را مطرح خواهیم کرد. از دوست خوبم آقای حسين پوران که در پست 58 مساله را به طور کامل حل کردند متشکرم. در اینجا مساله ای مطرح می کنم که تعمیم مساله هفته اول است و سپس آنرا حل می کنم.

مساله: قرار دهید

[ برای مشاهده لینک ، لطفا با نام کاربری خود وارد شوید یا ثبت نام کنید ]

ثابت کنید:
[ برای مشاهده لینک ، لطفا با نام کاربری خود وارد شوید یا ثبت نام کنید ]


حل مساله:


[ برای مشاهده لینک ، لطفا با نام کاربری خود وارد شوید یا ثبت نام کنید ]

حال اگر به جای آلفا، 18 قرار دهید، x به دست آمده ، مساله هفته اول حل می شود.

موفق باشید.

ارسال متن: 12 خرداد 1385

mofidy1
03-06-2006, 00:42
مساله این هفته را مطرح و انشاءالله جمعه راه حل آن را ارائه می کنیم. منتظر راه حلهای بازدید کنندگان محترم هستیم.

مساله: مثلث زیر را در نظر بگیرید. زوایای PBC و PCA و PAB همگی مساوی و برابر با 30 درجه اند. ثابت کنید این مثلث متساوی الاضلاع است. توجه کنید که هیچ اطلاعی درباره مکان نقطه P نداریم.

ارسال متن: 13 خرداد 1385

[ برای مشاهده لینک ، لطفا با نام کاربری خود وارد شوید یا ثبت نام کنید ]

ali_hp
04-06-2006, 03:11
لم:در مثلث دلخواه ABCداریم: a/b=sin(A)/sin( B)l
اثبات:ارتفاع خارج شده از راس Cرارسم می کنیم وطول ان را hمینامیم داریم:sin(A)=h/a,sin(B)=h/b
از تقسیم دو رابطه بالا بر هم حکم بدست می اید.


فرض کنید PAC=X,PBA=X,PCB=Z پس داریم X+Y+Z=90.طبق لم بالا در سه مثلث کوچک داریم:

PAB: pA/PB=sin(Y)/sin(30)l
PBC: PB/PC=sin(Z)/sin(30)l
PCA: PC/PA=sin(X)/sin(30)l




با ضرب روابط بالا داریمsin(X)sin(Y)sin(Z)=1/8 .
فرض کنید x=y=z=30 نباشد .پس حداقل یکی از x,y,zبزرگتر 30 است(زیرا در غیر اینصورت داریم X+Y+Z<30+30+30=90) و حداقل یکی از انها کوچکتر از30 است(زیرا در غیر اینصورت داریم X+Y+Z>30+30+30=90) بدون کم شدن از کلیت مساله فرض کنید X>30وY<30 از بین X,Yهر کدام راکه فاصله کمتری از 30 داشت را به 30تبدیل کرده ودیگری را به اندازه فاصله زاویه ای که به 30 تبدیل کردیم با 30 ; به 30 نزدیک میکنیم.دو زاویه ای که بدین صورت بدست می اید یکی 30 است.اندازه دیگری را Wمی نامیم.با توجه به نحوه تبدیل X,Yبه این دو زاویه W+30=X+Yوتفاضل X,Yاز تفاضلW,30بیشتر است پس با توجه به رابطه
sin(p)sin(q)=(cos(p-q)-cos(p+q))/2 داریم : sin(X)sin(Y)<sin(W)sin(30)l پس
sin(X )sin(Y )sin(Z )<sin(W )sin(30)sin(Z )l
حال اگر WوZرا به 30 تبدیل کنیم مشابه بالا خواهیم داشت:sin(w)sin(z)<sin(30)sin(30)l
پس: sin(x)sin(y)sin(y)<1/8 این تناقض نشان می دهد که باید داشته باشیم X=Y=Z=30پس A=B=C=60

ali_hp
05-06-2006, 19:49
با سلام،اقای مفیدی من هنوز مشکلم (در پست 64 مطرح کرده ام) درباره اثبات شما بر طرف نشده است.وهنوز ان را نفهمیده ام.لطفا بیشتر توضیح دهید.

mofidy1
05-06-2006, 21:36
با سلام

علی آقا فکر می کنم در پستهای قبلی تقریبا به همه سوالات شما جواب دادم. سوالاتی که در پست 64 مطرح کردید تقریبا همان سوالات قبلی است. متاسفانه یکی از مشکلات بزرگ گفتگو در اینترنت این است که نمی توان در آن خیلی صریح و قانع کننده صحبت کرد به ویژه اگر این صحبتها پیرامون علم ظریف و دقیقی همچون ریاضیات باشد. اگر بخواهیم این بحثها را ادامه دهیم مشکلات زیادی برای بنده و شما ایجاد خواهد شد. یک پیشنهاد دارم. فرض می کنیم که مباحث قبلی برای شما قانع کننده نبوده است و در درستی آن شک دارید. هیچ اشکالی ندارد. با توجه به شناختی که از شما پیدا کرده ام مطمئنم که با تحقیق و مطالعه می توانید راه حل دیگری -البته فقط در حد درس حسابان سال سوم ریاضی- پیدا کنید. بنابر این سر فرصت شروع کنید به حل این مساله. بنده هم مشتاقانه منتظر راه حل جدید شما هستم. (البته به شرطی که مزاحم درس خواندن شما نشود چون این روزها روزهای سرنوشت سازی برای شماست.)

موفق باشید

ارسال متن: 15 خرداد 1385

mofidy1
09-06-2006, 21:57
مساله این هفته را مطرح و انشاءالله جمعه راه حل آن را ارائه می کنیم. منتظر راه حلهای بازدید کنندگان محترم هستیم.

مساله: مثلث زیر را در نظر بگیرید. زوایای PBC و PCA و PAB همگی مساوی و برابر با 30 درجه اند. ثابت کنید این مثلث متساوی الاضلاع است. توجه کنید که هیچ اطلاعی درباره مکان نقطه P نداریم.

ارسال متن: 13 خرداد 1385

[ برای مشاهده لینک ، لطفا با نام کاربری خود وارد شوید یا ثبت نام کنید ]

با تشکر از آقای حسين پوران که در پست 67 به حل مساله هفته دوم پرداختند. و اما می پردازیم به حل مساله که تقریباً شبیه راه حل آقای حسين پوران است. راه حلی که خدمتتان تقدیم میکنم متعلق است به دوست بسیار عزیزم آقای «حسین تیموری فعال» در مرکز تحصیلات تکمیلی در علوم پایه زنجان.

حل مساله:

به شکل زیر توجه کنید:

[ برای مشاهده لینک ، لطفا با نام کاربری خود وارد شوید یا ثبت نام کنید ]


فرض کنید

[ برای مشاهده لینک ، لطفا با نام کاربری خود وارد شوید یا ثبت نام کنید ]

بنابر قضیه سینوسها در مثلث داریم:


[ برای مشاهده لینک ، لطفا با نام کاربری خود وارد شوید یا ثبت نام کنید ]

لذا می توان نوشت:


[ برای مشاهده لینک ، لطفا با نام کاربری خود وارد شوید یا ثبت نام کنید ]

به همین ترتیب می توان ثابت کرد که:


[ برای مشاهده لینک ، لطفا با نام کاربری خود وارد شوید یا ثبت نام کنید ]

باضرب اینها درهم خواهیم داشت:


[ برای مشاهده لینک ، لطفا با نام کاربری خود وارد شوید یا ثبت نام کنید ]

و بنابراین


[ برای مشاهده لینک ، لطفا با نام کاربری خود وارد شوید یا ثبت نام کنید ]

درنتیجه:


[ برای مشاهده لینک ، لطفا با نام کاربری خود وارد شوید یا ثبت نام کنید ]

حال فرض کنید


[ برای مشاهده لینک ، لطفا با نام کاربری خود وارد شوید یا ثبت نام کنید ]

می توان دید که اگر x>0 آنگاه [ برای مشاهده لینک ، لطفا با نام کاربری خود وارد شوید یا ثبت نام کنید ] لذا


[ برای مشاهده لینک ، لطفا با نام کاربری خود وارد شوید یا ثبت نام کنید ]

که نتیجه می دهد:


[ برای مشاهده لینک ، لطفا با نام کاربری خود وارد شوید یا ثبت نام کنید ]

بد نیست بدانیم که این مساله قابل تعمیم به چند ضلعی های محدب است. به طور مثال یک چهار ضلعی محدب را با نقطه ای در درون آن در نظر بگیرید. از این نقطه به چهار راس چهار ضلعی وصل کنید به گونه ای که همانند مساله بالا یک در میان زاویه های مساوی اما در اینجا 45 درجه ایجاد شود. در اینصورت این چهار ضلعی باید یک مربع باشد. این مطلب برای چند ضلعی های بالاتر نیز برقرار است. حالت کلی مساله را به طور ساده می توان به صورت زیر بیان کرد:
در داخل یک n-ضلعی محدب، نقطه P را در نظر بگیرید و آنرا به همه رئوس وصل کنید. یکی از n مثلث به وجود آمده و یکی از زوایای غیر هم راس با P را انتخاب کنید. این زاویه را آلفا بنامید. حال در جهت مثلثاتی حرکت کنید و همه زوایای مثلثهای دیگر را هم که از لحاظ مکانی مشابه با این زاویه هستند (به شکل زیر توجه کنید) آلفا بنامید. ثابت کنید اگر همه این زوایا با هم برابر باشند و داشته باشیم:


[ برای مشاهده لینک ، لطفا با نام کاربری خود وارد شوید یا ثبت نام کنید ]

آنگاه این n-ضلعی، منتظم است.

[ برای مشاهده لینک ، لطفا با نام کاربری خود وارد شوید یا ثبت نام کنید ]


البته حل این مساله چندان آسان نیست. اگر به راه حل خوبی از این مساله کلی تر دسترسی پیدا کردید، خوانندگان این اتاق را بی نصیب نگذارید. متشکریم.

ارسال متن: 19 خرداد 1385

mofidy1
10-06-2006, 11:54
ثابت کنید شکل تابع زیر محور x ها را قطع نمی کند:


[ برای مشاهده لینک ، لطفا با نام کاربری خود وارد شوید یا ثبت نام کنید ]


(راه حل این مساله را روز جمعه این هفته ملاحظه خواهید کرد. منتظر راه حلهای دوستان عزیز هستیم.)

ارسال متن:شنبه 20 خرداد 1385

ali_hp
10-06-2006, 16:25
سلام
اگر در مساله هفته گذشته به جای مثلث nضلعی قرار دهیم وبه جای زاویه 30درجه [ برای مشاهده لینک ، لطفا با نام کاربری خود وارد شوید یا ثبت نام کنید ] راقرار دهیم باز هم حکم مساله درست است.(مشابه روشی که در پست 67بکار رفت ثابت می شود)
اقای مفیدی لطفا حالت کلی از مساله را که در نظرتان بوده است (برای5ضلعی و بالا تر) بنویسید.
متشکرم.

mofidy1
10-06-2006, 18:58
علی آقا سلام

امر شما اطاعت شد. به پست 70 مراجعه فرمایید.

موفق باشید.

eh_mn
12-06-2006, 00:39
با سلام خدمت دوستان و با تشکر از زحماتی که می کشید.
راه حل های ارائه شده برای مسئله هفته دوم بسیار زیبا هستند. با اینکه نتوانستم این مسئله را حل کنم ولی از این راه حلها بسیار لذت بردم.

موفق باشید

mofidy1
12-06-2006, 16:18
با سلام

دوستان در آدرس زیر، کتابی معرفی شده است که ارتباط بین ریاضیات و آیات قرآن کریم را بررسی می کند. شما را به مطالعه آن دعوت می کنم. البته معرفی آن دلیل بر موافقت صد در صدی با آن نیست. هم اکنون بعضی از مفسرین قرآن کریم با اینکه نظم در قرآن کریم را کاملا قبول دارند اما با اینگونه «كشفیات سيستم رياضی!» در قرآن کریم موافق نیستند و آنرا تفسیر به رای و بازی با آیات می دانند. هر چند که بعضی از دانشمندان آنرا یکی از نشانه های بزرگ اعجاز قرآن می دانند و در اینباره کتابهایی نیز تالیف کرده اند.

موفق باشید.

[ برای مشاهده لینک ، لطفا با نام کاربری خود وارد شوید یا ثبت نام کنید ]

eh_mn
13-06-2006, 10:39
ثابت کنید شکل تابع زیر محور x ها را قطع نمی کند:


[ برای مشاهده لینک ، لطفا با نام کاربری خود وارد شوید یا ثبت نام کنید ]


(راه حل این مساله را روز جمعه این هفته ملاحظه خواهید کرد. منتظر راه حلهای دوستان عزیز هستیم.)

ارسال متن:شنبه 20 خرداد 1385

با سلام و خسته نباشید

داریم

[ برای مشاهده لینک ، لطفا با نام کاربری خود وارد شوید یا ثبت نام کنید ]

فرض کنیم [ برای مشاهده لینک ، لطفا با نام کاربری خود وارد شوید یا ثبت نام کنید ] .
در اینصورت چون
[ برای مشاهده لینک ، لطفا با نام کاربری خود وارد شوید یا ثبت نام کنید ]
و
[ برای مشاهده لینک ، لطفا با نام کاربری خود وارد شوید یا ثبت نام کنید ]
پس x=-1 طول نقطه می نیمم تابع g است. یعنی
[ برای مشاهده لینک ، لطفا با نام کاربری خود وارد شوید یا ثبت نام کنید ]
حال فرض کنیم x یک عدد حقیقی باشد. اگر x=-1 آنگاه
[ برای مشاهده لینک ، لطفا با نام کاربری خود وارد شوید یا ثبت نام کنید ]
و اگر x برابر منفی یک نباشد آنگاه
[ برای مشاهده لینک ، لطفا با نام کاربری خود وارد شوید یا ثبت نام کنید ]
و بدین ترتیب حکم ثابت می شود.

موفق باشید

mofidy1
13-06-2006, 14:14
دوست گرامي eh_mn سلام

خيلي علاقمندم كه با شما بيشتر آشنا شوم، نام شريفتان، شغلتان ،ميزان تحصيلاتتان و مكان سكونتتان. با توجه به مسائل خوبي كه در اين اتاق مطرح كرده و به آن جواب داده ايد فكر مي كنم اين آشنايي لازم باشد.

منتظرتان هستم. موفق باشيد.

ali_hp
13-06-2006, 15:37
بدون شرح:

[ برای مشاهده لینک ، لطفا با نام کاربری خود وارد شوید یا ثبت نام کنید ]

ali_hp
13-06-2006, 18:59
سلام
ايا هر چند جمله اي بدون ر يشه حقيقي و با ضريب بزرگترين درجه مثبت را مي توان به صورت مجموع مربعات چند چند جمله اي نوشت ؟(كاري كه در پست قبل براي f(x)lانجام شد) لطفا اقای مفیدی و دیگر دوستان برای حل این مساله به من کمک کنند.
متشکرم.

mofidy1
13-06-2006, 20:41
با سلام

مشخص است که نه. به طور مثال چند جمله ای x^4+x^2+2 را که همه شرایط مذکور را دارد نمي توان به صورت مجموع مربعات چند تا چند جمله اي نوشت، اما سوال جالبی است. میتوان سوال کرد که کدام چند جمله ای های همواره مثبت را می توان به صورت مجموع مربعات چند تا چند جمله اي نوشت؟ فکر می کنم جواب به این سوال آنقدرها هم ساده نباشد.

موفق باشید.

eh_mn
13-06-2006, 22:23
دوست گرامي eh_mn سلام

خيلي علاقمندم كه با شما بيشتر آشنا شوم، نام شريفتان، شغلتان ،ميزان تحصيلاتتان و مكان سكونتتان. با توجه به مسائل خوبي كه در اين اتاق مطرح كرده و به آن جواب داده ايد فكر مي كنم اين آشنايي لازم باشد.

منتظرتان هستم. موفق باشيد.

با سلام.
آقای مفیدی از لطف شما متشکرم.
من احسان منبتی هستم دانشجوی سال سوم رشته ریاضی کاربردی دانشگاه فردوسی مشهد و در مشهد زندگی می کنم.

در مورد راه حل علی آقا هم باید بگویم که بسیار زیباست!!

با تشکر و آرزوی موفقیت برای همه دوستان

mofidy1
13-06-2006, 22:57
با سلام
آقای منبتی از تماستان بسیار ممنونم. از اینکه دوستان خوبی همچون شما در این اتاق پیدا کرده ام، خدا را شکر می گویم. از شما خواهشی دارم. خودتان می دانید که این اتاق راه درازی برای رسیدن به اهداف خود دارد و فقط با تلاش یک نفر به نتیجه نمی رسد. لطف کنید و اولین پست این اتاق را یکبار دیگر مطالعه فرمایید و یکی از کارهای مذکور در آنجا را یا هر کار دیگری که صلاح می دانید؛ رسما به عهده بگیرید و وقت و زمان آنرا دقیقا اعلام کنید. به طور مثال این حقیر بخش «مساله هفته» را به عهده گرفته ام و شنبه ها مساله را طرح و جمعه ها به آن پاسخ می دهم. اگر چنین زحمتی را تقبل کنید بسیار ممنون خواهم شد.

منتظر تماستان هستم.

ali_hp
14-06-2006, 00:03
با سلام

مشخص است که نه. به طور مثال چند جمله ای x^4+x^2+2 را که همه شرایط مذکور را دارد نمي توان به صورت مجموع مربعات چند تا چند جمله اي نوشت، اما سوال جالبی است. میتوان سوال کرد که کدام چند جمله ای های همواره مثبت را می توان به صورت مجموع مربعات چند تا چند جمله اي نوشت؟ فکر می کنم جواب به این سوال آنقدرها هم ساده نباشد.

موفق باشید.

[ برای مشاهده لینک ، لطفا با نام کاربری خود وارد شوید یا ثبت نام کنید ]
من فكر مي كنم جواب اين سوال مثبت باشد. وفكر مي كنم فقط وفقط چند جمله ايهايي را مي توان به صورت مجموع مربعات چند چند جمله اي نوشت كه ريشه حقيقي نداشته باشند و ضريب بزرگترين درجه انها مثبت باشد.

eh_mn
14-06-2006, 10:05
با سلام خدمت همه دوستان.
آقای مفیدی از میان مواردی که در پست 1 بود مورد مشخصی که من بتوانم آن را بطور کامل انجام دهم پیدا نکردم.
در هر موردی که نیاز به کمک باشد من تا جایی که بتوانم در خدمت شما ( و البته دوستان) هستم.

با آرزوی موفقیت برای شما.

eh_mn
14-06-2006, 10:15
با سلام.
نظرتان را راجع به این سوال بنویسید
کلیه چند جمله ای های مانند f را بیابید که

[ برای مشاهده لینک ، لطفا با نام کاربری خود وارد شوید یا ثبت نام کنید ]
.
موفق باشید

mofidy1
14-06-2006, 13:18
با سلام
آقای منبتی با توجه به رشته تحصیلی شما و دانشگاه معتبری که در آن تحصیل می کنید، پیشنهاد می کنم هر هفته به طور منظم نکته های کوتاه یا بلندی پیرامون کاربردهای ریاضیات در علوم مختلف مانند فیزیک، شیمی، مهندسی، اقتصاد، روانشناسی، پزشکی و ... در این اتاق بگنجانید. با نهایت تاسف ذهنیت بسیاری از مردم -حتی دانشجویان- نسبت به ریاضیات با آنچه باید باشد بسیار متفاوت است. این مطلب بسیار دردآور است اما متاسفانه حقیقت دارد.( بد نیست به آدرس [ برای مشاهده لینک ، لطفا با نام کاربری خود وارد شوید یا ثبت نام کنید ] پست سوم مراجعه فرمایید.) این مقالات باید در حد امکان کوتاه و ساده اما کاملا علمی و در حد فهم دانش آموزان سالهای آخر دبیرستان یا حداکثر اوایل دانشگاه باشند. با توجه به دانشگاه شما و نیز شهر بزرگی که در آن سکونت دارید، امکانات نسبتا خوبی در اختیار دارید. از اینترنت، کتابهای درسیتان ، حتی اساتیدتان نیز می توانید استفاده کنید. از نکات بسیار کوتاه اما جذاب شروع کنید و کم کم آنها را توسعه دهید. اگر چنین زحمتی را تقبل فرمایید، منت بزرگی بر ما و بازدید کنندگان این اتاق گذاشته اید. انشاءالله این زحمت را قبول خواهید کرد و با شناختی که از شما پیدا کرده ام مطمئن هستم که این کار برایتان مشکل نیست.

منتظر تماستان هستم. موفق و پیروز باشید.

ali_hp
14-06-2006, 23:05
سلام
اقای مفیدی لطفا نظر خود را در باره این اثبات از قضیه هم ارزی بنو یسید.و بگویید که درست است یا غلط.
مشتق تابع sinxدر صفر می شود cos0=1 پس طبق تعریف مشتق برای تابع sinxدر x=0داریم:
[ برای مشاهده لینک ، لطفا با نام کاربری خود وارد شوید یا ثبت نام کنید ]

ali_hp
14-06-2006, 23:18
سلام
اقای مفیدی لطفا نظر خود را در باره این اثبات از قضیه هم ارزی بگویید .ایا این اثبات درست است؟
مشتق تابع sinxدر x=0می شود cos0=1حال اگر تعریف مشتق را برای تابع sinxدرx=0بنویسیم خواهیم داشت:
[ برای مشاهده لینک ، لطفا با نام کاربری خود وارد شوید یا ثبت نام کنید ]

mofidy1
15-06-2006, 00:03
با سلام

علی آقا در واقع چیزی را ثابت نکرده اید. چرا مشتق تابع sinx در x=0می شود cos0=1 ؟ این همان سوالی است که قبلا هم از بنده پرسیده بودید و منجر به بحثهای طولانی شد.

در پست 83 نیز مشخص شد که شما اعداد را هم جزو چند جمله ایها فرض کرده اید (که البته مطلب غلطی نیست). بنده امروز مدتی روی سوال پست 79 شما کار کردم. تاکنون حتی با استفاده از رایانه مثال نقضی پیدا نکرده ام. اگر به نتیجه قطعی رسیدم اطلاع خواهم داد. شما هم روی این مطلب کار کنید. به نظر مطلب جالبی است چه درست باشد چه نباشد.

موفق باشید.

mehrdad21
15-06-2006, 08:16
سلام خسته نباشيد من به شخصه عاشق رياضياتم اميدوارم بتونم اينجا كمكي بتونم برسونم

يه مسئله
اگر
[ برای مشاهده لینک ، لطفا با نام کاربری خود وارد شوید یا ثبت نام کنید ]

[ برای مشاهده لینک ، لطفا با نام کاربری خود وارد شوید یا ثبت نام کنید ]

ali_hp
15-06-2006, 10:25
سلام
من به جواب سوالی که پرسیده بودم رسیدم در حقیقت ثابت کردم اگر چند جمله ای f(x)l با ضرایب حقیقی که ضریب بزرگترین درجه ان مثبت است ریشه حقیقی نداشته باشد انگاه چند جمله ایهای p(x),q(x)lو (با ضرایب حقیقی)وجوددارند که:

[ برای مشاهده لینک ، لطفا با نام کاربری خود وارد شوید یا ثبت نام کنید ]

mofidy1
15-06-2006, 10:36
سلام خسته نباشيد من به شخصه عاشق رياضياتم اميدوارم بتونم اينجا كمكي بتونم برسونم

يه مسئله
اگر
[ برای مشاهده لینک ، لطفا با نام کاربری خود وارد شوید یا ثبت نام کنید ]

[ برای مشاهده لینک ، لطفا با نام کاربری خود وارد شوید یا ثبت نام کنید ]

با سلام

آقا مهرداد ورودتان را به اتاق ریاضی خوش آمد می گوییم. بنده از اینکه ورودتان را با یک مساله آغاز کردید، از شما ممنونم.
دوست عزیز معمولا رسم این اتاق این است که اعضای آن خودشان را کاملا معرفی می کنند. نام کامل، میزان تحصیلات و رشته تحصیلی، مکان سکونت و شغل. اگر همدیگر را کاملا بشناسیم راحت تر و دقیقتر با یکدیگر ارتباط خواهیم داشت. پس منتظریم.

اما حل مساله شما:

از اتحاد زیر استفاده می کنیم:


[ برای مشاهده لینک ، لطفا با نام کاربری خود وارد شوید یا ثبت نام کنید ]

اگر طرفین تساوی داده شده شما را به توان 2 برسانیم خواهیم داشت:


[ برای مشاهده لینک ، لطفا با نام کاربری خود وارد شوید یا ثبت نام کنید ]

حال اگر a=sinx و b=cosx با جایگذاری در عبارت بالا به جواب زیر می رسیم:


[ برای مشاهده لینک ، لطفا با نام کاربری خود وارد شوید یا ثبت نام کنید ]

موفق باشید.

eh_mn
15-06-2006, 10:41
سلام
من به جواب سوالی که پرسیده بودم رسیدم در حقیقت ثابت کردم اگر چند جمله ای f(x)l با ضرایب حقیقی که ضریب بزرگترین درجه ان مثبت است ریشه حقیقی نداشته باشد انگاه چند جمله ایهای p(x),q(x)lو (با ضرایب حقیقی)وجوددارند که:

[ برای مشاهده لینک ، لطفا با نام کاربری خود وارد شوید یا ثبت نام کنید ]

:thumbsup: :thumbsup: :question: :question:

ali_hp
15-06-2006, 10:44
به نظر من هم در پست 87چیزی ثابت نشده است منظور من این بود که اگر بخوا هیم از اینکه مشتق sinxمی شود cosxاستفاده کنیم (که شما هم در پست 13 برای اثبات قضیه هم ارزی استفاده کرده اید ) می توا ن اثبات پست 87 را نیز ارایه کرد

mofidy1
15-06-2006, 10:51
با سلام

علی آقا مشتاقانه منتظر دیدن اثبات شما هستم.

آقای منبتی پیرامون درخواستم از شما در پست 86 فکری کردید؟

mehrdad21
15-06-2006, 11:26
با سلام

آقا مهرداد ورودتان را به اتاق ریاضی خوش آمد می گوییم. بنده از اینکه ورودتان را با یک مساله آغاز کردید، از شما ممنونم.
دوست عزیز معمولا رسم این اتاق این است که اعضای آن خودشان را کاملا معرفی می کنند. نام کامل، میزان تحصیلات و رشته تحصیلی، مکان سکونت و شغل. اگر همدیگر را کاملا بشناسیم راحت تر و دقیقتر با یکدیگر ارتباط خواهیم داشت. پس منتظریم.

اما حل مساله شما:

از اتحاد زیر استفاده می کنیم:


[ برای مشاهده لینک ، لطفا با نام کاربری خود وارد شوید یا ثبت نام کنید ]

اگر طرفین تساوی داده شده شما را به توان 2 برسانیم خواهیم داشت:


[ برای مشاهده لینک ، لطفا با نام کاربری خود وارد شوید یا ثبت نام کنید ]

حال اگر a=sinx و b=cosx با جایگذاری در عبارت بالا به جواب زیر می رسیم:


[ برای مشاهده لینک ، لطفا با نام کاربری خود وارد شوید یا ثبت نام کنید ]

موفق باشید.




سلام
شرمنده من با روال تاپيك زياد آشنا نبودم
نام: مهرداد رضائي
سن:23 سال
دانشجوي سال آخر مهندسي نرم افزار
محل سكونت : تهران


جواب سوال كاملا درسته اما بدون اتحاد كسي ميتونه حلش كنه ؟
مرسي از اينكه جواب داديد

آقاي مفيدي لطف كنيد براي علاقه مندان به شروع و تقويت پايه رياضيات كتا و منابع مفيد را معرفي كنيد
سپاسگزارم

mehrdad21
15-06-2006, 11:28
پيدا كردن لگاريتم در مبناي 10 به صورت ذهني

يك نكته كه در اين لگاريتم مهم است اينه كه جواب از 1 مي تونه باشه تا 9.999 پس در موقع جواب دادن به اين بازه فكر كنيد.
نكته ديگر اينكه به محضي كه عددي به شما داده شد اونقدر تمرين كنيد تا بتوانيد آن را به صورت استاندار د و علمي بنويسيد يعني كه عدد شما بايد از سمت چپ با مميز فاصله داشته باشد يا به عبارتي ديگر عدد صحيح آن يك رقمي باشد با مثال بيشتر توضيح ميدهم

براي ادامه كار سعي كنيد جدول زير را با تمرين فراوان مانند جدول ضرب به خاطر بسپاريد

log[1]= 0
log[2]= .30
log[3]= .48
log[4]= .60
log[5]= .70
log[6]= .78
log[7]= .85
log[8]= .90
log[9]= .95

مثال: عدد 29012 به ما داده شده و از ما لگاريتم آن خواسته شده .
ابتدا به اين شكل عمل ميكنيم كه آن را به صورت نماد علمي مينويسيم
2.9012 X 10^4

حالا قسمت حساس ماجرا ميرسد ما در اينجا مهمترين عنصري كه باهاش سرو كار داريم توان عدد 10 است
هر عددي كه در توان ما بود رو مينويسم و يك مميز ميزاريم براي ادامه جواب چون هنوز يه قسمت كوچولو مونده
در اين مثال .4 رو مينويسم سپس براي ادامه جواب به عدد علمي شده برميگرديم ميبينيم كه عدد 2.9012 به عدد 3 بسيار نزديك است خب log3 رو از جدولي كه گفتم حفظ بايد باشيم مينويسم خب لگاريتم 3 عدد 48. است پس اين عدد رو پشت سر مميز مينويسم يعني جواب نهايي ما ميشود
4.48
حالا ميريم ماشين حسابو برميداريم ميبينيم كه جواب دقيق 4.46 است كه جواب ما بسيار به اون نزديك است. خب اين روش تقريبي است اما از اونجايي كه حساب كردن لگاريتم مشكل است روش بسيار خوبيست.



بسياري از مسايلي كه در رياضي وجود دارند راه حل هاي ساده تري نيز دارند كه با كمي تحقيق ميتوانيد به بسياري از شيرينهاي نامتناهي رياضي دست پي ببريد.

mehrdad21
15-06-2006, 11:32
از عدد 100 تا 999 يا به عبارتي ديگر بين اعداد سه رقمي تنها 4 عدد و جود دارند كه جمع مكعبات آنها با خودشان برابر ميشود.
153 = 1x1x1 + 5x5x5 + 3x3x3 = 1 + 125 + 27
دو تاي ديگر نيز اعداد 370 و 407 هستند عدد 4 ام چه عددي است؟
( از روي يكي از ارقام داده شده به سادگي قابل حدسه)

ali_hp
15-06-2006, 11:50
همه ریشه هایf غیر حقیقی هستند از طرفی اگر عدد مختلطxریشه ای از f باشد مز دوج ان(که به خاطر غیر حقیقی بودن xبرابر xنیست) نیز ریشه ای از fهست پس تعداد ریشه های f زوج است فرض می کنیم 2kتا ریشه داشته باشیم x_1,x_2,....x_k,a_1,a_2,....a_kکه x_iمزدوجa_iاست
برای a>0ای داریم:
f(x)=(x-x_1)(x-x_2)...(x-x_k)(x-a_1)(x_a_2)...(x-a_k)a
عبارت
(x-x_1)(x-x_2)...(x-x_k) را بسط می دهیم وتاجای ممکن به جای i^2ها منفی یک قرار می دهیم حال ازهمه ی تک جمله ای های بر حسب xکه iدارند یک i فاکتور می گیر یم و عبارت مفروض را به صورت R(x)+S(X)i می نو یسیم می توا ن دید که اگر همین کار را با عبا رت (x-a_1)(x-a_2)...(x-a_k) انجام بدهیم حاصل می شودR(X)-S(X)i
به سادگی می توان دید که

[ برای مشاهده لینک ، لطفا با نام کاربری خود وارد شوید یا ثبت نام کنید ]

شرایط مساله را دارند .
خیلی بد نوشتم ولی امید وارم متوجه شوید به هر حال هر جا را که مبهم بود بگویید تا دوباره توضیح دهم.

ali_hp
15-06-2006, 11:56
از عدد 100 تا 999 يا به عبارتي ديگر بين اعداد سه رقمي تنها 4 عدد و جود دارند كه جمع مكعبات آنها با خودشان برابر ميشود.
153 = 1x1x1 + 5x5x5 + 3x3x3 = 1 + 125 + 27
دو تاي ديگر نيز اعداد 370 و 407 هستند عدد 4 ام چه عددي است؟
( از روي يكي از ارقام داده شده به سادگي قابل حدسه)
یک به توان سه برابر یک است پس عدد چهارم 371است.

eh_mn
15-06-2006, 19:05
با سلام
آقای مفیدی ، با پیشنهاد شما موافقم. امیدوارم بتوانم این کار را به نحو احسنت انجام دهم.
به نظر شما بهتر نیست که این مطالب با تاپیک جدید ارائه شوند. مثلا با عنوان کاربردهای ریاضی ، ریاضی و زندگی ، ...( و البته در آن تاپیک می توانیم به اتاق ریاضیات لینک بدهیم )

موفق باشید

eh_mn
15-06-2006, 19:09
با سلام.
نظرتان را راجع به این سوال بنویسید
کلیه چند جمله ای های مانند f را بیابید که

[ برای مشاهده لینک ، لطفا با نام کاربری خود وارد شوید یا ثبت نام کنید ]
.
موفق باشید

لطفا برای حل این سوال مرا راهنمایی کنید. تنها چیزی که برای f بدست آمد اینست که f تابعی زوج است.

با تشکر

eh_mn
15-06-2006, 19:15
با سلام
چگونه می توان مینیمم تابع زیر را بدست آورد . لطفا راهنمایی کنید.

[ برای مشاهده لینک ، لطفا با نام کاربری خود وارد شوید یا ثبت نام کنید ]
که در آن

[ برای مشاهده لینک ، لطفا با نام کاربری خود وارد شوید یا ثبت نام کنید ]
و

[ برای مشاهده لینک ، لطفا با نام کاربری خود وارد شوید یا ثبت نام کنید ]
.
با تشکر

Fermat
15-06-2006, 19:38
پارادکس ریشتراش نمونه ای ساده از پارادکس معروف راسل است. به هر حال این پارادکس جالب و در عین حال گیج کننده است.

mofidy1
15-06-2006, 20:45
با سلام
آقای مفیدی ، با پیشنهاد شما موافقم. امیدوارم بتوانم این کار را به نحو احسنت انجام دهم.
به نظر شما بهتر نیست که این مطالب با تاپیک جدید ارائه شوند. مثلا با عنوان کاربردهای ریاضی ، ریاضی و زندگی ، ...( و البته در آن تاپیک می توانیم به اتاق ریاضیات لینک بدهیم )

موفق باشید

با سلام
آقای منبتی، از اینکه زحمت را قبول فرمودید بسیار ممنونم. بنده با تاپیک جدید مخالفتی ندارم، هر چند بهتر می دانم که در همین جا باشد تا از پراکندگی مطالب ریاضی در سایت p30world جلوگیری شود. به تجربه در طول این یک سال و اندی عضویتم در p30world دریافته ام که جستجوی مطالب از طریق ماشین جستجوی سایت کار پر زحمت و وقت گیری است. منتهی برای اینکه بازدید کنندگان اتاق به علت پراکندگی مطالب به زحمت نیفتند مدتی است به فکر ایجاد یک فهرست موضوعی برای اتاق افتاده ام که دقیقا در اولین پست قرار می گیرد و تقریبا تمامی مباحث اتاق را هم پوشش میدهد. بنابر این نگران نباشید. البته اگر باز هم نظرتان تاپیک جدید است، همان کار را انجام دهید. در ضمن فکر می کنم همان عنوان کلی «کاربردهای ریاضی» کاملا مناسب باشد. باز هم خواهش می کنم در ارائه نکات کاملا دقت فرمایید. مقالات علمی، جذاب، کوتاه و در حد امکان ساده باشند (البته اگر امکان ساده بیان کردن مفاهیم نبود اشکالی ندارد) به گونه ای که خواننده را وادار به اظهار نظر کند. از لطفتان مجددا تشکر می کنم و موفق باشید.

ارسال متن: پنجشنبه 25 خرداد 1385

ali_hp
15-06-2006, 21:10
لطفا برای حل این سوال مرا راهنمایی کنید. تنها چیزی که برای f بدست آمد اینست که f تابعی زوج است.

با تشکر
سلام
راه حل من برای این مساله ساده نیست
باتوجه به رابطه ای که fدر ان صدق می کند می توان بدست اورد که اگر xریشه ای از f باشد (نه لزو ما حقیقی)
x^2+x+1,x^-x+1 نیز ریشه هایی از f هستند (با قرار دادن x,x-1در رابطه مساله) بعد هم ثابت کردم که در صفحه
ختلط فاصله حد اقل یکی از اعداد x^2-x+1,x^2+x+1 از مبدا بیش از فاصله xاز مبدا است و بدین ترتیب از روی یک ریشه برای f نا متناهی ریشه بدست اوردم پس اگر fریشه ای در اعداد مختلط داشته باشد بی نهایت ریشه دارد
و باید متحد باصفر باشد و اگر ریشه ای در اعداد مختلط نداشته باشد هم که باید متحد با عددی ثابت و مخا لف صفر
باشد.

ali_hp
15-06-2006, 21:34
سلام
راه حل من برای این مساله ساده نیست
باتوجه به رابطه ای که fدر ان صدق می کند می توان بدست اورد که اگر xریشه ای از f باشد (نه لزو ما حقیقی)
x^2+x+1,x^-x+1 نیز ریشه هایی از f هستند (با قرار دادن x,x-1در رابطه مساله) بعد هم ثابت کردم که در صفحه
ختلط فاصله حد اقل یکی از اعداد x^2-x+1,x^2+x+1 از مبدا بیش از فاصله xاز مبدا است و بدین ترتیب از روی یک ریشه برای f نا متناهی ریشه بدست اوردم پس اگر fریشه ای در اعداد مختلط داشته باشد بی نهایت ریشه دارد
و باید متحد باصفر باشد و اگر ریشه ای در اعداد مختلط نداشته باشد هم که باید متحد با عددی ثابت و مخا لف صفر
باشد.
ببخشید این راه حل مشکلی دارد که امید وارم سریعتر بر طرف شود

ali_hp
15-06-2006, 21:41
ببخشید این راه حل مشکلی دارد که امید وارم سریعتر بر طرف شود

مشکل راه حل بر طرف شد وهمه جوابها ی مساله عبارتند از
f(x)=a(x^2+1)^n
که aو nدلخواه اند.

eh_mn
15-06-2006, 21:42
ببخشید این راه حل مشکلی دارد که امید وارم سریعتر بر طرف شود

با سلام .
نظر من هم همين است. چون تابع

[ برای مشاهده لینک ، لطفا با نام کاربری خود وارد شوید یا ثبت نام کنید ]

در شرط مسئله صدق مي كند.

ولي اشتباه را پيدا نكردم!!!!

آقاي حسين پوران از توجه شما متشكرم.

موفق باشيد.

eh_mn
15-06-2006, 22:56
مشکل راه حل بر طرف شد وهمه جوابها ی مساله عبارتند از
f(x)=a(x^2+1)^n
که aو nدلخواه اند.

آقای حسین پوران اگر ممکن است در مورد راه حل توضیح دهید

متشکرم

ali_hp
15-06-2006, 23:37
سلام
راه حل من برای این مساله ساده نیست
باتوجه به رابطه ای که fدر ان صدق می کند می توان بدست اورد که اگر xریشه ای از f باشد (نه لزو ما حقیقی)
x^2+x+1,x^-x+1 نیز ریشه هایی از f هستند (با قرار دادن x,x-1در رابطه مساله) بعد هم ثابت کردم که در صفحه
مختلط فاصله حد اقل یکی از اعداد x^2-x+1,x^2+x+1 از مبدا بیش از فاصله xاز مبدا است و بدین ترتیب از روی یک ریشه برای f نا متناهی ریشه بدست اوردم پس اگر fریشه ای در اعداد مختلط داشته باشد بی نهایت ریشه دارد
و باید متحد باصفر باشد و اگر ریشه ای در اعداد مختلط نداشته باشد هم که باید متحد با عددی ثابت و مخا لف صفر
باشد.
ریشه های x^2+1=0را kوk- می نا میم برای هر عدد مختلط xکه x=a+bk تعریف می کنیم N(x)=a^2+b^2
(بخوانید نرم x) می توا ن ثابت کرد برای هر xغیر از k,-k نرم حد اقل یکی از اعداد x^2-x+1,x^2+x+1از نرم x بیشتر است پس اگر xریشه ای از f باشد و xغیر از k,-k باشد می توانیم از روی x ریشه ای با نرم بزرگتر بدست اوریم و این کار را تا جایی که به k,-k نرسیده ایم ادامه دهیم .}
بزرگترین nای را در نظر می گیریم که f بر x^2+1به توان n بخش پذیر باشد و.
f(x)=g(x)(x^2+1)^nبا قرار دادن این رابطه در رابطه داده شده برای f نتیجه می شود
g(x)g(x+1)=g(x^2+x+1)l (که رابطه ای مثل رابطه ای است که fدر ان صدق می کند ) اگر k یا k- ریشه g باشند می توان نشان داد دیگری نیز هست(زیراgدر رابطه ای مشابه با رابطه ای که برای fداشتیم صدق می کند پس اگر kریشه ان باشد k^-k+1 که برابر k- است نیز ریشه ان است
و اگر k- ریشه ان باشد به روش مشابه kریشه ان می شود ) در نتیجه g بر x^2+1
بخش پذیر می شود که با بزرگترین بودن n متناقض است .پس k,-k پس k,-kریشه های g نیستند و حالا می توانیم از رو ی یک ریشه g نا متناهی ریشه برای ان بدست اوریم.زیرا اگرx ریشه ای از g باشد x^2+x+1,x^2-x+1 نیز ریشه های g هستند وباتوجه به اینکه x نمی تواند kوk- نرم حد اقل یکی از اعداد x^2+x+1,x^-x+1 از نرم x بیشتر است
و بدین ترتیب توانستیم از روی ریشه ای دلخو ا ه g ریشه ای با نرم بزر گتر بدست اوریم پس اگر از یک ریشه g شروع
کنیم ری توانیم دنباله ای نامتناهی از ریشه ها بسازیم که نرمشان صعودی است پس هیچ دو تایی برابر نیستند .
پس g یا ریشه ندارد یا نامتناهی ریشه دارد.و در هر صورت بایددرجه اش0باشد(زیرا طبق قضیه اساسی جبر هر چند جمله ای از در جه mکه m>0 دقیقا m ریشه دارد . به صورت مقدماتی هم می توان با استفاده از مفاهیم بخش پذیری
ثابت کرد هر چند جمله ای از درجه m>0 حد اکثر mریشه دارد.)
پسg ثابت است .
پس همه جوابهای مسا له به صورت f(x)=a(x^2+1)^nهستند که با جا یگزاری در صورت مساله a=0,1و همه kهای صحیح جوابهای قابل قبول می دهند .
بسیاری از جزییات را ننوشتم هر جا مبهم بود بگویید تا بیشتر توضیح دهم

eh_mn
16-06-2006, 05:44
ریشه های x^2+1=0را kوk- می نا میم برای هر عدد مختلط xکه x=a+bk تعریف می کنیم N(x)=a^2+b^2
(بخوانید نرم x) می توا ن ثابت کرد برای هر xغیر از k,-k نرم حد اقل یکی از اعداد x^2-x+1,x^2+x+1از نرم x بیشتر است .پس اگر xریشه ای از f باشد و xغیر از k,-k باشد می توانیم از روی x ریشه ای با نرم بزرگتر بدست اوریم و این کار را تا جایی که به k,-k نرسیده ایم ادامه دهیم .

بزرگترین nای را در نظر می گیریم که f بر x^2+1به توان n بخش پذیر باشد و
f(x)=g(x)(x^2+1)^nبا قرار دادن این رابطه در رابطه داده شده برای f نتیجه می شود
g(x)g(x+1)=g(x^2+x+1)l اگر k یا k- ریشه g باشند می توان نشان داد دیگری نیز هست و در نتیجه g بر x^2+1
بخش پذیر می شود که با بزرگترین بودن n متناقض است .پس k,-k پس k,-kریشه های g نیستند و حالا می توانیم از رو ی یک ریشه g نا متناهی ریشه برای ان بدست اوریم و نتیجه بگیریمg ثابت است .
پس همه جوابهای مسا له به صورت f(x)=a(x^2+1)^nهستند که با جا یگزاری در صورت مساله a=0,1و همه kهای صحیح جوابهای قابل قبول می دهند .
بسیاری از جزییات را ننوشتم هر جا مبهم بود بگویید تا بیشتر توضیح دهم

با سلام
اگر ممکن است این قسمتهای را توضیح دهید

1.اگر k یا k- ریشه g باشند می توان نشان داد دیگری نیز هست
2. حالا می توانیم از رو ی یک ریشه g نا متناهی ریشه برای ان بدست اوریم
3. اگر یک چندجمله ای بینهایت ریشه داشته باشد آنگاه چندجمله ای ثابت است.

با تشکر

mehrdad21
16-06-2006, 12:08
كسي راهي براي محاسبه كسينوس 7 درجه بدون ماشين حساب بلده؟

mofidy1
16-06-2006, 13:35
كسي راهي براي محاسبه كسينوس 7 درجه بدون ماشين حساب بلده؟

با سلام

آقا مهرداد سلام. از اینکه از دیروز تا حالا شاهد چندین بار تماس شما بوده ایم بسیار خوشحالم. امیدوارم ارتباطتان را به همین ترتیب ادامه دهید. اما مساله شما:

7 درجه برابر است با 7*(Pi/ 180) رادیان. برای x های به حد کافی کوچک، x با sinx تقریبا برابر است. بنابراین سينوس 7 درجه تقریبا با
7*(Pi/ 180) برابر است. پس كسينوس 7 درجه برابر است با


[ برای مشاهده لینک ، لطفا با نام کاربری خود وارد شوید یا ثبت نام کنید ]

موفق باشید.

mehrdad21
16-06-2006, 13:40
با سلام

آقا مهرداد سلام. از اینکه از دیروز تا حالا شاهد چندین بار تماس شما بوده ایم بسیار خوشحالم. امیدوارم ارتباطتان را به همین ترتیب ادامه دهید. اما مساله شما:

7 درجه برابر است با 7*(Pi/ 180) رادیان. برای x های به حد کافی کوچک، sinx با x برابر است. بنابراین سينوس 7 درجه تقریبا با
7*(Pi/ 180) برابر است. پس كسينوس 7 درجه برابر است با


[ برای مشاهده لینک ، لطفا با نام کاربری خود وارد شوید یا ثبت نام کنید ]

موفق باشید.
مرسي من تازه با شماها آشنا شدم
از اين به بعد زياد مزاحمتون ميشم
مرسي از ايكه جواب دادي بسطي براي اينكار وجود نداره؟

mofidy1
16-06-2006, 13:56
چرا بسط معروف زیر را برای کسینوس داریم:


[ برای مشاهده لینک ، لطفا با نام کاربری خود وارد شوید یا ثبت نام کنید ]

البته در اینجا x به اندازه کافی به صفر نزدیک است.

ali_hp
16-06-2006, 14:23
با سلام
اگر ممکن است این قسمتهای را توضیح دهید

1.اگر k یا k- ریشه g باشند می توان نشان داد دیگری نیز هست
2. حالا می توانیم از رو ی یک ریشه g نا متناهی ریشه برای ان بدست اوریم
3. اگر یک چندجمله ای بینهایت ریشه داشته باشد آنگاه چندجمله ای ثابت است.

با تشکر

در همان پست 111راه حل را ویرایش کردم و این قسمتها را توضیح دادم.

ali_hp
16-06-2006, 14:29
همه ریشه هایf غیر حقیقی هستند از طرفی اگر عدد مختلطxریشه ای از f باشد مز دوج ان(که به خاطر غیر حقیقی بودن xبرابر xنیست) نیز ریشه ای از fهست پس تعداد ریشه های f زوج است فرض می کنیم 2kتا ریشه داشته باشیم x_1,x_2,....x_k,a_1,a_2,....a_kکه x_iمزدوجa_iاست
برای a>0ای داریم:
f(x)=(x-x_1)(x-x_2)...(x-x_k)(x-a_1)(x_a_2)...(x-a_k)a
عبارت
(x-x_1)(x-x_2)...(x-x_k) را بسط می دهیم وتاجای ممکن به جای i^2ها منفی یک قرار می دهیم حال ازهمه ی تک جمله ای های بر حسب xکه iدارند یک i فاکتور می گیر یم و عبارت مفروض را به صورت R(x)+S(X)i می نو یسیم می توا ن دید که اگر همین کار را با عبا رت (x-a_1)(x-a_2)...(x-a_k) انجام بدهیم حاصل می شودR(X)-S(X)i
به سادگی می توان دید که

[ برای مشاهده لینک ، لطفا با نام کاربری خود وارد شوید یا ثبت نام کنید ]

شرایط مساله را دارند .
خیلی بد نوشتم ولی امید وارم متوجه شوید به هر حال هر جا را که مبهم بود بگویید تا دوباره توضیح دهم.
چند تا از دوستان این راه حل را فهمیدند ؟ لطفا هر کسی که متوجه شده بگه.
متشکرم.

mofidy1
16-06-2006, 15:23
چند تا از دوستان این راه حل را فهمیدند ؟ لطفا هر کسی که متوجه شده بگه.
متشکرم.

علی آقا روش شما کاملا درست و بسیار زیباست. بنده تعمیم کوچکی از آنرا نیز به دست آورده ام که سر فرصت درباره آن صحبت خواهیم کرد.

ممنون و موفق باشید.

mofidy1
17-06-2006, 00:14
ثابت کنید شکل تابع زیر محور x ها را قطع نمی کند:


[ برای مشاهده لینک ، لطفا با نام کاربری خود وارد شوید یا ثبت نام کنید ]


(راه حل این مساله را روز جمعه این هفته ملاحظه خواهید کرد. منتظر راه حلهای دوستان عزیز هستیم.)

ارسال متن:شنبه 20 خرداد 1385

با سلام

از دو دوست عزیزم آقایان منبتی و حسین پوران که در پستهای 76 و 78 مساله هفته سوم را بسیار خوب و زیبا حل کردند متشکرم. بنده از روش دیگر مساله را حل و سپس حالت کلی مساله را بیان می کنم:

حل مساله:
تابع بالا را f بنامید. توجه کنید که اگر x منفی یا صفر باشد آنگاه


[ برای مشاهده لینک ، لطفا با نام کاربری خود وارد شوید یا ثبت نام کنید ]

حال فرض کنید x مثبت باشد. عبارت [ برای مشاهده لینک ، لطفا با نام کاربری خود وارد شوید یا ثبت نام کنید ] را محاسبه کنید و به دست آورید:


[ برای مشاهده لینک ، لطفا با نام کاربری خود وارد شوید یا ثبت نام کنید ]

که باز نتیجه می دهد [ برای مشاهده لینک ، لطفا با نام کاربری خود وارد شوید یا ثبت نام کنید ]

نکته: مشابه روش بالا می توان ثابت کرد که در حالت کلی تابع زیر، ریشه حقیقی ندارد:


[ برای مشاهده لینک ، لطفا با نام کاربری خود وارد شوید یا ثبت نام کنید ]

(آیا می توانید ثابت کنید مقدار مینیمم تابع بالا n+1 است؟ )

موفق باشید.

ارسال متن:شنبه 27 خرداد 1385

mofidy1
17-06-2006, 00:27
با سلام:

مساله:

فرض کنید a و b دو عدد مثبت باشند. مقدار انتگرال ناسره زیر را بیابید:


[ برای مشاهده لینک ، لطفا با نام کاربری خود وارد شوید یا ثبت نام کنید ]

موفق باشید.

ارسال متن:شنبه 27 خرداد 1385

eh_mn
17-06-2006, 10:37
با سلام
اگر ممکن است این قسمتهای را توضیح دهید

1.اگر k یا k- ریشه g باشند می توان نشان داد دیگری نیز هست
2. حالا می توانیم از رو ی یک ریشه g نا متناهی ریشه برای ان بدست اوریم
3. اگر یک چندجمله ای بینهایت ریشه داشته باشد آنگاه چندجمله ای ثابت است.

با تشکر

با سلام
آقای حسین پوران مشکل من در موارد 1 و 2 مرتفع شد.
اگر یک چندجمله ای نامتناهی ریشه داشته باشد آیا می تواند مخالف صفر باشد ؟
در واقع منظور من این است که تنها چند جمله ای که نامتناهی ریشه دارد چند جمله ی صفر است.

با تشکر

ali_hp
17-06-2006, 11:00
با سلام
آقای حسین پوران مشکل من در موارد 1 و 2 مرتفع شد.
اگر یک چندجمله ای نامتناهی ریشه داشته باشد آیا می تواند مخالف صفر باشد ؟
در واقع منظور من این است که تنها چند جمله ای که نامتناهی ریشه دارد چند جمله ی صفر است.

با تشکر
سلام
قضیه اساسی جبر: هر چند جمله ای که ضرایبش از اعداد مختلط باشند و از درجه m>0 باشد دقیقا m ریشه در مجموعه اعداد مختلط دارد.
بنا بر این نمی شود چند جمله ای نا متناهی ریشه داشته باشد و درجه اش مثبت باشد (زیرا اگر درجه اش مثبت باشد طبق قضیه بالا دقیقا به اندازه درجه اش ریشه خواهد داشت .)

البته راحت تر هم می توان نشان داد که اگر چند جمله ای بی نها یت ریشه داشته باشد متحد با صفر است
مثلا با استفاده از مفاهیم مر بوط به تقسیم در چند جمله ای هامی توا ن ثابت کرد هر چند جمله ای از درجه m>0حدکثر mریشه دارد.

eh_mn
17-06-2006, 11:18
با سلام

آقای حسین پوران از اینکه با حوصله به سوالات من پاسخ می دهید متشکرم.

دیگر هیچ ابهامی وجود ندارد.

موفق باشید.

ali_hp
19-06-2006, 16:55
سلام
برای اعدادطبیعی m,n,p,q می دانیم [ برای مشاهده لینک ، لطفا با نام کاربری خود وارد شوید یا ثبت نام کنید ]
ایا درست است که یا m=n یا m=p ؟
توضیح درباره سوال:
می دانیم اگر در اتم هيدروژن الکترونی از تراز انرژ ي nام به تراز انرژي mام برود بسامد موجی که گسیل میکند از رابطه [ برای مشاهده لینک ، لطفا با نام کاربری خود وارد شوید یا ثبت نام کنید ] بدست می اید که v بسامد وc عددی ثابت است .(منبع:كتاب درسي فيزيك پيش دانشگاهي رشته رياضي صفحات 213و214)
حالا سوال بالا معادل این است که ایا بسامد یک موج که از اتم هیدروژن گسیل شده به طور منحصر به فردی مشخص می کند که ان موج بر اثر جابجایی یک الکترون در بین کدام دو تراز ازاد شده است؟
احتمالا حل مساله ساده نیست ولي شايد بتوان براي ان مثال نقضي پيدا كرد(كه احتمالا مثال نقض نداشته باشد)
با توجه به صورت مساله احتمالا قبلا مطرح وتكليفش مشخص شده اگر كسي قبلا مطلبي در باره اين سوال ديده بگه

mofidy1
19-06-2006, 23:34
سلام
برای اعدادطبیعی m,n,p,q می دانیم [ برای مشاهده لینک ، لطفا با نام کاربری خود وارد شوید یا ثبت نام کنید ]
ایا درست است که یا m=n یا m=p ؟
توضیح درباره سوال:
می دانیم اگر در اتم هيدروژن الکترونی از تراز انرژ ي nام به تراز انرژي mام برود بسامد موجی که گسیل میکند از رابطه [ برای مشاهده لینک ، لطفا با نام کاربری خود وارد شوید یا ثبت نام کنید ] بدست می اید که v بسامد وc عددی ثابت است .(منبع:كتاب درسي فيزيك پيش دانشگاهي رشته رياضي صفحات 213و214)
حالا سوال بالا معادل این است که ایا بسامد یک موج که از اتم هیدروژن گسیل شده به طور منحصر به فردی مشخص می کند که ان موج بر اثر جابجایی یک الکترون در بین کدام دو تراز ازاد شده است؟
احتمالا حل مساله ساده نیست ولي شايد بتوان براي ان مثال نقضي پيدا كرد(كه احتمالا مثال نقض نداشته باشد)
با توجه به صورت مساله احتمالا قبلا مطرح وتكليفش مشخص شده اگر كسي قبلا مطلبي در باره اين سوال ديده بگه

با سلام

مثال نقض: m=5 n=6 p=9 q=90

ali_hp
20-06-2006, 00:30
با سلام

مثال نقض: m=5 n=6 p=9 q=90
متشکرم.

awqse
20-06-2006, 17:45
با سلام
اين مساله چگونه حل مي گردد
انتگرال جزء صحيح( ايكس تقسيم بر 2)منظور ايكس ضربدر يك دوم است كه در جزء صحيح قرار دارد

از منفي هفت تا مثبت هفت
با چه استدلالي بايد بازه ها را تفكيك كرد؟

ali_hp
21-06-2006, 02:13
با سلام
اين مساله چگونه حل مي گردد
انتگرال جزء صحيح( ايكس تقسيم بر 2)منظور ايكس ضربدر يك دوم است كه در جزء صحيح قرار دارد

از منفي هفت تا مثبت هفت
با چه استدلالي بايد بازه ها را تفكيك كرد؟
سلام
با توجه به دامنه تغییرات x مقدار x/2 بین 3.5و 3,5-قرار دارد پس[x/2] مقادیر صحیح بین 4-و4 را می پذیرد حال برای هر مقدار k که بین 4-,4 (kی صحیح) است همه مقادیر xرا که k=[x/2]l مشخص می کنیم بدین ترتیب دامنه تغییرات x را به قسمتهایی تقسیم کرده ایم(هر قسمت یک بازه می شود ) که در داخل انها مقدار [x/2] تغییری نمی کند و ثابت است.و با تفکیک انتگرال به چند انتگرال روی این بازه ها کافی است انتگرال تابع ثابت بر روی چند بازه را بدست اوریم
برای انکه بتوانیم درک بهتری از تابع f(x)=[g(x)]l داشته باشیم می توانیم به روش زیر مقدار fرا برحسب xارائه دهیم.
برای هرk صحیح همه مقادیر x را می یابیم که k=[g(x)]l که معادل این است که همه xهایی رابیا بیم که
k<=g(x)<x (=>یعنی کوچکتر مساوی)
وقتی مجموعه مقادیر ممکن برای g(x)l کراندار باشد فقط برای متناهی k ایکسی وجود دارد که k=[g(x)]l پس می توانیم fرا به صورت تابعی چند ضابطه ای بر حسب xبیان کنیم.

awqse
21-06-2006, 17:03
ممنون از شما
جزء صحيح 3.5 عدد سه است پس چگونه 4در با لا اورده شده
در ضمن اگر امكان دارد براي نمونه همين مساله را حل نماييد تا جواب اخر ان بدست ايد

mofidy1
21-06-2006, 18:14
جواب پست 26: دوست گرامی حق با شماست و متن ویرایش شد.

جواب پست 27: اگر با زبان TEX آشنایی دارید، کافیست به آدرس ذیل بروید و یک متن کوتاه را در پنجره سفید رنگ این صفحه، به زبان TEX بنویسید و دگمه Submit را فشار دهید و نتیجه را به قالب GIF در پایین این دگمه ببینید . روی این عکس کلیک راست کنید و آدرس عکس را با Copy Image Location در مرورگر Firefox و در مرورگر IE با رفتن به قسمت Properties کپی و آدرس را در مکان مناسب Paste کنید.
آدرس صفحه: [ برای مشاهده لینک ، لطفا با نام کاربری خود وارد شوید یا ثبت نام کنید ]

موفق باشید.

ارسال متن: 13 اردیبهشت 1385

با سلام خدمت دوستان عزیز

همانطور که ملاحظه می فرمایید در پست 28 درباره چگونگی تبدیل زبان TEX به فرمولهای مناسب GIF صحبت کردیم. بعضی از دوستان هم تاکنون از لینکی که در آنجا معرفی کردم برای فرمولها استفاده می کنند. نمی دانم چه مشکلی در سرور این سایت معروف پیش آمده است که تقریبا تمام فرمولهای آپلود شده از بین رفته اند. چند روزی صبر کردم اما این مشکل حل نشد و مجبور شدم با زحمت تمام همه فرمولها را یکبار دیگر تایپ و در فضای شخصی خودم در google آپلود کنم و پستهای قبل را هم تصحیح کنم تا مشکلی برای خوانندگان عزیز پیش نیاید. خواهشی که از دوستان دارم اینست که از سایت معرفی شده فقط برای تبدیل TEX به GIF استفاده کنند و تصویر به دست آمده را در فضای شخصی مطمئنی مانند google، yahoo یا جاهای دیگر آپلود کنند. همچنین از دوستان خوبم خواهشمندم که پستهای قبلی خودشان را به ترتیبی که عرض کردم تصحیح فرمایند. متشکرم.

ارسال متن: 31 خرداد 1385

mofidy1
23-06-2006, 15:51
با سلام:

مساله:

فرض کنید a و b دو عدد مثبت باشند. مقدار انتگرال ناسره زیر را بیابید:


[ برای مشاهده لینک ، لطفا با نام کاربری خود وارد شوید یا ثبت نام کنید ]

موفق باشید.

ارسال متن:شنبه 27 خرداد 1385

حل مساله:

انتگرال بالا را I بنامید. می توان دید که:


[ برای مشاهده لینک ، لطفا با نام کاربری خود وارد شوید یا ثبت نام کنید ]

و لذا


[ برای مشاهده لینک ، لطفا با نام کاربری خود وارد شوید یا ثبت نام کنید ]

موفق باشید.

ارسال متن: جمعه 2 تیر 1385

mofidy1
24-06-2006, 10:19
با سلام خدمت دوستان عزیز

مساله:

(الف) فرض کنید m و n دو عدد طبیعی باشند و n>1. ثابت کنید عدد زیر عددی اصم است:


[ برای مشاهده لینک ، لطفا با نام کاربری خود وارد شوید یا ثبت نام کنید ]

(ب) یک نقطه در صفحه را نقطه گویا ی نابدیهی گوییم اگر طول و عرض آن عدد گویای ناصفر باشد. فرض کنید x عددی با قدر مطلق کمتر یا مساوی 1 باشد و n عددی طبیعی بزرگتر یا مساوی 3 . ثابت کنید منحنی تابع زیر دارای نقطه گویا ی نابدیهی نیست:


[ برای مشاهده لینک ، لطفا با نام کاربری خود وارد شوید یا ثبت نام کنید ]

(توجه کنید که این دو مساله با وجود تفاوت ساختاری، تقریبا به یک روش حل می شوند.)

منتظر حل مساله توسط دوستان عزیز هستیم.

موفق باشید.

ارسال متن: شنبه 3 تیر 1385

mofidy1
01-07-2006, 10:44
دوستان عزیز سلام

به دلیل حال و هوای کنکوری و امتحانی روزهای قبل، حل مساله هفته پنجم را جمعه خدمتتان تقدیم می کنیم به این امید که دوستانی که به هر دلیل نتوانستند هفته گذشته به اتاق سر بزنند، روی این مساله و نیز مساله این هفته کار کنند. و اما مساله هفته ششم.

فرض کنید m و n اعداد طبیعی باشند. نامساوی زیر را ثابت کنید:


[ برای مشاهده لینک ، لطفا با نام کاربری خود وارد شوید یا ثبت نام کنید ]


ارسال متن: شنبه 10 تیر 1385

ali_hp
01-07-2006, 11:55
سلام
اگر عدد مورد نظر گویا باشد می توان ان را به صورت a/b که a,b دو عدد طبیعی ونسبت به هم اول هستند نشان داد به سادگی بدست می اید
[ برای مشاهده لینک ، لطفا با نام کاربری خود وارد شوید یا ثبت نام کنید ]

با توجه به رابطه بالا a مضربی از bاست پس چون a,b نسبت به هم اول هستند b=1 پس
[ برای مشاهده لینک ، لطفا با نام کاربری خود وارد شوید یا ثبت نام کنید ]
ولی چون n>1 داریم:

[ برای مشاهده لینک ، لطفا با نام کاربری خود وارد شوید یا ثبت نام کنید ] (برای اثبات مثلا میتوان از بسط دو جمله ای استفاده کرد)
پس خوا هیم داشت :
[ برای مشاهده لینک ، لطفا با نام کاربری خود وارد شوید یا ثبت نام کنید ]
که با طبیعی بودن a,m,m+1 متناقض است .

mofidy1
01-07-2006, 12:13
با سلام

علی آقا روش شما کاملا درست است اما سعی کنید هر دو را آنهم با روشی مشابه حل کنید. این روش برای دومی مناسب نیست. راستی از کنکور چه خبر؟

ali_hp
01-07-2006, 16:30
با سلام

علی آقا روش شما کاملا درست است اما سعی کنید هر دو را آنهم با روشی مشابه حل کنید. این روش برای دومی مناسب نیست. راستی از کنکور چه خبر؟
سلام
کنکورو خیلی بد تر از اون چیزی که حساب کرده بودم دادم مخصوصا در دروس اختصاصی .
سوال زیر یکی از سوالات ریاضی کنکور هست و جوابی که در کلید سازمان سنجش اعلام شده گزینه 4 است.
اگر متوجه سوال شده اید لطفا توضیح دهید.
تحت یک تبدیل،خط مفروض،با تبدیل یافته ان ، موازی است. در کدام حالت ، نوع تبدیل کاملا مشخص است؟
1) تجانس 2)دوران 3) بازتاب نسبت به نقطه 4) بازتاب نسبت به خط
قسمت ب سوال هفته:این قسمت معادل قضیه ی معروف فرما است .
قضیه فرما : اگر n>2 و n عددی طبیعی باشد اعداد طبیعی x,y,z وجود ندارند به طوری که :
[ برای مشاهده لینک ، لطفا با نام کاربری خود وارد شوید یا ثبت نام کنید ]
از قضیه فرما نتیجه می شود اگر n>2 اعداد گو یا و نا صفر x,y,z وجود ندارند که :
[ برای مشاهده لینک ، لطفا با نام کاربری خود وارد شوید یا ثبت نام کنید ]
در حقیقت اگر اعداد گو یاو ناصفر x,y,z در رابطه بالا صدق کننداز روی انها اعدادی طبیعی بدست می اید که در رابطه بالا صدق کنند که با قضیه فرما متناقض است.
اگر x=a/b,y=c/d,z=e/f که a,b,c,d,e,f ناصفر وصحیح
هستند با ضرب رابطه بالادر [ برای مشاهده لینک ، لطفا با نام کاربری خود وارد شوید یا ثبت نام کنید ] بدست می اید [ برای مشاهده لینک ، لطفا با نام کاربری خود وارد شوید یا ثبت نام کنید ] حال اگر فرض کنیم r=adf,s=cbf,t=ebd داریم [ برای مشاهده لینک ، لطفا با نام کاربری خود وارد شوید یا ثبت نام کنید ] اگر n زوج باشد هرکدام از اعداد r,s,t را که منفی بود با قرینه اش جایگزین می کنیم به وضوح جوابی طبیعی برای معادله بدست می اوریم .اگر n فرد باشد هرکدام از اعداد r,s,tرا که منفی بود مثلا x،[ برای مشاهده لینک ، لطفا با نام کاربری خود وارد شوید یا ثبت نام کنید ] را به طرف دیگر می بریم
وبا توجه به [ برای مشاهده لینک ، لطفا با نام کاربری خود وارد شوید یا ثبت نام کنید ] باز هم جوابی طبیعی برای مساله بدست می اوریم.
حال برای حل مساله به سادگی می توان دید که هر نقطه از تابع در رابطه [ برای مشاهده لینک ، لطفا با نام کاربری خود وارد شوید یا ثبت نام کنید ] صدق می کند پس با توجه به اینکه [ برای مشاهده لینک ، لطفا با نام کاربری خود وارد شوید یا ثبت نام کنید ] ممکن نیست x,f(x)l هم زمان گویا ناصفر باشند.
قسمت الف : در پست 135 به رابطه رسیدیم[ برای مشاهده لینک ، لطفا با نام کاربری خود وارد شوید یا ثبت نام کنید ] که طبق قضیه فرما برای n>2 غیر ممکن است ولی برای n=2 باید به طور جدا حکم را ثابت کرد .

mofidy1
01-07-2006, 17:38
با سلام

علی آقا فکر نمی کنم تست درستی باشد.

ali_hp
03-07-2006, 00:08
با سلام

علی آقا ظاهرا تبدیل هر خط تحت تجانس، بازتاب نسبت به نقطه و بازتاب نسبت به خط، خطی موازی آن است. فقط تبدیل یک خط تحت دوران ممکن است خطی موازی نتیجه ندهد. فکر نمی کنم تست درستی باشد.
سلام
تبدیل هر خط تحت تجانس و بازتاب نسبت به نقطه خطی موازی ان است ولی تبدیل یک خط تحت بازتاب نسبت به خط تنها در صورتی موازی ان است که خط اولیه با محور تبدیل موازی باشدو یا بر ان عمود باشد تبدیل یک خط تحت دوران تنها در صورتی موازی خودش است که زاویه دوران مضربی از پي باشد. من هر مفهومی که برای صورت سوال در نظر گرفتم سوال باتوجه به گزینه هایش درست نبود.

ali_hp
03-07-2006, 00:24
با سلام خدمت دوستان عزیز

همانطور که ملاحظه می فرمایید در پست 28 درباره چگونگی تبدیل زبان TEX به فرمولهای مناسب GIF صحبت کردیم. بعضی از دوستان هم تاکنون از لینکی که در آنجا معرفی کردم برای فرمولها استفاده می کنند. نمی دانم چه مشکلی در سرور این سایت معروف پیش آمده است که تقریبا تمام فرمولهای آپلود شده از بین رفته اند. چند روزی صبر کردم اما این مشکل حل نشد و مجبور شدم با زحمت تمام همه فرمولها را یکبار دیگر تایپ و در فضای شخصی خودم در google آپلود کنم و پستهای قبل را هم تصحیح کنم تا مشکلی برای خوانندگان عزیز پیش نیاید. خواهشی که از دوستان دارم اینست که از سایت معرفی شده فقط برای تبدیل TEX به GIF استفاده کنند و تصویر به دست آمده را در فضای شخصی مطمئنی مانند google، yahoo یا جاهای دیگر آپلود کنند. همچنین از دوستان خوبم خواهشمندم که پستهای قبلی خودشان را به ترتیبی که عرض کردم تصحیح فرمایند. متشکرم.

ارسال متن: 31 خرداد 1385
سلام
اقای مفیدی لینکی که معرفی کرده اید فیلتر شده است!(حداقل برای من فیلتر شده است)

mofidy1
03-07-2006, 14:22
سلام
اقای مفیدی لینکی که معرفی کرده اید فیلتر شده است!(حداقل برای من فیلتر شده است)

علی آقا به لینک زیر مراجعه کنید و نرم افزار MathType5.2 را دانلود و از آن برای فرمولسازی ریاضی - حتی فرمولهای بسیار پیشرفته- استفاده کنید. می توانید از این نرم افزار برای تبدیل یک فرمول به دستور TEX و نیز ساختن GIF از فرمولهای حروف چینی شده هم استفاده کنید.

[ برای مشاهده لینک ، لطفا با نام کاربری خود وارد شوید یا ثبت نام کنید ]

موفق باشید.

mirmohammadi
03-07-2006, 15:49
با سلام خدمت دوستان
راستش من تا حالا اين اتاق رو نديده بودم وگرنه حتماَ زودتر از اينها ميومدم
جيلي كار جالب و خوبي هست من هم تا حدي كه بتونم در فعال نگهداشتن اينجا كمك مي كنم
موفق باشيد

mofidy1
03-07-2006, 16:16
با سلام خدمت دوستان
راستش من تا حالا اين اتاق رو نديده بودم وگرنه حتماَ زودتر از اينها ميومدم
جيلي كار جالب و خوبي هست من هم تا حدي كه بتونم در فعال نگهداشتن اينجا كمك مي كنم
موفق باشيد

آقای میرمحمدی به اتاق ریاضیات خوش آمدید. لطفا خودتان را کامل معرفی کنید (میزان تحصیلات، رشته تحصیلی، شغل و محل سکونت) . یک مساله مناسب هم برای حل کردن در اختیار اتاق قرار دهید.

ضمنا از حدیث زیبا و پرمعنای آخر نوشته هایتان هم ممنونم.

موفق باشید.

mirmohammadi
04-07-2006, 12:08
سلام
نام: سيد حسين مير محمدي. تحصيلات: كارشناسي رياضي(محض). شغل: فعلاَ سرباز ;) . محل خدمت: تهران :blush: .
از فردا هم سعي مي كنم مساله براي حل بدم.
راستي آقاي مفيدي نرم افزاري كه تو پست قبلي معرفي كرديد خيلي خوب بود فقط اگر كرك اون رو هم داريد يه جا بگذاريد تا استفاده كنيم
خيلي ممنون
خداحافظ

ali_hp
04-07-2006, 20:34
سلام
حد عبارت زير وقتي n به سمت بي نهايت ميل مي كند چيست؟
l(1-1/2)(1-1/4)...(1-1/2^n)l
خيلي روش فكر كردم ولي حل نشد.اميدوارم دوستان براي حل اين مساله به من كمك كنند.

mofidy1
05-07-2006, 11:06
سلام
حد عبارت زير وقتي n به سمت بي نهايت ميل مي كند چيست؟
l(1-1/2)(1-1/4)...(1-1/2^n)l
خيلي روش فكر كردم ولي حل نشد.اميدوارم دوستان براي حل اين مساله به من كمك كنند.

با سلام

علی آقا به لینک زیر مراجعه کنید. در این زمینه بحثی مختصر وجود دارد:

[ برای مشاهده لینک ، لطفا با نام کاربری خود وارد شوید یا ثبت نام کنید ]

موفق باشید.

ali_hp
06-07-2006, 11:01
سلام
نامساوي داده شده معادل نامساوي زير است
c(m+n,m)(m^m)(n^n)<(m+n)^(n+m)l
كه اين هم با توجه به بسط دو جمله اي واضح است. در حقيقت سمت چپ يك جمله از m+n+1 جمله مثبتي است كه جمع همه انها برابرسمت راست نامساوي مي شود.

mofidy1
08-07-2006, 07:42
با سلام خدمت دوستان عزیز

مساله:

(الف) فرض کنید m و n دو عدد طبیعی باشند و n>1. ثابت کنید عدد زیر عددی اصم است:


[ برای مشاهده لینک ، لطفا با نام کاربری خود وارد شوید یا ثبت نام کنید ]

(ب) یک نقطه در صفحه را نقطه گویا ی نابدیهی گوییم اگر طول و عرض آن عدد گویای ناصفر باشد. فرض کنید x عددی با قدر مطلق کمتر یا مساوی 1 باشد و n عددی طبیعی بزرگتر یا مساوی 3 . ثابت کنید منحنی تابع زیر دارای نقطه گویا ی نابدیهی نیست:


[ برای مشاهده لینک ، لطفا با نام کاربری خود وارد شوید یا ثبت نام کنید ]

(توجه کنید که این دو مساله با وجود تفاوت ساختاری، تقریبا به یک روش حل می شوند.)

منتظر حل مساله توسط دوستان عزیز هستیم.

موفق باشید.

ارسال متن: شنبه 3 تیر 1385


دوستان عزیز سلام

به دلیل حال و هوای کنکوری و امتحانی روزهای قبل، حل مساله هفته پنجم را جمعه خدمتتان تقدیم می کنیم به این امید که دوستانی که به هر دلیل نتوانستند هفته گذشته به اتاق سر بزنند، روی این مساله و نیز مساله این هفته کار کنند. و اما مساله هفته ششم.

فرض کنید m و n اعداد طبیعی باشند. نامساوی زیر را ثابت کنید:


[ برای مشاهده لینک ، لطفا با نام کاربری خود وارد شوید یا ثبت نام کنید ]


ارسال متن: شنبه 10 تیر 1385

با سلام خدمت دوستان عزیز

از آقای علی حسین پوران به خاطر حل مسائل هفته های قبل تشکر می کنم.

================================================== ===============
حل مسائل هفته پنجم:

آخرین قضیه فرما یا FLT بیان میکند که


[ برای مشاهده لینک ، لطفا با نام کاربری خود وارد شوید یا ثبت نام کنید ]

به راحتی می توان ثابت کرد که مساله بالا معادل مطلب زیر است:


[ برای مشاهده لینک ، لطفا با نام کاربری خود وارد شوید یا ثبت نام کنید ]

حل مساله (الف): توجه کنید که اگر n=2 ، آنگاه می توان ثابت کرد


[ برای مشاهده لینک ، لطفا با نام کاربری خود وارد شوید یا ثبت نام کنید ]

فرض کنید n بزرگتر یا مساوی 3 باشد. به برهان خلف عمل می کنیم:


[ برای مشاهده لینک ، لطفا با نام کاربری خود وارد شوید یا ثبت نام کنید ]

که این مخالف قضیه فرماست.

حل مساله (ب):

فرض کنیم (x,y) نقطه گویای نابدیهی روی تابع f باشد. با جایگذاری مقادیر در تابع f و رساندن طرفین به توان n و تنظیم آن به عبارت معادل دوم از قضیه فرما می رسیم که باز تناقض است.

================================================== ===============

حل مساله هفته ششم:

می توان دید


[ برای مشاهده لینک ، لطفا با نام کاربری خود وارد شوید یا ثبت نام کنید ]

حال اگر جمله سمت راست عبارت آخر را بسط دهید، عبارت سمت چپ یکی از جملات آن است.

ارسال متن: شنبه17 تیر 1385

mofidy1
08-07-2006, 09:26
با سلام

فرض کنید a, b , c , d همگی مثبت باشند. نامساوی زیر را ثابت کنید:


[ برای مشاهده لینک ، لطفا با نام کاربری خود وارد شوید یا ثبت نام کنید ]


=================================
مساله بالا به زبان tex برای دوستانی که تصویر بالا را مشاهده نمی کنند.
این کار برای دوستانی که تصویر بالا را می بینند اما با زبان tex آشنایی کافی ندارند،
فرصت خوبی است تا دستورات این زبان را فراگیرند.


\frac{{a^3 + b^3 + c^3 }}
{{a + b + c}} + \frac{{b^3 + c^3 + d^3 }}
{{b + c + d}} + \frac{{c^3 + d^3 + a^3 }}
{{c + d + a}} + \frac{{d^3 + a^3 + b^3 }}
{{d + a + b}} \geq a^2 + b^2 + c^2 + d^2


==============================
ارسال متن: شنبه17 تیر 1385

ali_hp
08-07-2006, 17:49
با سلام خدمت دوستان عزیز

همانطور که ملاحظه می فرمایید در پست 28 درباره چگونگی تبدیل زبان TEX به فرمولهای مناسب GIF صحبت کردیم. بعضی از دوستان هم تاکنون از لینکی که در آنجا معرفی کردم برای فرمولها استفاده می کنند. نمی دانم چه مشکلی در سرور این سایت معروف پیش آمده است که تقریبا تمام فرمولهای آپلود شده از بین رفته اند. چند روزی صبر کردم اما این مشکل حل نشد و مجبور شدم با زحمت تمام همه فرمولها را یکبار دیگر تایپ و در فضای شخصی خودم در google آپلود کنم و پستهای قبل را هم تصحیح کنم تا مشکلی برای خوانندگان عزیز پیش نیاید. خواهشی که از دوستان دارم اینست که از سایت معرفی شده فقط برای تبدیل TEX به GIF استفاده کنند و تصویر به دست آمده را در فضای شخصی مطمئنی مانند google، yahoo یا جاهای دیگر آپلود کنند. همچنین از دوستان خوبم خواهشمندم که پستهای قبلی خودشان را به ترتیبی که عرض کردم تصحیح فرمایند. متشکرم.

ارسال متن: 31 خرداد 1385
سلام
من فكر مي كنم عكس هاي موجود در نوشته ها ----- شده است.من فقط عكسهايي را مي بينم كه در فضاي
شخصي اقاي مفيدي هستند .وقتي به ادرس عكسهاي ديگر مي روم صفحه مربوط به -------- و پيغام دسترسي به اين سايت مجاز نمي باشد مي ايد .عكس هاي دو پست اخر اقاي مفيدي را هم نمي بينم.

mofidy1
08-07-2006, 19:35
سلام
من فكر مي كنم عكس هاي موجود در نوشته ها ----- شده است.من فقط عكسهايي را مي بينم كه در فضاي
شخصي اقاي مفيدي هستند .وقتي به ادرس عكسهاي ديگر مي روم صفحه مربوط به -------- و پيغام دسترسي به اين سايت مجاز نمي باشد مي ايد .عكس هاي دو پست اخر اقاي مفيدي را هم نمي بينم.

آقای حسین پوران سلام علیکم

بنده مشکلی با سایت معرفی شده در پست 28 ندارم و برای من فیلتر نشده است. احتمال می دهم مشکل از ISP شما باشد. اگر امکان دارد از چند کارت اینترنتی مختلف (با ISP های مختلف) استفاده کنید، شاید مشکل حل شد. در ضمن بسیار بعید است که عکسهای آپلود شده در پستهای آخر بنده قابل دیدن نباشد مگر اینکه سرعت پایین اینترنت علت آن باشد. مدتی است که به خاطر تشدید فیلترینگ، سرعت اینترنت، بد که بود، بدترهم شده است. لطفاً یکی دو بار صفحه را refresh کنید (صفحه را دوباره باز کنید) و یا مدتی صبر کنید و بعد وارد صفحه شوید؛ اگر باز هم مشکل حل نشد، بفرمایید تا کارهای دیگری را که مد نظرم هست، انجام دهیم. در ضمن امید وارم نرم افزاری که در پست 141 معرفی شده است، دانلود کرده باشید.

موفق باشید.

ali_hp
08-07-2006, 20:38
آقای حسین پوران سلام علیکم

بنده مشکلی با سایت معرفی شده در پست 28 ندارم و برای من فیلتر نشده است. احتمال می دهم مشکل از ISP شما باشد. اگر امکان دارد از چند کارت اینترنتی مختلف (با ISP های مختلف) استفاده کنید، شاید مشکل حل شد. در ضمن بسیار بعید است که عکسهای آپلود شده در پستهای آخر بنده قابل دیدن نباشد مگر اینکه سرعت پایین اینترنت علت آن باشد. مدتی است که به خاطر تشدید فیلترینگ، سرعت اینترنت، بد که بود، بدترهم شده است. لطفاً یکی دو بار صفحه را refresh کنید (صفحه را دوباره باز کنید) و یا مدتی صبر کنید و بعد وارد صفحه شوید؛ اگر باز هم مشکل حل نشد، بفرمایید تا کارهای دیگری را که مد نظرم هست، انجام دهیم. در ضمن امید وارم نرم افزاری که در پست 141 معرفی شده است، دانلود کرده باشید.

موفق باشید.
سلام
من چند كارت ديگر را امتحان خواهم كرد تا شايد مشكل بر طرف شود. [ برای مشاهده لینک ، لطفا با نام کاربری خود وارد شوید یا ثبت نام کنید ] اين ادرس اخرين عكس شما است . كه براي من ----- شده است اينجا هم [ برای مشاهده لینک ، لطفا با نام کاربری خود وارد شوید یا ثبت نام کنید ] ----- شده است من چندين بار صفحه refreshرا كردم ولي نتيجه اي
نداشت.من نرم افزاري را كه معرفي كرده بوديد دانلود كرده ام اما نميدانم چطور بايد فرمولهاي ساخته شده را در داخل نوشته هايم قرار دهم (توضيحات شما براي يك فرد بسيار مبتدي مثل من كافي نيست )
لطفا شما هم به ادرس چند تا از عكس هايي كه نمي توانيد ببينيد برويدشا يد بعضي از عكسها براي شما هم ----- شده باشد .

mofidy1
09-07-2006, 07:55
سلام
من چند كارت ديگر را امتحان خواهم كرد تا شايد مشكل بر طرف شود. [ برای مشاهده لینک ، لطفا با نام کاربری خود وارد شوید یا ثبت نام کنید ] اين ادرس اخرين عكس شما است . كه براي من ----- شده است اينجا هم [ برای مشاهده لینک ، لطفا با نام کاربری خود وارد شوید یا ثبت نام کنید ] ----- شده است من چندين بار صفحه refreshرا كردم ولي نتيجه اي
نداشت.من نرم افزاري را كه معرفي كرده بوديد دانلود كرده ام اما نميدانم چطور بايد فرمولهاي ساخته شده را در داخل نوشته هايم قرار دهم (توضيحات شما براي يك فرد بسيار مبتدي مثل من كافي نيست )
لطفا شما هم به ادرس چند تا از عكس هايي كه نمي توانيد ببينيد برويدشا يد بعضي از عكسها براي شما هم ----- شده باشد .

باسلام

برای بنده هیچ کدام از اینها فیلتر نشده. علی آقا به لینک زیر مراجعه کنید ببینید فیلتر شده یا نه و به من سریع اطلاع دهید. این لینک کاملا شبیه قبلی است اما آدرس آن متفاوت است.

[ برای مشاهده لینک ، لطفا با نام کاربری خود وارد شوید یا ثبت نام کنید ]

به امید خدا درباره نرم افزاری که معرفی کردم مفصلا صحبت خواهیم کرد. علی آقا به پست مساله هفته هفتم زبان tex را هم اضافه کردم که اگر تصویر را نمی بینید، حداقل مساله را متوجه شوید.

ali_hp
09-07-2006, 20:08
باسلام

برای بنده هیچ کدام از اینها فیلتر نشده. علی آقا به لینک زیر مراجعه کنید ببینید فیلتر شده یا نه و به من سریع اطلاع دهید. این لینک کاملا شبیه قبلی است اما آدرس آن متفاوت است.

[ برای مشاهده لینک ، لطفا با نام کاربری خود وارد شوید یا ثبت نام کنید ]

به امید خدا درباره نرم افزاری که معرفی کردم مفصلا صحبت خواهیم کرد. علی آقا به پست مساله هفته هفتم زبان tex را هم اضافه کردم که اگر تصویر را نمی بینید، حداقل مساله را متوجه شوید.
سلام
متاسفانه اينجا هم براي من ----- شده است.من با نرم افزاري كه شما معرفي كرديد مي توانم كار كنم ولي نمي دانم فرمولها را در كجا و چطور اپلود كنم.
متشكرم.

mofidy1
09-07-2006, 22:01
با سلام

علی آقا به لینک زیر بروید:

[ برای مشاهده لینک ، لطفا با نام کاربری خود وارد شوید یا ثبت نام کنید ]

با توجه به شکل زیر روی آیکنی که با فلش مشخص شده است، کلیک کنید تا عبارت

<math>Insert formula here</math>

ظاهر شود. حال به جای عبارت Insert formula here فرمول tex خود را بنویسید. سپس در پایین صفحه سفید، دگمه
Show Preview را فشار دهید تا فرمولتان ایجاد شود. با کلیک راست، آدرس آن را بردارید و استفاده کنید. توصیه می کنم که این تصویر را save کنید که اگر به هر دلیلی آدرس مذکور از کار افتاد بتوانید با آپلود کردن تصو یرهای save شده، مشکل را حل کنید.

[ برای مشاهده لینک ، لطفا با نام کاربری خود وارد شوید یا ثبت نام کنید ]


علی آقا به احتمال قوی انجام مراحل بالا مشکلتان را به طور موقت حل می کند و فعلاً نیازی به آموزش مفصل آپلود تصاویر نیست. اگر باز مشکل حل نشد خبرم کنید. دنبال تعویض کارت اینترنتی خود هم باشید. بنده واقعا متعجبم که چگونه ISP شما سایتهای علمی به این مهمی را فیلتر کرده است.

لطف کنید به گفتگوها سریع جواب دهید. منتظرم .

موفق باشید.

eh_mn
13-07-2006, 21:23
به نام خدا
سلام به همه دوستان.
بخاطر اينكه مدت طولاني به اينجا نيامدم مرا ببخشيد. گرچه علت غيبتم تقريبا موجه بود.
آقاي مفيدي سلام و خسته نباشيد. بعد از اينكه فهميدم ممكن است مدتي نتوانم به p30world بيايم تصميم گرفتم حداقل كار ممكن در جهت قراري كه با هم گذاشتيم را انجام دهم. البته بهتر بود اين كار با هماهنگي با شما انجام مي شد.
چند مطلب در رابطه با موضوع مورد بحث تاپيك "كاربردهاي رياضي" پيدا شد. بعنوان مثال كاربردهايي از نظريه رمز نگاري و رمزگشايي در انتقال و فشرده سازي اطلاعات و همينطور كاربردهايي از نظريه بازي ها در سياست و اقتصاد و تصميم گيري و ... . ولي همان طور كه مي دانيد بيان اين مطالب نياز به مقدماتي دارد كه ممكن است خارج از حوصله خوانندگان باشد. لطفا مرا راهنمايي كنيد.

موفق باشيد

ali_hp
14-07-2006, 20:19
سلام
لم: براي هر سه عدد حقيقي و مثبت x,y,z داريم:
[ برای مشاهده لینک ، لطفا با نام کاربری خود وارد شوید یا ثبت نام کنید ]
اثبات:
[ برای مشاهده لینک ، لطفا با نام کاربری خود وارد شوید یا ثبت نام کنید ]
[ برای مشاهده لینک ، لطفا با نام کاربری خود وارد شوید یا ثبت نام کنید ]
[ برای مشاهده لینک ، لطفا با نام کاربری خود وارد شوید یا ثبت نام کنید ]
[ برای مشاهده لینک ، لطفا با نام کاربری خود وارد شوید یا ثبت نام کنید ]
[ برای مشاهده لینک ، لطفا با نام کاربری خود وارد شوید یا ثبت نام کنید ]
حال اگر در لم به جايx,y,z يكبار a,b,c و يكبار b,c,d و يكبار a,c,d و يكبار a,b,d را قرار دهيم وچهار نامساوي حاصل را با هم جمع كنيم حكم مساله نتيجه مي شود.

mofidy1
14-07-2006, 20:47
با سلام

علی آقا تصویر فرمول شما در پست قبل بسیار طولانی است. ظاهرا همه فرمول را در یک سطر تایپ کرده اید و به احتمال قوی همه مرورگرها قادر به جای دادن آن در یک صفحه نیستند . اگر امکان دارد فرمولتان را edit کنید.

علی آقا مطلب قبلی بنده را بی جواب گذاشتید! آیا مشکل شما در دیدن تصاویر حل شد یا نه؟ دوست عزیز سعی کنید که به پستها سریع جواب دهید حتی اگر در جمله ای بسیار کوتاه باشد. متشکرم.


موفق باشید.

mofidy1
15-07-2006, 08:30
با سلام

برای تبدیل فرمولها به زبان TEX به وسیله نرم افزاری که در پست 141 معرفی شد ( نرم افزار MathType ) کارهای زیر را انجام دهید:

1) به قسمت Preferences (در قسمت منوهای اصلی و در بالای صفحه) بروید و بخش Trancslators را کلیک کنید.

2) در این قسمت Translation to other language را علامت بزنید و از داخل آن TEX-AMS-LATEX را کلیک کنید. علامت یا تیک بقیه دستورات این پنجره را -اگر تیک خورده اند - بردارید و OK را کلیک کنید.

3) فرمول دلخواه خود را با نرم افزار مذکور طراحی کنید و تمام آنها را انتخاب و سپس COPY کنید. این کار باعث می شود که فرمول شما به زبان LATEX -که پیشرفته از TEX است - تبدیل شود.

4) حال در مکان مناسبی که در پستهای28 یا 153 یا 155 معرفی شده است، دستور LATEX به دست آمده را PASTE کنید . (گاهی لازم است که علامتهای [/ و \] را از اول و آخر دستورات حذف کنید.)

موفق باشید.

mofidy1
15-07-2006, 11:37
با سلام

فرض کنید a, b , c , d همگی مثبت باشند. نامساوی زیر را ثابت کنید:


[ برای مشاهده لینک ، لطفا با نام کاربری خود وارد شوید یا ثبت نام کنید ]


=================================
مساله بالا به زبان tex برای دوستانی که تصویر بالا را مشاهده نمی کنند.
این کار برای دوستانی که تصویر بالا را می بینند اما با زبان tex آشنایی کافی ندارند،
فرصت خوبی است تا دستورات این زبان را فراگیرند.


\frac{{a^3 + b^3 + c^3 }}
{{a + b + c}} + \frac{{b^3 + c^3 + d^3 }}
{{b + c + d}} + \frac{{c^3 + d^3 + a^3 }}
{{c + d + a}} + \frac{{d^3 + a^3 + b^3 }}
{{d + a + b}} \geq a^2 + b^2 + c^2 + d^2


==============================
ارسال متن: شنبه17 تیر 1385

با سلام

از دوست عزیزم آقای حسین پوران به خاطر حل زیبای مساله هفتم تشکر می کنم. راه حل ایشان را در پست 157 ملاحظه فرمایید. این روش جالب و نسبتا کوتاه، تقریبا از شیوه حل مستقیم پیروی کرده است. این شیوه با اینکه در بسیاری از اوقات برای حل مسائل مقدماتی کافیست اما گاهی به دلیل پیچیدگی های آن، چندان مناسب به نظر نمی رسد. بنده مساله هفتم را به روشی که معمولا به کار می رود حل می کنم؛ با تذکر این نکته که این راه حل طولانی تر از روش آقای حسین پوران است. (این روش را از کتاب «حل مساله از طریق مساله» ترجمه «علی ساوجی» اقتباس کرده ام.)

حل مساله:

همانطور که آقای حسین پوران توضیح داده ا ند کافیست ثابت کنیم:


[ برای مشاهده لینک ، لطفا با نام کاربری خود وارد شوید یا ثبت نام کنید ]

می توان نوشت:


[ برای مشاهده لینک ، لطفا با نام کاربری خود وارد شوید یا ثبت نام کنید ]

پس ثابت می کنیم:


[ برای مشاهده لینک ، لطفا با نام کاربری خود وارد شوید یا ثبت نام کنید ]

بنابر نامساوی کوشی شوارتز (رجوع کنید به اینجا ([ برای مشاهده لینک ، لطفا با نام کاربری خود وارد شوید یا ثبت نام کنید ])) می توان نوشت:


[ برای مشاهده لینک ، لطفا با نام کاربری خود وارد شوید یا ثبت نام کنید ]

بنابراین


[ برای مشاهده لینک ، لطفا با نام کاربری خود وارد شوید یا ثبت نام کنید ]

حال با رساندن طرفین به توان دو و سپس ساده کردن طرفین مساله حل می شود.

مشابه روش بالا می توان در حالت کلی ثابت کرد:


[ برای مشاهده لینک ، لطفا با نام کاربری خود وارد شوید یا ثبت نام کنید ]

موفق باشید.

ارسال متن: شنبه 24 تیر 1385

mofidy1
15-07-2006, 12:43
با سلام

حداقل به دو روش متفاوت و کاملاً هندسی تساوی زیر را ثابت کنید:


[ برای مشاهده لینک ، لطفا با نام کاربری خود وارد شوید یا ثبت نام کنید ]

ارسال متن: شنبه 24 تیر 1385

موفق باشید.

ali_hp
15-07-2006, 16:04
با سلام

علی آقا تصویر فرمول شما در پست قبل بسیار طولانی است. ظاهرا همه فرمول را در یک سطر تایپ کرده اید و به احتمال قوی همه مرورگرها قادر به جای دادن آن در یک صفحه نیستند . اگر امکان دارد فرمولتان را edit کنید.

علی آقا مطلب قبلی بنده را بی جواب گذاشتید! آیا مشکل شما در دیدن تصاویر حل شد یا نه؟ دوست عزیز سعی کنید که به پستها سریع جواب دهید حتی اگر در جمله ای بسیار کوتاه باشد. متشکرم.


موفق باشید.
سلام
به خاطر دير كرد در جواب پستها معذرت مي خواهم.احتمالا از روز دوشنبه تا يك مدت طولاني(مثلا دوهفته) نتوانم به اتاق سر بزنم.در باره ديدن تصاوير هم بايد بگم مشكل من رفع نشد اما وقتي با ----- شكن به اتاق رياضيات مي ايم بعضي از تصاوير را مي توانم ببينم.(تصاوير دوپست شما كه راه حل مساله هفته قبل و مساله اين افته است را نمي توانم ببينم) سعي مي كنم همين امروز پستم را edit كنم.

ali_hp
15-07-2006, 17:41
سلام
به خاطر دير كرد در جواب پستها معذرت مي خواهم.احتمالا از روز دوشنبه تا يك مدت طولاني(مثلا دوهفته) نتوانم به اتاق سر بزنم.در باره ديدن تصاوير هم بايد بگم مشكل من رفع نشد اما وقتي با ----- شكن به اتاق رياضيات مي ايم بعضي از تصاوير را مي توانم ببينم.(تصاوير دوپست شما كه راه حل مساله هفته قبل و مساله اين افته است را نمي توانم ببينم) سعي مي كنم همين امروز پستم را edit كنم.
سلام
من توانستم راه حل مساله هفته قبل و صورت مساله اين هفته را به طور كامل ببينم. به نظر من راه حل مساله هفته قبل واقعا يك استفاده استادانه از نامساوي كوشي بود.

eh_mn
18-07-2006, 08:55
با سلام

نشان دهيد تابعي از اعداد حقيقي مانند f با اين خاصيت كه fof=x^2-2 وجود ندارد !!!!

بنظر شما اين عجيب نيست؟؟

موفق باشيد

hadi_mdk
19-07-2006, 20:53
سلام
رياضي بسيار سخته :puke: :angry:
شبه هم براش كابوس مي بينم :sad:
يكي بياد يك چيز خوب در موردش بگه كه من با رياضي بتونم كنار بيام ;)
چون عجيب گيرشم :sad:
با تشكر

mofidy1
19-07-2006, 22:26
با سلام

هادی جان شنا بلدی یا نه؟ می دونی چه جوری باید شنا یاد گرفت؟ می شه خیلی ساده و خودمونی برام بگی که اگر کسی باشه که حتما باید شنا یاد بگیره اما از غرق شدن، حسابی می ترسه باید چی کار کنه؟ آقا هادی منظورم رو فهمیدی؟ منتظرتم

موفق باشی

mofidy1
21-07-2006, 19:43
با سلام

حداقل به دو روش متفاوت و کاملاً هندسی تساوی زیر را ثابت کنید:


[ برای مشاهده لینک ، لطفا با نام کاربری خود وارد شوید یا ثبت نام کنید ]

ارسال متن: شنبه 24 تیر 1385

موفق باشید.

روش اول:

در مستطیل زیر فرض کنید 1=FG=GH=HE=AB=BC=CD. داریم:


[ برای مشاهده لینک ، لطفا با نام کاربری خود وارد شوید یا ثبت نام کنید ]


[ برای مشاهده لینک ، لطفا با نام کاربری خود وارد شوید یا ثبت نام کنید ]

روش دوم:

مثلث ABC را چنان رسم کنید که:


[ برای مشاهده لینک ، لطفا با نام کاربری خود وارد شوید یا ثبت نام کنید ]

با توجه به قانون کسینوسها در مثلث می توان نوشت:


[ برای مشاهده لینک ، لطفا با نام کاربری خود وارد شوید یا ثبت نام کنید ]

باز هم قانون کسینوسها در مثلث و رابطه ای که بین کسینوس و تانژانت موجود است نتیجه می دهد که:


[ برای مشاهده لینک ، لطفا با نام کاربری خود وارد شوید یا ثبت نام کنید ]

که مطلب را نتیجه می دهد.

توضیحات: روش اول را از یکی از المپیادهای ریاضی دانش آموزی اقتباس کرده ام(رجوع کنید به کتاب المپیادهای ریاضی در ایران جلد اول تالیف دکتر عباد الله محمودیان). روش دوم نیز به وسیله یک دانشجوی چینی برای اینجانب ارسال شده است. به لینک زیر مراجعه فرمایید:

[ برای مشاهده لینک ، لطفا با نام کاربری خود وارد شوید یا ثبت نام کنید ]

در ضمن شکل هندسی بالا نیز با نرم افزار WinGCLC رسم شده است.

ارسال متن: جمعه 30 تیر 1385

mofidy1
22-07-2006, 17:05
با سلام

مساله هفته نهم: ثابت کنید حاصلجمع هیچ k عدد طبیعی متوالی(k>1) را نمی توان به صورت توانی از عدد 2 نوشت.

موفق باشید.

ارسال متن: شنبه 31 تیر 1385

hadi_mdk
22-07-2006, 18:31
اگه بلد بودم جواب مي دم .

baphomet
28-07-2006, 21:47
با سلام

مساله هفته نهم: ثابت کنید حاصلجمع هیچ k عدد طبیعی را نمی توان به صورت توانی از عدد 2 نوشت.

موفق باشید.

ارسال متن: شنبه 31 تیر 1385
فک نمیکنم صورت مسئله درست باشه ! به عنوان مثال : 1+3=4=2^2!!!
البته فک میکنم منظور یه مسئله دیگه بوده که صورتش حالا بنا به دلایلی اشتباه مطرح شده سعی میکنم مسئله مورد نظر رو براتون با روش حل پیدا کنم.....

mofidy1
29-07-2006, 17:17
با سلام

دوست گرامی حق با شماست و متن مساله تصحیح شد.

متشکرم.

mofidy1
29-07-2006, 17:34
با سلام

مساله هفته نهم: ثابت کنید حاصلجمع هیچ k عدد طبیعی متوالی(k>1) را نمی توان به صورت توانی از عدد 2 نوشت.

موفق باشید.

ارسال متن: شنبه 31 تیر 1385

فرض کنید حاصلجمع k عدد طبیعی متوالی که از n شروع شده باشند به صورت توانی از عدد 2 نوشته شده باشد. می توان نوشت:


[ برای مشاهده لینک ، لطفا با نام کاربری خود وارد شوید یا ثبت نام کنید ]

بنابر این k باید زوج باشد که در این صورت عبارت داخل پرانتز آخر، فرد خواهد شد که تناقض با طرف راست دارد زیرا طرف راست، توانی از 2 است.

ارسال متن: شنبه 7 مرداد 1385

mofidy1
29-07-2006, 22:01
با سلام

فرض کنید f و g دو تابع با شرایط زیر باشند:


[ برای مشاهده لینک ، لطفا با نام کاربری خود وارد شوید یا ثبت نام کنید ]

ثابت کنید f در هر نقطه مشتق پذیر است و مشتق آنرا بیابید.

موفق باشید.

ارسال متن: شنبه 7 مرداد 1385

s2r
30-07-2006, 01:48
ترخدا كمك كنيد من تا صبح بايد جواب اين سوال رو بدونم. سور يه چي هست تو رياضي كه 3 نوع هم داره.
سور وجودي - سور عمومي - سور صفر
اگه بشه سور رو به طور كلي توضيح بديد و بگيد هر كدوم از اينا چيه ممنون ميشم.

mofidy1
01-08-2006, 13:51
با سلام

دوستان سوالات ریاضی خود را در آدرس زیر مطرح فرمایید. متشکرم.

[ برای مشاهده لینک ، لطفا با نام کاربری خود وارد شوید یا ثبت نام کنید ]

abay
01-08-2006, 22:01
سلام
اگر برايتان امكان دارد پاسخنامه تشريحي بيست وسومين وبيست وچهار مين المپياد رياضي ايران را در سايت قرار دهيد در ضمن به اين سوالات نيز جواب دهيد

[ برای مشاهده لینک ، لطفا با نام کاربری خود وارد شوید یا ثبت نام کنید ]

mohammadi mohammad
02-08-2006, 11:19
C:\Documents and Settings\public\My Documents\My Pictures\12.gif با سلام

در مورد حد

طبق بسط مك لورن C:\Documents and Settings\public\My Documents\My Pictures\1.gif

mofidy1
02-08-2006, 14:13
C:\Documents and Settings\public\My Documents\My Pictures\12.gif با سلام

در مورد حد

طبق بسط مك لورن C:\Documents and Settings\public\My Documents\My Pictures\1.gif

با سلام

آقای محمدی لطفا لینک را تصحیح کنید.

موفق باشید.

mofidy1
02-08-2006, 14:43
سلام
اگر برايتان امكان دارد پاسخنامه تشريحي بيست وسومين وبيست وچهار مين المپياد رياضي ايران را در سايت قرار دهيد در ضمن به اين سوالات نيز جواب دهيد
[ برای مشاهده لینک ، لطفا با نام کاربری خود وارد شوید یا ثبت نام کنید ]

باسلام

فایلی را که آقا یا خانم 136784 آپلود کرده و خواستار حل مسائل آن شده اند، ذیلاً مشاهده می فرمایید. منتظر حل آنها توسط دوستان عزیز هستیم.

(برای دیدن راه حلها به لینک زیر مراجعه کنید:
[ برای مشاهده لینک ، لطفا با نام کاربری خود وارد شوید یا ثبت نام کنید ])

موفق باشید.
================================================== =====


[ برای مشاهده لینک ، لطفا با نام کاربری خود وارد شوید یا ثبت نام کنید ]

'POP'
02-08-2006, 18:41
[ برای مشاهده لینک ، لطفا با نام کاربری خود وارد شوید یا ثبت نام کنید ]

;)

mofidy1
04-08-2006, 17:24
با سلام

فرض کنید f و g دو تابع با شرایط زیر باشند:


[ برای مشاهده لینک ، لطفا با نام کاربری خود وارد شوید یا ثبت نام کنید ]

ثابت کنید f در هر نقطه مشتق پذیر است و مشتق آنرا بیابید.

موفق باشید.

ارسال متن: شنبه 7 مرداد 1385

[ برای مشاهده لینک ، لطفا با نام کاربری خود وارد شوید یا ثبت نام کنید ]

amintnt
05-08-2006, 03:36
متاسفانه اينجا ديگه نميتونم كاري كنم.آخه دوم دبيرستان كه اين چيزا رو نداره.(تازه دارم ميرم دوم)

mofidy1
05-08-2006, 11:56
متاسفانه اينجا ديگه نميتونم كاري كنم.آخه دوم دبيرستان كه اين چيزا رو نداره.(تازه دارم ميرم دوم)

با سلام

دوست عزیز، این اتاق فقط مخصوص اول دبیرستانی ها نیست. اگر در اتاق ریاضیات بگردید مسائلی که در سطح اول و دوم دبیرستان هستند، پیدا خواهید کرد. از طرف دیگر چرا این قدر عصبانی هستید؟! دستتان را بالا بزنید و خودتان مسائل ناب ریاضی در سطح اول دبیرستان را در اینجا مطرح کنید تا به کمک خود شما حلش کنیم. یا علی مدد.

منتظر تماس بعدیتان هستیم.

موفق باشید.

mofidy1
05-08-2006, 12:09
باسلام

دوستان عزیز تایپیک جدیدی با عنوان «کاربردهای ریاضی» با مدیریت آقای احسان منبتی-دانشجوی ریاضیات کاربردی دانشگاه فردوسی مشهد- با آدرس زیر افتتاح شده است و قرار است در آن هر هفته شنبه ها يكي از كاربردهاي رياضي بطور مختصر بيان شود. از ایشان به خاطر این کار مهم تشکر می کنیم.

[ برای مشاهده لینک ، لطفا با نام کاربری خود وارد شوید یا ثبت نام کنید ]

mofidy1
05-08-2006, 13:24
با سلام

فرض کنید:

[ برای مشاهده لینک ، لطفا با نام کاربری خود وارد شوید یا ثبت نام کنید ]
ثابت کنید:

[ برای مشاهده لینک ، لطفا با نام کاربری خود وارد شوید یا ثبت نام کنید ]

موفق باشید.

ارسال متن: شنبه 14 مرداد 1385

abay
06-08-2006, 02:03
سلام
من در تاپيك قبلي در خواست كرده بودم كه پاسخهاي تشريحي بيست وسومين وبيست وچهارمين المپياد رياضي را قرار دهيد اگر برايتان مقدور نيست لطفا"بفرماييدتا از جاي ديگري تهيه كنم در ضمن در مورد سوالي كه در پست قرار داده ايد من فكر ميكنم درحكم سوال اشتباهي روي داده ودر طرف دوم نامساوي احتمالا"به جاي عدد يك چهارم عدديك هشتم است ومن با اين احتمال اين سوال راحل كردم با اين توضيح كه چون مجموع سه زاويه 180است ودرمثلث چنين حالتي را داريم وبا توجه به قضيه كسينوسها خواهيم داشت:
[ برای مشاهده لینک ، لطفا با نام کاربری خود وارد شوید یا ثبت نام کنید ]

mofidy1
06-08-2006, 16:23
با سلام

آقا یا خانم 136784 متن سوال کاملاً درست است. خودتان نیز این مطلب را در راه حلتان ثابت کرده اید(مشخص است عددی که کمتر یا مساوی یک هشتم است از یک چهارم کمتر است). چیزی که شما حل کرده اید در واقع «قویتر کردن صورت مساله» است. از ارائه راه حلتان هم بسیار خوشحال شدم. حال که این مساله را خودتان کاملتر کردید اگر مایلید به این سوال هم جواب دهید: شرط لازم و کافی برای اینکه عبارت ذکر شده مساوی یک هشتم شود چیست؟

ضمنا در دو لینک زیر بعضی از مسائل بيست وچهارمين المپياد رياضی کشور حل شده است که اولی با فرمت pdf است. (اگر لینک زیر در دسترس نبود فایل ضمیمه زیر را دانلود کنید.) توجه بفرمایید که بنده ادعایی درباره درستی یا نادرستی حل این مسائل ندارم.

[ برای مشاهده لینک ، لطفا با نام کاربری خود وارد شوید یا ثبت نام کنید ]

[ برای مشاهده لینک ، لطفا با نام کاربری خود وارد شوید یا ثبت نام کنید ]

اگر خودتان را هم به طور کامل معرفی کنید تا بیشتر با هم آشنا شویم ممنون خواهم شد-نام و نام خانوادگی، میزان تحصیلات، شغل و مکان سکونت

موفق باشید.

abay
06-08-2006, 20:50
سلام
من بخاطر قرار دادن جواب سوالات از شما تشكر ميكنم اما جواب سوالات واضح نيفتاده ولي باز هم از هيچي بهتر است ودر راجع به سواليكه كرديد بايد بگويم من 136784هستم مدركم ديپلم شغلم فوتباليست(بازيكن جوانان ماشين سازي تبريز)ومكان سكونتم شهر زيباي تبريز است و17سالمه درضمن من از شما سوالاتي داشتم اكه مايل بوديد جواب دهيديكي اينكه نرم افزاري كه براي رسم شكل هندسي در مسئله هفته دوم استفاده كرده ايد را معرفي كنيد وديگر اينكه اگر سوالات المپياد هاي مزبور را داريد هر هفته چند تايي راقرار دهيد تا شماودوستاني كه توانايي حل آنها را دارند به من كمك كنند البته من دفترچه سوالات را دارم ولي شكلهاي مربوط به سوالات خيلي سياهه وهيچي ديده نميشه اگه نتوانستيد من سوالات را با همان كيفيت قرار ميدم

mofidy1
06-08-2006, 22:05
با سلام

شکل هندسی هفته دوم بوسیله خود ویندوز (نرم افزار Paint ) رسم شده است.

ضمنا برای دسترسی به سوالات المپیادهای ریاضی دانش آموزی به لینک زیر مراجعه کنید:

[ برای مشاهده لینک ، لطفا با نام کاربری خود وارد شوید یا ثبت نام کنید ]

موفق باشید.

abay
07-08-2006, 15:49
سلام
در پست 184شرط لازم وكافي براي برابر شدن عبارت با يك هشتم به نظر من اينست
[ برای مشاهده لینک ، لطفا با نام کاربری خود وارد شوید یا ثبت نام کنید ]
[ برای مشاهده لینک ، لطفا با نام کاربری خود وارد شوید یا ثبت نام کنید ]

در ضمن بنده سوالي داشتم كه در اينجا مطرح مي كنم. در معادله ي زير مقدار nرا بدست آوريد
[ برای مشاهده لینک ، لطفا با نام کاربری خود وارد شوید یا ثبت نام کنید ]

mofidy1
07-08-2006, 18:23
با سلام

احسنت، برای سوال بنده جواب درستی پیدا کردید.

اما درباره سوالی که مطرح کردید. فکر می کنم منظور شما از «معادله روبرو را به دست آورید» این است که معادله روبرو را برای n حل کنید. ظاهرا این مساله دارای جواب نیست زیرا عدد زیر رادیکال اولی حدودا 76 و دومی از 1 کمتر است. مجموع ریشه های دوم این دو عدد حداکثر 9 است و اگر n را بزرگتر کنیم این مجموع کوچکتر و کوچکتر می شود.

دوست عزیز سعی کنید که اندازه تصویرهایی را که آپلود می کنید، کوچکتر انتخاب کنید تا مطالعه آن ساده تر باشد و جای کمتری را در صفحه اشغال کند.

متشکرم.

abay
07-08-2006, 19:35
سلام
اين سوالي كه در بالا نوشته شده مربوط به المپياد كشور چين است البته من سوال را حل كرده ام ودر واقع مشكل اصلي بنده در بدست آوردن جواب آخر است كه من آنرا به صورت شهودي بدست مي آورم ولي راه حل آن را نمي دانمو هدف اصلي من از طرح كردن اين سوال در واقع ارايه حل براي قسمت آخر آن از طريق شما بود چون من هر چه وقت گذاشتم نتوانستم آن را حل كنم البته نحوه حل سوال هركسي ممكن است متفاوت باشد اما من فكر ميكنم قسمت آخرحل سوال باهر روشي يكسان باشد درضمن شما جوابي كه در بالا داده ايد مي تواند براي آن سوال درست باشد ولي من صورت سوال را غلط نوشته ام و از شما به خاطر آن عذر خواهي ميكنم ومن روي سوال را اصلاح كردم ودر طرف دوم تساوي بايد عدد راديكال 20 مي بودنه 20.

abay
09-08-2006, 11:52
سلام
اگر x,y,z اعداد حقيقي ومثبت باشند چنانكه x+y+z=1 آنگاه ثابت كنيد كه:

[ برای مشاهده لینک ، لطفا با نام کاربری خود وارد شوید یا ثبت نام کنید ]

ali_hp
10-08-2006, 01:28
سلام
در ضمن بنده سوالي داشتم كه در اينجا مطرح مي كنم. در معادله ي زير مقدار nرا بدست آوريد
[ برای مشاهده لینک ، لطفا با نام کاربری خود وارد شوید یا ثبت نام کنید ]
سلام به همه دوستان
داريم:
[ برای مشاهده لینک ، لطفا با نام کاربری خود وارد شوید یا ثبت نام کنید ]
پس براي هر n خواهيم داشت:
[ برای مشاهده لینک ، لطفا با نام کاربری خود وارد شوید یا ثبت نام کنید ]
همچنين داريم:
[ برای مشاهده لینک ، لطفا با نام کاربری خود وارد شوید یا ثبت نام کنید ]
براي اثبات دقت كنيدكه:
[ برای مشاهده لینک ، لطفا با نام کاربری خود وارد شوید یا ثبت نام کنید ]
[ برای مشاهده لینک ، لطفا با نام کاربری خود وارد شوید یا ثبت نام کنید ]
براي n هاي بزرگتر يا مساوي با چهار داريم:
[ برای مشاهده لینک ، لطفا با نام کاربری خود وارد شوید یا ثبت نام کنید ]
پس براي n>3 خواهيم داشت:
[ برای مشاهده لینک ، لطفا با نام کاربری خود وارد شوید یا ثبت نام کنید ]
بنابراين براي n>3 تساوي ممكن نيست.براي n<3 هم بع سادگي مي توان ديد كه سمت چپ تساوي بزرگتر از
سمت راست ان است.
حالا ثابت ميكنم براي n=3 تساوي برقرار است:
فرض كنيد :
[ برای مشاهده لینک ، لطفا با نام کاربری خود وارد شوید یا ثبت نام کنید ]
بايد ثابت كنيم :
[ برای مشاهده لینک ، لطفا با نام کاربری خود وارد شوید یا ثبت نام کنید ]
با به توان 3 رساندن صرفين تساوي داريم:
[ برای مشاهده لینک ، لطفا با نام کاربری خود وارد شوید یا ثبت نام کنید ]
[ برای مشاهده لینک ، لطفا با نام کاربری خود وارد شوید یا ثبت نام کنید ]
پس:
[ برای مشاهده لینک ، لطفا با نام کاربری خود وارد شوید یا ثبت نام کنید ]

ali1234
10-08-2006, 09:38
با سلام
يه سوال داشتم
به ازاي هر عدد a,b,c نشان دهيد:
a^2+b^2+c^2>=ab+bc+ac
با تشكر فراوان.

ali_hp
10-08-2006, 15:55
با سلام
يه سوال داشتم
به ازاي هر عدد a,b,c نشان دهيد:
a^2+b^2+c^2>=ab+bc+ac
با تشكر فراوان.
سلام
[ برای مشاهده لینک ، لطفا با نام کاربری خود وارد شوید یا ثبت نام کنید ]

mofidy1
10-08-2006, 16:04
با سلام

دو مساله جالب:

1) دوستان به نظر شما ((sin(cos(x بزرگتراست یا ((cos(sin(x ؟

2) ریشه سوم 60 بزرگتر است یا 2 به اضافه ریشه سوم 7 ؟ (بدون استفاده از ماشین حساب!)


موفق باشید.

Encor3
11-08-2006, 02:12
این فرمول اینارو چطوری مینویسید. اگه برنامه ای چیزی داره بزارید. منم می خوام جواب بدم.[ برای مشاهده لینک ، لطفا با نام کاربری خود وارد شوید یا ثبت نام کنید ]

ali1234
11-08-2006, 08:44
سلام
[ برای مشاهده لینک ، لطفا با نام کاربری خود وارد شوید یا ثبت نام کنید ]

دستت درد نكنه .
ميشه بگيد اين عبارت هاي رياضي با چه نرم افزاري نوشتيد؟

'POP'
11-08-2006, 08:49
این فرمول اینارو چطوری مینویسید. اگه برنامه ای چیزی داره بزارید. منم می خوام جواب بدم.[ برای مشاهده لینک ، لطفا با نام کاربری خود وارد شوید یا ثبت نام کنید ]
با Word براحتي ميتونيد .
[ برای مشاهده لینک ، لطفا با نام کاربری خود وارد شوید یا ثبت نام کنید ] %B1%D8%AF
[ برای مشاهده لینک ، لطفا با نام کاربری خود وارد شوید یا ثبت نام کنید ]
;)

mofidy1
11-08-2006, 14:43
با Word براحتي ميتونيد .
[ برای مشاهده لینک ، لطفا با نام کاربری خود وارد شوید یا ثبت نام کنید ] %B1%D8%AF
[ برای مشاهده لینک ، لطفا با نام کاربری خود وارد شوید یا ثبت نام کنید ]
;)

با سلام

از آقا مرتضی به خاطر تذکر مفیدشان متشکرم. متاسفانه WORD با اینکه در بسیاری از فرمولسازیها به درد می خورد اما در طراحی فرمولهای پیچیده خیلی کارا نیست، به ویژه شامل همه فونتهای ریاضی معتبر که به فونتهای ams معروف هستند، نیست. قبلاً در این اتاق مفصلا درباره ساختن فرمولهای ریاضی (حتی پیشرفته) و سوار کردن آنها در صفحات وب صحبت کرده ایم که باز به طور خلاصه آنها را عرض می کنم:

در دو لینک زیر نرم افزار mathtype و طریقه دانلود و استفاده از آن آمده است:

[ برای مشاهده لینک ، لطفا با نام کاربری خود وارد شوید یا ثبت نام کنید ]
[ برای مشاهده لینک ، لطفا با نام کاربری خود وارد شوید یا ثبت نام کنید ]

در لینک زیر برای کسانی که به زبان tex آشنایی مختصر دارند طریقه فرمولسازی را در محیط وب توضیح داده ایم.

[ برای مشاهده لینک ، لطفا با نام کاربری خود وارد شوید یا ثبت نام کنید ]

البته در پستهای دیگر نیز مطالب مهمی ذکر شده است که با کمی صرف وقت آنها را پیدا خواهید کرد.

موفق باشید.

Encor3
11-08-2006, 14:55
با Word براحتي ميتونيد .
[ برای مشاهده لینک ، لطفا با نام کاربری خود وارد شوید یا ثبت نام کنید ] %B1%D8%AF
[ برای مشاهده لینک ، لطفا با نام کاربری خود وارد شوید یا ثبت نام کنید ]
;)
دستت درد نکنه مرتضی جان. من نه اینکه تازه کلاس اول دبیرستان رو تموم کردم (تو تیز هوشان درس می خونم) برا همین در حد توانم کمک خواهم کرد. البته کمک که چه عرض کنم همه ماشالا دکتر مهندس [ برای مشاهده لینک ، لطفا با نام کاربری خود وارد شوید یا ثبت نام کنید ]من زیر پا له میشم.[ برای مشاهده لینک ، لطفا با نام کاربری خود وارد شوید یا ثبت نام کنید ]
خلاصه منتظر سولات بعدی هستم. مشکل ورد هم حل شد.(خیلی حال می ده)[ برای مشاهده لینک ، لطفا با نام کاربری خود وارد شوید یا ثبت نام کنید ]

mofidy1
11-08-2006, 15:58
با سلام

فرض کنید:

[ برای مشاهده لینک ، لطفا با نام کاربری خود وارد شوید یا ثبت نام کنید ]
ثابت کنید:

[ برای مشاهده لینک ، لطفا با نام کاربری خود وارد شوید یا ثبت نام کنید ]

موفق باشید.

ارسال متن: شنبه 14 مرداد 1385

با سلام

از آقای 136784 که در پست 184 مساله را حل کردند، تشکر می کنیم. بنده روش دیگری را ارائه می کنم که هر چند احتیاج به مقداری اطلاعات هندسی خوب دارد، اما بسیار ساده تر است. البته روش دیگری نیز برای این مساله وجود دارد که از صعودی بودن سینوس در ناحیه اول استفاده می کند.

حل مساله:

فرض کنیم R شعاع دایره محیطی و r شعاع دایره محاطی مثلثی باشد که زوایای آن آلفا، بتا و گاما هستند. بنابر یک قضیه معروف داریم:


[ برای مشاهده لینک ، لطفا با نام کاربری خود وارد شوید یا ثبت نام کنید ]

چون r از R کوچکتر است نامساوی مورد نظر ثابت می شود.

این مساله یکی از مسائل المپیاد ریاضی مجارستان سال 1897 است!!

موفق باشید.

ali_hp
11-08-2006, 18:19
دستت درد نكنه .
ميشه بگيد اين عبارت هاي رياضي با چه نرم افزاري نوشتيد؟
سلام
خواهش ميكنم.
من از لينكي كه اقاي مفيدي در پست 155معرفي كرده اند استفاده كرده ام.درباره روش نوشتن فرمولهادر اين اتاق
در پستهاي قبلي توضيحات مفيدي داده شده است مثلا پستهاي:و159و155و141و28و 199

mofidy1
12-08-2006, 00:12
با سلام

در مثلث دلخواه زیر، M وسط BC و I وسط AM است. BI را ادامه دهید تا AC را در D قطع کند. ثابت کنید مساحت مثلث ABC ، دوازده برابر مساحت مثلث AID است.

موفق باشید.


[ برای مشاهده لینک ، لطفا با نام کاربری خود وارد شوید یا ثبت نام کنید ]

ارسال متن: شنبه 21 مرداد 1385

ali1234
12-08-2006, 10:00
با سلام
بدون محاسبه نشان دهيد:
10/1 > 100/99 * 99/98 ...4/3 *2/1

ali_hp
12-08-2006, 12:18
با سلام
بدون محاسبه نشان دهيد:
10/1 > 100/99 * 99/98 ...4/3 *2/1
سلام
نظمي بين اعداد داده شده وجود ندارد كه بتوان متوجه بقيه اعداد (كه به جاي انها نقطه چين گذاشته شده است)
شد.شايد منظور شما اين باشد:
[ برای مشاهده لینک ، لطفا با نام کاربری خود وارد شوید یا ثبت نام کنید ]

abay
12-08-2006, 15:45
با سلام

در مثلث دلخواه زیر، M وسط BC و I وسط AM است. BI را ادامه دهید تا AC را در D قطع کند. ثابت کنید مساحت مثلث ABC ، دوازده برابر مساحت مثلث AID است.

موفق باشید.


[ برای مشاهده لینک ، لطفا با نام کاربری خود وارد شوید یا ثبت نام کنید ]

ارسال متن: شنبه 21 مرداد 1385
سلام
پس شكل مثلثه كو؟ آقاي مفيدي اگه ميشه عكسهاي مربوط به سوالات را در جاي ديگري آپلود كنيد آخه عكسهايي كه در اين جا قرار داده ايد همه ----- شده اند

mofidy1
12-08-2006, 18:30
با سلام

شکل مثلث جای دیگر آپلود شد. اگر باز مشکلی در دیدن این شکل دارید اطلاع دهید.

متشکرم.

ali1234
13-08-2006, 09:15
سلام
نظمي بين اعداد داده شده وجود ندارد كه بتوان متوجه بقيه اعداد (كه به جاي انها نقطه چين گذاشته شده است)
شد.شايد منظور شما اين باشد:
[ برای مشاهده لینک ، لطفا با نام کاربری خود وارد شوید یا ثبت نام کنید ]

سلام دوست عزيز دقيقا منظورم همان چيزي است كه شما نوشتيد.

ali_hp
13-08-2006, 16:54
فرض كنيد
[ برای مشاهده لینک ، لطفا با نام کاربری خود وارد شوید یا ثبت نام کنید ]
مي خواهيم ثابت كنيم [ برای مشاهده لینک ، لطفا با نام کاربری خود وارد شوید یا ثبت نام کنید ]
برای هر n داریم:
[ برای مشاهده لینک ، لطفا با نام کاربری خود وارد شوید یا ثبت نام کنید ]
اگر در نامساوی بالا به جای n مقادیر 1و3و...97و99 را قرار دهیم و نامساوی های بدست امده را در هم ضرب کنیم نتیجه می شود:
[ برای مشاهده لینک ، لطفا با نام کاربری خود وارد شوید یا ثبت نام کنید ]
از طرفی داریم:
[ برای مشاهده لینک ، لطفا با نام کاربری خود وارد شوید یا ثبت نام کنید ]
پس:
[ برای مشاهده لینک ، لطفا با نام کاربری خود وارد شوید یا ثبت نام کنید ]

abay
13-08-2006, 20:52
با سلام

در مثلث دلخواه زیر، M وسط BC و I وسط AM است. BI را ادامه دهید تا AC را در D قطع کند. ثابت کنید مساحت مثلث ABC ، دوازده برابر مساحت مثلث AID است.

موفق باشید.


[ برای مشاهده لینک ، لطفا با نام کاربری خود وارد شوید یا ثبت نام کنید ]

ارسال متن: شنبه 21 مرداد 1385
سلام
[ برای مشاهده لینک ، لطفا با نام کاربری خود وارد شوید یا ثبت نام کنید ]

ali1234
14-08-2006, 09:05
فرض كنيد
[ برای مشاهده لینک ، لطفا با نام کاربری خود وارد شوید یا ثبت نام کنید ]
مي خواهيم ثابت كنيم [ برای مشاهده لینک ، لطفا با نام کاربری خود وارد شوید یا ثبت نام کنید ]
برای هر n داریم:
[ برای مشاهده لینک ، لطفا با نام کاربری خود وارد شوید یا ثبت نام کنید ]
اگر در نامساوی بالا به جای n مقادیر 1و3و...97و99 را قرار دهیم و نامساوی های بدست امده را در هم ضرب کنیم نتیجه می شود:
[ برای مشاهده لینک ، لطفا با نام کاربری خود وارد شوید یا ثبت نام کنید ]
از طرفی داریم:
[ برای مشاهده لینک ، لطفا با نام کاربری خود وارد شوید یا ثبت نام کنید ]
پس:
[ برای مشاهده لینک ، لطفا با نام کاربری خود وارد شوید یا ثبت نام کنید ]
دستت درد نكنه
به سوال ديگه هم داشتم.

آيا تابعي از R اشتراك آن با بازه بسته [0,1] بهR وجود دارد كه مجموعه نقاط ناپيوستگي آن مجموعه اعداد گنگ اشتراك آن با بازه بسته [0,1] باشد؟ يا مثال بياوريد يا ثابت كنيد وجود ندارد.

.
با تشكر .
خدا نگهدار.

mofidy1
14-08-2006, 14:51
دستت درد نكنه
به سوال ديگه هم داشتم.

آيا تابعي از R اشتراك آن با بازه بسته [0,1] بهR وجود دارد كه مجموعه نقاط ناپيوستگي آن مجموعه اعداد گنگ اشتراك آن با بازه بسته [0,1] باشد؟ يا مثال بياوريد يا ثابت كنيد وجود ندارد.

.
با تشكر .
خدا نگهدار.

با سلام

علی آقا سوال شما کمی گنگ است. احتمالا سوال شما این است که آیا تابعی حقیقی با دامنه [0,1] وجود دارد که در نقاط گنگ ناپیوسته و در نقاط گویا پیوسته باشد؟

باید عرض کنم چنین تابعی وجود ندارد. اثباتی مقدماتی برای آن سراغ ندارم. اثبات فنی آن در درس آنالیز ریاضی و معمولا به وسیله مفهوم «نوسان یک تابع بر یک مجموعه» ارائه می شود.

موفق باشید.

Mehdi_Full
14-08-2006, 22:40
دوستان عزيز سه تا سوال در مورد رياضيات دارم كه اميدوارم منو راهنمايي بفرماييد البته براي خودم نميخوام براي يكي از اقوام كه دانشجو هستش ميخوام .
1- آيا تعريف دقيق و كاملي براي نقطه كه خط از آن تشكيل شده در رياضيات وجود داره يا نه ؟ اگه مقاله اي هم هست لطفا لينكشو بزارين
2- آيا اثبات اينكه زواياي داخلي مثلث 180 درجه هست وجود داره يا نه ؟
3- در مورد رياضيات نظريه ريسمانها و ابر ريسمانها اگه اطلاعاتي و يا مقاله اي دارين بزارين
خيلي ممنون ميشم

mofidy1
15-08-2006, 00:40
دوستان عزيز سه تا سوال در مورد رياضيات دارم كه اميدوارم منو راهنمايي بفرماييد البته براي خودم نميخوام براي يكي از اقوام كه دانشجو هستش ميخوام .
1- آيا تعريف دقيق و كاملي براي نقطه كه خط از آن تشكيل شده در رياضيات وجود داره يا نه ؟ اگه مقاله اي هم هست لطفا لينكشو بزارين
2- آيا اثبات اينكه زواياي داخلي مثلث 180 درجه هست وجود داره يا نه ؟
3- در مورد رياضيات نظريه ريسمانها و ابر ريسمانها اگه اطلاعاتي و يا مقاله اي دارين بزارين
خيلي ممنون ميشم

با سلام

- در ریاضیات مفهوم نقطه، مفهومی «تعریف نشده» است و همانند خط و صفحه تعریفی برای آن ارائه نمی شود.

- اثبات اينكه زواياي داخلي مثلث 180 درجه است به عنوان یک قضیه ساده در کتاب هندسه 1 دبیرستان موجود است.

- فکر می کنم سوال سوم را بهتر است در بخش فیزیک p30world مطرح فرمایید با آدرس زیر:

[ برای مشاهده لینک ، لطفا با نام کاربری خود وارد شوید یا ثبت نام کنید ]

موفق باشید.

ali1234
15-08-2006, 10:28
سلام آقاي مفيدي .
در مورد سوال قبلي هر زماني كه وقت كردي جوابشو بذاري ممنونت ميشم.امادوسوال ديگه:
1-آيا دامنه يك تابع مي تواند مجموعه تهي باشد؟

2-نشان دهيد از بين تمام مستطيل هاي با محيط معلوم P ؛مربع بيشترين مساحت را دارد.اين مساحت چقدر است.؟

abay
15-08-2006, 13:23
سلام آقاي مفيدي .
در مورد سوال قبلي هر زماني كه وقت كردي جوابشو بذاري ممنونت ميشم.امادوسوال ديگه:
1-آيا دامنه يك تابع مي تواند مجموعه تهي باشد؟

2-نشان دهيد از بين تمام مستطيل هاي با محيط معلوم P ؛مربع بيشترين مساحت را دارد.اين مساحت چقدر است.؟
سلام
[ برای مشاهده لینک ، لطفا با نام کاربری خود وارد شوید یا ثبت نام کنید ]

ali_hp
15-08-2006, 18:02
سلام
لطفا کمی درباره چگونگی اثبات رابطه های محیط ومساحت دایره توضیح دهید.
متشکرم.

ali_hp
15-08-2006, 19:30
سلام
اگر x,y,z اعداد حقيقي ومثبت باشند چنانكه x+y+z=1 آنگاه ثابت كنيد كه:

[ برای مشاهده لینک ، لطفا با نام کاربری خود وارد شوید یا ثبت نام کنید ]
سلام
هیچکس راه حلی برای این سوال ندارد؟؟؟

ali1234
16-08-2006, 09:23
با سلام
با تشكر از 136784 . فقط يه اشكال جزئي داشت : x=p/4,
s=p^2/16
اما يه سوال:
نشان دهيد تمام مثلث هاي قائم الزاويه داراي زاويه حاده يكسان؛ متشابه اند.

mofidy1
16-08-2006, 14:06
سلام
اگر x,y,z اعداد حقيقي ومثبت باشند چنانكه x+y+z=1 آنگاه ثابت كنيد كه:

[ برای مشاهده لینک ، لطفا با نام کاربری خود وارد شوید یا ثبت نام کنید ]

با سلام

دوستانی که علاقمند به حل این مساله هستند به لینک زیر مراجعه کنند. اگر فیلتر شده بود اطلاع دهید. ضمنا برای مطالعه مباحث لینک زیر احتیاج به معلوماتی در زمینه « نامساوی ینسن» دارید.

[ برای مشاهده لینک ، لطفا با نام کاربری خود وارد شوید یا ثبت نام کنید ]

موفق باشید.

mofidy1
16-08-2006, 15:15
سلام
لطفا کمی درباره چگونگی اثبات رابطه های محیط ومساحت دایره توضیح دهید.
متشکرم.

با سلام

علی آقا اثبات این رابطه ها معمولا در هر کتاب ریاضیات عمومی دانشگاهی وجود دارد. به طور خلاصه خدمتتان عرض کنم که به طور معمول برای تعیین طول قوس یک منحنی که مختصات قطبی آن در دست باشد و نیز برای تعیین مساحت محصور بین دو تابع پیوسته با مختصات دکارتی از دو فرمول زیر استفاده می کنند. در اولی محدوده ار زاویه آلفا تا زاویه بتاست.

برای به دست آوردن محیط دایره در فرمول اولی آلفا را صفر و بتا را 2Pi بگیرید و دیفرانسیل زیر رادیکال را هم صفر (زیرا شعاع دایره ثابت است).

برای به دست آوردن مساحت دایره در فرمول دومی a را صفر، b را r و g را تابع صفر و f را ریشه دوم تابع «r^2 منهای x^2» بگیرید که r شعاع دایره y^2+x^2=r^2 است. در نهایت حاصل را در عدد چهار ضرب کنید.

موفق باشید.


[ برای مشاهده لینک ، لطفا با نام کاربری خود وارد شوید یا ثبت نام کنید ]

ali_hp
16-08-2006, 22:22
با سلام

علی آقا اثبات این رابطه ها معمولا در هر کتاب ریاضیات عمومی دانشگاهی وجود دارد. به طور خلاصه خدمتتان عرض کنم که به طور معمول برای تعیین طول قوس یک منحنی که مختصات قطبی آن در دست باشد و نیز برای تعیین مساحت محصور بین دو تابع پیوسته با مختصات دکارتی از دو فرمول زیر استفاده می کنند. در اولی محدوده ار زاویه آلفا تا زاویه بتاست.

برای به دست آوردن محیط دایره در فرمول اولی آلفا را صفر و بتا را 2Pi بگیرید و دیفرانسیل زیر رادیکال را هم صفر (زیرا شعاع دایره ثابت است).

برای به دست آوردن مساحت دایره در فرمول دومی a را صفر، b را یک، g تابع صفر و f را ریشه دوم تابع «یک منهای x^2». در نهایت حاصل را در عدد چهار ضرب کنید.

موفق باشید.


[ برای مشاهده لینک ، لطفا با نام کاربری خود وارد شوید یا ثبت نام کنید ]
سلام
ممنونم اقاي مفيدي.عكس اين پست براي من ----- شده است.

mofidy1
17-08-2006, 00:43
سلام
ممنونم اقاي مفيدي.عكس اين پست براي من ----- شده است.

با سلام

علی آقا عکس را جای دیگری آپلود کردم. اگر باز هم مشکلی پیش آمد اطلاع دهید. راستی آیا توضیحات بالا برایتان مفید بود؟

abay
17-08-2006, 00:50
با سلام

علی آقا اثبات این رابطه ها معمولا در هر کتاب ریاضیات عمومی دانشگاهی وجود دارد. به طور خلاصه خدمتتان عرض کنم که به طور معمول برای تعیین طول قوس یک منحنی که مختصات قطبی آن در دست باشد و نیز برای تعیین مساحت محصور بین دو تابع پیوسته با مختصات دکارتی از دو فرمول زیر استفاده می کنند. در اولی محدوده ار زاویه آلفا تا زاویه بتاست.

برای به دست آوردن محیط دایره در فرمول اولی آلفا را صفر و بتا را 2Pi بگیرید و دیفرانسیل زیر رادیکال را هم صفر (زیرا شعاع دایره ثابت است).

برای به دست آوردن مساحت دایره در فرمول دومی a را صفر، b را یک، g تابع صفر و f را ریشه دوم تابع «یک منهای x^2». در نهایت حاصل را در عدد چهار ضرب کنید.

موفق باشید.


[ برای مشاهده لینک ، لطفا با نام کاربری خود وارد شوید یا ثبت نام کنید ]
سلام
اين سواليكه توسط جناب ali_hpپرسيده شده سواليكه منم قبلا" از اشخاص مختلف (دبير رياضيمان ودوستان واستادان دانشگاه و.......)پرسيدم وبه نتايج زياد جالبي نرسيدم حالا هم كه شما به اين سوال جواب داديدنمي دانم كه ali_hpجواب سوالشان را پيدا كرده اند يا نه ولي من كه نه و من اين روشهايي را كه شما ارايه كرده ايد وحداقل دو يا سه روش ديگر بامباحث ديگري از رياضيات(ماننداثبات به وسيله ي اعداد مختلط و....)را بلد بودم در واقع اين مبحث ها را مي توان گفت كه دستاورد هاي جديد رياضي بود كه در اواخر قرن 18و 19توسط نيوتن ولايبنيتز ارايه شد در حاليكه دايره و عوامل مربوط به آن به زمانهاي خيلي قبل و حتي پيش از ميلاد مسيح مي رسد كه توسط دانشمندان يوناني ارايه شد ومن ميخواستم بدانم رابطه هايي كه اينها براي مساحت ومحيط دايره بدست آورند چگونه به اينها رسيدند خب مسلما" آن زمان مباحثي مانند انتگرال وجود نداشت ويا حداقل كامل نبودند ومن تا آنجا كه اطلاعاتي بدست آورده ام براي بدست آوردن مساحت دايره سطح محصور به دايره را به nقطعه همشكل با مساحتهاي مساوي تقسيم ميكرده اند وبا بدست آوردن مساحتهاي اين قطعه ها مساحت دايره را تقريبا" با اين مساحتها برابر مي دانسته اند.حالا اگه آقاي مفيدي وبقيه ي دوستان اطلا عات بيشتر ي داريد كه مسلما" همينطور است در اختيار ما قرار دهيد.

ali_hp
17-08-2006, 12:16
با سلام

علی آقا عکس را جای دیگری آپلود کردم. اگر باز هم مشکلی پیش آمد اطلاع دهید. راستی آیا توضیحات بالا برایتان مفید بود؟
سلام
من فكر مي كنم تعريف راديان مبتني بر اين است:
عدد ثابت K وجود دارد كه براي هر R مثبت محيط دايره اي به شعاع R مي شود KR .
با اين حساب ايا مي توان براي ثابت كردن فرمول مربوط به محيط دايره از توابع مثلثاتي كه متغير انها بر اساس راديان تعريف شده است استفاده كرد؟
در باره رابطه دوم:
من با يك تغيير متغير مثلثاتي مقدار انتگرال ودر نتيجه مساحت دايره را محاسبه كردم.فقط مشكلي كه دارم اين است
روشي كه من براي بدست اوردن مشتق sin وcos بلدم همان روش كتاب حسابان است كه ابتدا در كتاب حسابان قضيه هم ارزي را ثابت كرده اند ودر اثبات ارايه شده براي ان از اينكه مساحت دايره مي شود پي اردو استفاده شده است و بعد با استفاده از قضيه هم ارزي مشتق توابع مختلف مثلثاتي را بدست اورده است.

mofidy1
17-08-2006, 15:45
با سلام

علی آقا بنده سر فرصت روشی بسیار ساده برای اثبات فرمول مساحت دایره بدون استفاده از مشتق و انتگرال ارائه خواهم کرد. حداکثر چیزی که بنده در این روش از آن استفاده می کنم همان قضیه هم ارزی است.

موفق باشید.

mofidy1
18-08-2006, 11:27
با سلام

در مثلث دلخواه زیر، M وسط BC و I وسط AM است. BI را ادامه دهید تا AC را در D قطع کند. ثابت کنید مساحت مثلث ABC ، دوازده برابر مساحت مثلث AID است.

موفق باشید.


[ برای مشاهده لینک ، لطفا با نام کاربری خود وارد شوید یا ثبت نام کنید ]

ارسال متن: شنبه 21 مرداد 1385

با سلام

از دوست خوبمان 136784 برای حل این مساله تشکر می کنیم. ایشان با اینکه مدت کوتاهی است با اتاق ریاضیات همکاری دارند اما در این مدت بسیار فعال بوده اند. برای دیدن راه حل ایشان به اینجا ([ برای مشاهده لینک ، لطفا با نام کاربری خود وارد شوید یا ثبت نام کنید ]) مراجعه فرمایید. بنده راه حل دیگری را ارائه میکنم.

به شکل زیر توجه کنید.


[ برای مشاهده لینک ، لطفا با نام کاربری خود وارد شوید یا ثبت نام کنید ]

چون میانه، مثلث را به دو قسمت هم مساحت تقسیم می کند بنابر این مساحت مثلث ABC چهار برابر AIC است. چون دو مثلث AIC و َAID دارای ارتفاع مشترک CH هستند کافیست ثابت کنیم AC سه برابر AD است. از I خطی موازی BC رسم می کنیم تا AC را در 'I قطع کند. چون I وسط AM است پس MC دو برابر I'I و لذا BC چهار برابر I'I است. از تشابه دو مثلث DI'I و DBC داریم:


[ برای مشاهده لینک ، لطفا با نام کاربری خود وارد شوید یا ثبت نام کنید ]

از D خطی موازی AM رسم می کنیم تا BC را در "D قطع کند. از تشابه دو مثلث AMC و D"DC داریم:


[ برای مشاهده لینک ، لطفا با نام کاربری خود وارد شوید یا ثبت نام کنید ]

اما دو مثلث BD"D و BIM نیز متشابهند، لذا


[ برای مشاهده لینک ، لطفا با نام کاربری خود وارد شوید یا ثبت نام کنید ]

موفق باشید.

ارسال متن: جمعه 27 مرداد 1385

ali_hp
18-08-2006, 20:38
با سلام

علی آقا بنده سر فرصت روشی بسیار ساده برای اثبات فرمول مساحت دایره بدون استفاده از مشتق و انتگرال ارائه خواهم کرد. حداکثر چیزی که بنده در این روش از آن استفاده می کنم همان قضیه هم ارزی است.

موفق باشید.
سلام
من مي توانم با فرض درست گرفتن قضيه هم ارزي فرمول مربوط به مساحت دايره را ثابت كنم اما همانطور كه در پست 225 گفتم قضيه هم ارزي را هم فقط با استفاده از فرمول مساحت دايره توانسته ام ثابت كنم .
يك سوال ديگرهم در پست 225 پرسيده بودم(در باره استفاده از روابط مثلثاتي براي اثبات فرمول مربوط به محيط دايره).
متشكرم.

mofidy1
18-08-2006, 21:11
سلام
من مي توانم با فرض درست گرفتن قضيه هم ارزي فرمول مربوط به مساحت دايره را ثابت كنم اما همانطور كه در پست 225 گفتم قضيه هم ارزي را هم فقط با استفاده از فرمول مساحت دايره توانسته ام ثابت كنم .
يك سوال ديگرهم در پست 225 پرسيده بودم(در باره استفاده از روابط مثلثاتي براي اثبات فرمول مربوط به محيط دايره).
متشكرم.

با سلام

علی آقا به نکته زیبایی اشاره کرده اید. در واقع این دو قضیه هم ارز هستند. یعنی اگر فرمول مساحت یک قطاع دایره را قبول کنیم میتوان به وسیله آن قضیه هم ارزی را ثابت کرد و بالعکس. به همین دلیل و البته دلایل دیگر، ارائه این برهان را در درس حسابان مناسب نمی دانم. قبلا هم مفصلا در باره برهان دیگر برای این قضیه با جنابعالی صحبت کرده ایم.

سوال دوم شما را کاملا متوجه نشدم. معمولا در معالات یا اتحادهای ریاضی به واحد سنجش دو طرف کاری نداریم و منظور ما این است که به طور مطلق دو طرف برابرند یا می خواهیم برابر باشند. به طور مثال 1=1 اما 1 کیلو با 1 گرم برابر نیست.

موفق باشید.

ali_hp
19-08-2006, 01:53
با سلام

علی آقا به نکته زیبایی اشاره کرده اید. در واقع این دو قضیه هم ارز هستند. یعنی اگر فرمول مساحت یک قطاع دایره را قبول کنیم میتوان به وسیله آن قضیه هم ارزی را ثابت کرد و بالعکس. به همین دلیل و البته دلایل دیگر، ارائه این برهان را در درس حسابان مناسب نمی دانم. قبلا هم مفصلا در باره برهان دیگر برای این قضیه با جنابعالی صحبت کرده ایم.

سوال دوم شما را کاملا متوجه نشدم. معمولا در معالات یا اتحادهای ریاضی به واحد سنجش دو طرف کاری نداریم و منظور ما این است که به طور مطلق دو طرف برابرند یا می خواهیم برابر باشند. به طور مثال 1=1 اما 1 کیلو با 1 گرم برابر نیست.

موفق باشید.
سلام
فرمول مربوط به محيط دايره در حقيقت مي گو يد نسبت محيط يك دايره به شعاع ان مقداري ثابت است.
حالا فرض كنيد راديان و درجه هر دو واحدهايي براي سنجش زاويه هستند(منظور من همان واحدهاي شناخته شده است) حالا يك زاويه 360 درجه در نظر بگيريد و فرض كنيد زاويه هاي 360 درجه برابر x راديان باشند.
يك دايره دلخواه به مركز راس زاويه رسم مي كنيم دقت كنيد كه كمان رو برو به زاويه 360 درجه برابر با محيط دايره است از طرفي بنابر تعريف راديان مقدار يك زاويه برحسب راديان برابر است با طول كمان روبرو به ان تقسيم بر شعاع دايره.پس با توجه به تعريف راديان:
دايره ما دلخواه بود ولي نسبت محيط ان(كه كمان روبرو به زاويه 360درجه است) به شعاعش مي شود مقدار زاويه
360 درجه بر حسب راديان كه به وضوح عددي ثابت است.
نظرتان در رابطه با اين اثبات چيست؟

mofidy1
19-08-2006, 07:21
با سلام

علی آقا برهان شما تقریبا همان برهانی است که در کتاب ریاضی 1 دبیرستان بعد از معرفی مفهوم رادیان برای دانش آموزان تشریح می شود. اجازه دهید سوال دیگری مطرح کنم که البته خیلی به این بحث ما ارتباطی ندارد. فکر می کنید بشر چگونه به این مطلب پی برد که اگر محیط هر دایره را بر قطر آن تقسیم کنیم همواره به یک عدد منحصر به فرد می رسیم (که ما آن را «عدد پی» می خوانیم) ؟ آیا فقط به طور شهودی این مطلب را پذیرفت یا استدلالی درستی هم برای آن داشت؟

موفق باشید.

mofidy1
19-08-2006, 07:46
با سلام

در پست 164 آقای منبتی مساله ی زیبایی را مطرح کردند که وقتی به حل آن مشغول شدم زیبایی آن برایم دو چندان شد. متن مساله بسیار ساده و به ظاهر پیش پا افتاده است، اما فکر نمی کنم از روشهای ساده و معمولی قابل حل باشد. اگر دوستان راه حلی برای آن هر چند طولانی پیدا کردند در اینجا مطرح فرمایند. متشکرم.

مساله:

ثابت کنید تابعی با شرایط زیر وجود ندارد:


[ برای مشاهده لینک ، لطفا با نام کاربری خود وارد شوید یا ثبت نام کنید ]

موفق باشید.

ارسال متن: شنبه 28 مرداد 1385

ali_hp
19-08-2006, 20:37
با سلام

علی آقا برهان شما تقریبا همان برهانی است که در کتاب ریاضی 1 دبیرستان بعد از معرفی مفهوم رادیان برای دانش آموزان تشریح می شود. اجازه دهید سوال دیگری مطرح کنم که البته خیلی به این بحث ما ارتباطی ندارد. فکر می کنید بشر چگونه به این مطلب پی برد که اگر محیط هر دایره را بر قطر آن تقسیم کنیم همواره به یک عدد منحصر به فرد می رسیم (که ما آن را «عدد پی» می خوانیم) ؟ آیا فقط به طور شهودی این مطلب را پذیرفت یا استدلالی درستی هم برای آن داشت؟

موفق باشید.
منظور من از اوردن چنین چیزی به عنوان اثبات این بود که اگربرای اثبات فرمول محیط دایره مجاز به استفاده از مفهوم رادیان باشیم (در روشی که شما برای محاسبه محیط دایره پیشنهاد کرده اید از این مفهوم استفاده می شود)
خوب می توان اثبات پست 230 را ارایه کرد که بسیار ساده تر است.
من فکر می کنم روشی که شما برای اثبات فرمول محیط دایره پیشنهاد کرده اید یک دور است یعنی در مراحل قبلی ان بارها از اینکه نسبت محیط یک دایره به شعاعش مقداری ثابت است استفاده شده است .
در حقیقت لازمه انکه در مراحل مختلف اثبات شما عبارات معنی دار باشند این است که نسبت محیط یک دایره به شعاعش مقداری ثابت باشد.
در باره سوالی که پر سیدید من اطلاعی ندارم ولی ممکن است از اینکه همه دایره ها متشابه ومتجانس هستند به شکلی نه چندان دقیق این نتیجه را گرفته باشند.
اگر ممکن است بگویید چه تعریفی برای طول یک منحنی وجود دارد؟
متشکرم.

mofidy1
19-08-2006, 22:27
با سلام

اشکالی را که وارد کرده اید تقریبا قبول دارم. هدف بنده از فرمولها ارائه روشی برای محاسبه طول قوس منحنی های کلی تر بود.

«یک خط و یک منحنی ساده را همنهشت گوییم اگر بتوان فقط با تغییر دلخواه انحنای خط، این دو را روی یکدیگر منطبق کرد. در این صورت گوییم این دو هم طول هستند.»

موفق باشید.

ali_hp
20-08-2006, 00:33
با سلام

اشکالی را که وارد کرده اید تقریبا قبول دارم. هدف بنده از فرمولها ارائه روشی برای محاسبه طول قوس منحنی های کلی تر بود.

«یک خط و یک منحنی ساده را همنهشت گوییم اگر بتوان فقط با تغییر دلخواه انحنای خط، این دو را روی یکدیگر منطبق کرد. در این صورت گوییم این دو هم طول هستند.»

موفق باشید.
سلام
منظور از تغيير در انحناي خط چيست ؟ اصلا انحنا چيست؟
اگر مفهوم انحنا مفهوم مقدماتي نيست و پيش نياز هاي زيادي دارد لطفا يك منبع مناسب در اين باره معرفي كنيد.
متشكرم.

mofidy1
20-08-2006, 16:24
سلام
منظور از تغيير در انحناي خط چيست ؟ اصلا انحنا چيست؟
اگر مفهوم انحنا مفهوم مقدماتي نيست و پيش نياز هاي زيادي دارد لطفا يك منبع مناسب در اين باره معرفي كنيد.
متشكرم.

با سلام

بله علی آقا اگر بخواهید تعریف دقیق انحنا را در اینجا عرض کنم، احتیاج به مقدمات نسبتا پیشرفته ای دارد (به کتاب حساب دیفرانسیل و انتگرال و هندسه تحلیلی «لوئیس لیتهلد Louis Leithold» ترجمه دکتر مهدی بهزاد و ... ، جلد دوم صفحه 480 مراجعه کنید). اما انحنای خط به طور شهودی کاملا قابل فهم است و به طور خودمانی همان کج و کولگی یک منحنی است. اگر بتوانید به یک خط یا پاره خط بدون اینکه آن را بفشارید یا از دو طرف بکشید، طوری انحنا دهید که بتوان روی منحنی مورد نظر گذاشت تا آنرا کاملا بپوشاند آنگاه گوییم این دو «هم طول» هستند. با هر واحدی طول خط اولی را بسنجید با همان هم منحنی مورد نظر را خواهیم سنجید.

موفق باشید.

ali_hp
21-08-2006, 01:11
با سلام

دو مساله جالب:

1) دوستان به نظر شما ((sin(cos(x بزرگتراست یا ((cos(sin(x ؟

2) ریشه سوم 60 بزرگتر است یا 2 به اضافه ریشه سوم 7 ؟ (بدون استفاده از ماشین حساب!)


موفق باشید.
سلام
1)دومی بزرگتر است.
[ برای مشاهده لینک ، لطفا با نام کاربری خود وارد شوید یا ثبت نام کنید ]
دقت کنید که:
[ برای مشاهده لینک ، لطفا با نام کاربری خود وارد شوید یا ثبت نام کنید ]
[ برای مشاهده لینک ، لطفا با نام کاربری خود وارد شوید یا ثبت نام کنید ]
اما برای هر عدد حقیقی A که:
[ برای مشاهده لینک ، لطفا با نام کاربری خود وارد شوید یا ثبت نام کنید ]
خواهیم داشت :
[ برای مشاهده لینک ، لطفا با نام کاربری خود وارد شوید یا ثبت نام کنید ]
وباتوجه به اینکه :
[ برای مشاهده لینک ، لطفا با نام کاربری خود وارد شوید یا ثبت نام کنید ]
نتیجه می شود:
[ برای مشاهده لینک ، لطفا با نام کاربری خود وارد شوید یا ثبت نام کنید ]
پس خواهیم داشت:
[ برای مشاهده لینک ، لطفا با نام کاربری خود وارد شوید یا ثبت نام کنید ]

ali_hp
21-08-2006, 01:45
با سلام

بله علی آقا اگر بخواهید تعریف دقیق انحنا را در اینجا عرض کنم، احتیاج به مقدمات نسبتا پیشرفته ای دارد (به کتاب حساب دیفرانسیل و انتگرال و هندسه تحلیلی «لوئیس لیتهلد Louis Leithold» ترجمه دکتر مهدی بهزاد و ... ، جلد دوم صفحه 480 مراجعه کنید). اما انحنای خط به طور شهودی کاملا قابل فهم است و به طور خودمانی همان کج و کولگی یک منحنی است. اگر بتوانید به یک خط یا پاره خط بدون اینکه آن را بفشارید یا از دو طرف بکشید، طوری انحنا دهید که بتوان روی منحنی مورد نظر گذاشت تا آنرا کاملا بپوشاند آنگاه گوییم این دو «هم طول» هستند. با هر واحدی طول خط اولی را بسنجید با همان هم منحنی مورد نظر را خواهیم سنجید.

موفق باشید.
سلام
ممنونم اقای مفیدی.
در پست 231 سوال جالبی را مطرح کرده اید که فکر می کنم با سوالی که دوست عزیز 136784 در پست 224 پرسیده اند نیز در ارتباط باشد.اگر ممکن است خودتان به این سوال جواب دهید.
متشکرم.

abay
21-08-2006, 14:45
[ برای مشاهده لینک ، لطفا با نام کاربری خود وارد شوید یا ثبت نام کنید ]
سلام
البته در این قسمت بزرگتر بودن پی دوم از رادیکال دو بدیهیه ولی من میخواستم بدانم شما چطوراز این نامساویهایی که نوشتید این نتیجه را گرفتید من که هر چه دقت کردم متوجه نشدم

ali_hp
21-08-2006, 15:20
[ برای مشاهده لینک ، لطفا با نام کاربری خود وارد شوید یا ثبت نام کنید ]
سلام
البته در این قسمت بزرگتر بودن پی دوم از رادیکال دو بدیهیه ولی من میخواستم بدانم شما چطوراز این نامساویهایی که نوشتید این نتیجه را گرفتید من که هر چه دقت کردم متوجه نشدم
سلام
فکر کنم اینجوری بهتر باشه:
[ برای مشاهده لینک ، لطفا با نام کاربری خود وارد شوید یا ثبت نام کنید ]

abay
21-08-2006, 15:57
سلام
فکر کنم اینجوری بهتر باشه:
[ برای مشاهده لینک ، لطفا با نام کاربری خود وارد شوید یا ثبت نام کنید ]
سلام
خیلی ممنون فکر کنم شما هم قبول داشته باشید بعضی وقتها پیش بیاد که آدم بتونه مسایل خیلی مشکل رو حل کنه ویا در مسایل خیلی ساده ای مثل این گیر کنه

abay
21-08-2006, 16:09
با سلام

دو مساله جالب:

1) دوستان به نظر شما ((sin(cos(x بزرگتراست یا ((cos(sin(x ؟

2) ریشه سوم 60 بزرگتر است یا 2 به اضافه ریشه سوم 7 ؟ (بدون استفاده از ماشین حساب!)


موفق باشید.
[ برای مشاهده لینک ، لطفا با نام کاربری خود وارد شوید یا ثبت نام کنید ]
[ برای مشاهده لینک ، لطفا با نام کاربری خود وارد شوید یا ثبت نام کنید ]

ali_hp
21-08-2006, 17:02
با سلام

دو مساله جالب:

1) دوستان به نظر شما ((sin(cos(x بزرگتراست یا ((cos(sin(x ؟

2) ریشه سوم 60 بزرگتر است یا 2 به اضافه ریشه سوم 7 ؟ (بدون استفاده از ماشین حساب!)


موفق باشید.
سلام
2) طبق اتحاد اویلر داریم:
[ برای مشاهده لینک ، لطفا با نام کاربری خود وارد شوید یا ثبت نام کنید ]
حالا فرض کنید:
[ برای مشاهده لینک ، لطفا با نام کاربری خود وارد شوید یا ثبت نام کنید ]
می خواهیم علامت
[ برای مشاهده لینک ، لطفا با نام کاربری خود وارد شوید یا ثبت نام کنید ]
را تعیین کنیم که طبق اتحاد اویلر با
[ برای مشاهده لینک ، لطفا با نام کاربری خود وارد شوید یا ثبت نام کنید ]
هم علامت است. ثابت می کنیم که مثبت است. دقت کنیدکه:
[ برای مشاهده لینک ، لطفا با نام کاربری خود وارد شوید یا ثبت نام کنید ]
اما داریم:
[ برای مشاهده لینک ، لطفا با نام کاربری خود وارد شوید یا ثبت نام کنید ]
[ برای مشاهده لینک ، لطفا با نام کاربری خود وارد شوید یا ثبت نام کنید ]

ali_hp
21-08-2006, 18:54
[ برای مشاهده لینک ، لطفا با نام کاربری خود وارد شوید یا ثبت نام کنید ]
[ برای مشاهده لینک ، لطفا با نام کاربری خود وارد شوید یا ثبت نام کنید ]
سلام
در سوال دوم رابطه ای که استفاده کرده اید تساوی نیست همانطور که نوشته اید دو طرف رابطه تقریبا با هم برابرند.
پس دو عدد فرجه سوم شصت ودو باضافه فرجه سوم هفت تقریبا با 12/47 برابرند.وممکن است مقدار واقعی انها از
12/47 کمی بیشتر یا کمتر باشد ( که البته هر دو عدد از 12/47 کمتر هستند)
در حقیقت دوعدد فرجه سوم شصت ودو باضافه فرجه سوم هفت اختلافی کمتر از یک صدم دارند.

ali_hp
21-08-2006, 19:01
سلام
خیلی ممنون فکر کنم شما هم قبول داشته باشید بعضی وقتها پیش بیاد که آدم بتونه مسایل خیلی مشکل رو حل کنه ویا در مسایل خیلی ساده ای مثل این گیر کنه
سلام
100%تایید میشه.
البته توضیح قبلی من برای این مساله مبهم بود.

mofidy1
21-08-2006, 21:44
با سلام

دو مساله جالب:

1) دوستان به نظر شما ((sin(cos(x بزرگتراست یا ((cos(sin(x ؟

2) ریشه سوم 60 بزرگتر است یا 2 به اضافه ریشه سوم 7 ؟ (بدون استفاده از ماشین حساب!)


موفق باشید.

با سلام

از اینکه می بینم دوستان به این زیبایی روی مسائل بحث می کنند بسیار خوشحالم . به ویژه از این دو دوست گرامی (ali_hp و 136784) تشکر می کنم. با نرم افزار maple شکل این دو تابع را رسم و برایتان آپلود کردم تا صحت راه حلها مشخص شود.

شکل را در پست 249 مشاهده فرمایید. متشکرم.

اما حالت کلی تر مساله دوم:

عبارت زیر نامنفی است یا نامثبت؟ (x و y اعداد نامنفی هستند.)


[ برای مشاهده لینک ، لطفا با نام کاربری خود وارد شوید یا ثبت نام کنید ]

موفق باشید.

ali1234
22-08-2006, 09:54
با سلام وخسته نباشيد خدمت همه عزيزان
يه سوال داشتم :
[ برای مشاهده لینک ، لطفا با نام کاربری خود وارد شوید یا ثبت نام کنید ]
با تشكر.

abay
22-08-2006, 13:25
[QUOTE=mofidy1]

(
[ برای مشاهده لینک ، لطفا با نام کاربری خود وارد شوید یا ثبت نام کنید ]


.[/QUOTEسلام
آقای مفیدی این نموداری که رسم کرده اید من فکر می کردم که اینطور هم میشه به این سوال جواب داد ولی چون خیلی وقت بود که ریاضی مطالعه نمی کردم وتازه شروع کرده ام نتونستم نمودارهای آنها را بکشم واگر برایتان امکان دارد شما نمودارهای آنها را با استفاده از مشتق وجدول تغییرات واین جور چیز ها رسم کنید در ضمن این نموداری که شما با استفاده ازmaple رسم کرده اید چگونه رسم کرده اید اگه میشه یه آموزشی از این نرم افزار مخصوصا" رسم خط وصفحه در فضا توسط این نرم افزار قرار دهید البته ببخشید که این مطلب اینجا نوشتم من این سوال را در صفحه اول هم مطرح کرده بودم ولی مثل اینکه به آن قسمت توجهی نمی شود و توجه همه معطوف به اتاق ریا ضیات است وهمچنین به سوالاتی که من در پست 177 قرار داده بودم هیچ کدام از دوستان جوابی ندارند